Vous êtes sur la page 1sur 125

Indias Fastest Growing Test Prep Portal

Bulls Eye 1
www.hitbullseye.com

CHP 1 SERIES

Series
Now before we proceed to discuss the various types of questions related to alphabetical series, we will talk of
some basic facts which are essential to an understanding of these types of questions,

I. THE ALPHABET: The normal English alphabet contains 26 letters in all, as shown below
ABCDEFGHIJKLMNOPQRSTUVWXYZ

(Usually, questions on alphabet are accompanied by this normal alphabet). From A to M, the alphabet
completes its first half, while the other half starts from N and ends at Z.
A M 1 13 (First Alphabetical Half)
N Z 14 26 (Second Alphabetical Half)

II. EJOTY: For purpose of convenience, it is helpful to remember this simple formula called EJOTY, with
the help of which you can easily find the position of any letter without much effort. But for practical purposes,
you should learn by heart the positions of different letters in the alphabet.
E J O T Y
5 10 15 20 25

Now, for instance, we wish to find the position of, say, the 17th letter from the left side. You already know that
the 15th letter from the left side is O, therefore, the only thing you have to do is to find a letter which is two
positions ahead of O, which is Q (The Answer). Using this simple formula, you can quickly find the position
of any letter from the left side without much brain-rattling. Remembering the positions of different alphabets
is basic to solving any question on alphabetical series, so examinees must remember their positions. One of
the best ways to achieve it is to practice EJOTY. Simply write down the full names of any 200 people you can
imagine and do like follows:

For example, lets say the name of the person imagined is ZUBINA. Now from EJOTY, we know that Z
stands for 26, U stands for 22, B stands for 2, I stands for 9, N stands for 14 and A stands for 1. Now add up
all these positions (26+22+29+14+1). What you get on addition does not have any significance, but it can be a
very good way to try to make out and remember the individual positions of all the letters in the alphabet.

III. Finding Positions: Much more commonly, you get questions in the tests, which provide you alphabetical
positions from the right side. Since we are used to counting from the left side i.e. A, B, C and not Z, Y,
X, the formula we discussed earlier will be applicable with a bit of modification. But before we proceed to
discuss it, it is essential to remember one simple mathematical fact.

*Lets say there is a row of 7 boys in which a boy is standing 3rd from left. We want to know his position from
the right side.
I I I I I I I
1st 2nd 3rd 4th 5th 6th 7th
You can see for yourself that the boy who was 3rd from the left is placed 5th from the right side.
Bulls Eye
2 www.hitbullseye.com

The sum of both the positions is 8 (3 + 5), while the total number of boys is 7. This happens because we are
counting a single boy twice in the calculation process. If we had subtracted 3 from 7 (as some of us might do),
we would have got 4, which is obviously not the correct position from the right side. An important conclusion
emerges from this discussion. If we are dealing with an alphabet and we have been given the position of any
letter from either side, we will add 1 to the total no. of letters and then subtract position from one side to get
its position from the other side. For example, lets find the position from the right of a letter, which is the 9th
from the left side.

A B C D E F G H I J K L M N O P Q R S T U V W X Y Z
1 2 3 4 5 6 7 8 9 10 11 12 13 14 15 16 17 18 19 20 21 22 23 24 25 26
26 25 24 23 2221 20 19 1817 16 15 14 13 12 11 10 9 8 7 6 5 4 3 2 1

As you can see for yourself, the 9th letter from the left side, I, comes out to be the 18th from the right side.
Their sum (9 + 18 = 27) is again one more than the total number of alphabets i.e. 26. We can do this operation
easily by adding one to the total number of letters (26 + 1 = 27) and then subtracting 9 from it. It gives you the
letter position 18th from the right, which you can verify yourself from the above alphabet. The same procedure
will be applicable if we are given an initial right position and are supposed to find it from the left side. Take
for example, a letter, which is placed 11th from the right side. If we want to locate its position from the left
side, we will add 1 to total no. of letters and then subtract the right position from it to get its position from the
left side. 27 11 gives you 16. Using EJOTY, you can easily conclude that the letter is P (16th from left, 11th
from right).

The same logic is applicable if we are dealing with situation in which their position of an item is given to us
from the top and we want to find it from the bottom side and vice-versa.

IV. Still another type of question concerns finding the midpoint between two letters in the alphabet. For
instance, lets talk of a case, which requires us to find the mid-point between the 11th letter and the 17th letter
from the left side.

A B C D E F G H I J K L M N O P Q R S T U V W X Y Z

1 2 3 4 5 6 7 8 9 10 11 12 13 14 15 16 17 18 19 20 21 22 23 24 25 26

26 25 24 23 22 21 20 19 18 17 16 15 14 13 12 11 10 9 8 7 6 5 4 3 2 1

You can see that there are five letters between these two positions i.e. L, M, N, O and P. Obviously, the
midpoint of 5 items is the third item from either side, whether counted from the left or the right. It comes out
to be N, which is the correct answer. But frankly speaking, so much labor is not exactly required in solving
such questions. Lets let the cat out of the bag. In such questions, if the positions are given from the same
side( i.e. either both are from left or both are from right), simply add up the two positions, get their average
and you have the answer. In this case, the two positions are 11 and 17 from left. Adding them and averaging
them gives you 14. Recollect the EJOTY formula and you immediately come up with the letter which is 14th
from the left side (preceding O). The same procedure will be applicable if you are given a case in which both
the positions are counted from the right side. Remember that the answer you will get will be from the same
sides, which you have been given. Lets make this thing clearer by taking a practical example.

*Consider a case in which we have to find the mid-point between the 13th and the 19th letter from the right
side. Adding the two positions gives us 32, the average of which is 16. So we get the mid-point, which is 16th
from the right side (the same as the sides given in the question). Now we have to convert this position into a
Bulls Eye 3
www.hitbullseye.com

position from the left. Applying the logic discussed earlier, we subtract 16 from 27 and get 11th from the left,
which is obviously K. You can verify this answer by looking up the above alphabet. In fact, for such
questions, one should have so much practice that one does not need to look up the alphabet, which proves to
be time-consuming.
A B C D E F G H I J K L M N O P Q R S T U V W X Y Z

1 2 3 4 5 6 7 8 9 10 11 12 13 14 15 16 17 18 19 20 21 22 23 24 25 26

26 25 24 23 22 21 20 19 18 17 16 15 14 13 12 11 10 9 8 7 6 5 4 3 2 1

Now let us consider the third case in which we have to find the mid-point between two alphabetical positions,
one of which is given from the left and the other from the right.

*Take, for instance, a case in which we have to find the mid-point between the 6th position from the left side
and the 11th position from the right side. The first thing we have to do is to convert the right position into a left
position to make the data comparable in nature. Doing so gives us 16. Now add up 16 and 6 (because now
both are from the same side), average them, apply EJOTY and you get the correct answer.

IV. REVERSING: Many questions concerning reversing of alphabets are a part of reasoning tests. Consider
this question

Solved Examples
Example No. 1 : What will the 11th letter of the following alphabet if the second half of the alphabet is
written in reverse order?

A B C D E F G H I J K L M N O P Q R S T U V W X Y Z

1 2 3 4 5 6 7 8 9 10 11 12 13 14 15 16 17 18 19 20 21 22 23 24 25 26

26 25 24 23 22 21 20 19 18 17 16 15 14 13 12 11 10 9 8 7 6 5 4 3 2 1

The most important thing to remember in a question like the above is to determine where the required position
falls, i.e. in the first or the second half. In this case, the answer falls in the first half. Therefore, if the second
half is written in reversed order, it will have no impact on the position of our letter. This can be likened to
reversing the tail of an elephant and asking if it would have an impact on its trunk. The answer is obviously
no. So in this case, we have to simply decide the 11th letter from the left side, which is K, by using the EJOTY
method.

Example No. 2 : What will be the 12th letter from the right side if the first half of the alphabet is written in
reverse order?

Now examine it carefully. This question is pretty same as the earlier one. The examiner has cleverly phrased
his question so as to trap you. If you want to count from the right side and the answer falls in the first half
from the right side (the normal second half is now your first half because you have started counting from this
side), reversing the first half of a normal alphabet will have no impact on the right answer. So now the
question can be rephrased like finding the 12th letter from the right side. The correct answer as you can find
quite comfortably is O.

A B C D E F G H I J K L M N O P Q R S T U V W X Y Z
Bulls Eye
4 www.hitbullseye.com

1 2 3 4 5 6 7 8 9 10 11 12 13 14 15 16 17 18 19 20 21 22 23 24 25 26
26 25 24 23 22 21 20 19 18 17 16 15 14 13 12 11 10 9 8 7 6 5 4 3 2 1

Example No. 3 : Which letter will be the 17th from the left side if the second half of the normal alphabet is
reversed?

Now this is posing a bit difficult problem. Your answer will fall in the second half( because the first half is
complete when you are at position 13 from left). So the letters A-M remain the same while from N Z are
written like Z, Y, X till N. In effect, we can count 13 letter from A M and then simply add four letters from
the behind of the alphabet. You can see for yourself that the fourth letter from the rear side is W, which is the
correct answer. But we have solved this question by looking at the above alphabet. Lets solve it the faster
way without looking up the alphabet.

Since only the 2nd half is being reversed, we can easily skip the first 13 letters, which will assume to have been
counted in a normal way. The question is how to find the 17th letter from the left with the reversed second
half. Simply count the fourth letter from the right side (which is obviously the 23rd from the left side), which if
added to the number 13, makes it the 17th from left. Refer to the following figure for clarity. Applying our
earlier procedure, the 4th from right is W; we get the same correct answer.

A B C D E F G H I J K L M Z Y X W V U T S R Q P O N
1 2 3 4 5 6 7 8 9 10 11 12 13 14 15 16 17 18 19 20 21 22 23 24 25 26

Now lets consider a case in which the entire alphabet is reversed.

Example No. 4 : What would be the 16th letter from the left side if the normal English alphabet were written
in reverse order?

A B C D E F G H I J K L M N O P Q R S T U V W X Y Z

1 2 3 4 5 6 7 8 9 10 11 12 13 14 15 16 17 18 19 20 21 22 23 24 25 26

26 25 24 23 22 21 20 19 18 17 16 15 14 13 12 11 10 9 8 7 6 5 4 3 2 1
Obviously the alphabet will look like Z, Y, X from the left if we reverse the whole of it. Now just imagine
that what was on the left earlier has become on the right now. So A will go to the right extreme, followed by B
and so on. In reality, what was 16th from the left earlier has now become the 16th from the right side. So if this
16th from right can be converted to the left side i.e. 11 and we already know from EJOTY that the 11th letter
from left is K.

Example No. 5 : What will be the 5th letter to the right of the 9th letter from the left side?

A B C D E F G H I J K L M N O P Q R S T U V W X Y Z

1 2 3 4 5 6 7 8 9 10 11 12 13 14 15 16 17 18 19 20 21 22 23 24 25 26

Go to the end of the question. It asks you to start counting from left and arrive at the 9th from the left which is
I. Now starting from I, we are to find the 5th letter to its right as per the statement, which is N.
In reality N is placed at the 14th from left which is a figure you can get by adding the numbers in the question
(9 + 5).
Rule No. 1: Whenever we are given two opposite directions, i.e., left-right or right-left, we will add up those
two numerals and will count from the direction given at the end of the question.
Bulls Eye 5
www.hitbullseye.com

Number Series

Given below are some of the major types of series. Try to understand each and every type and then practice it
with the questions given afterwards.

Solved Examples
Example No. 1: Find the next number in the following series 1, 4, 9, 16, 25, 36, 49,?
Solution: This is one of the simpler kind of series, as it can be seen that the difference between the numbers
goes like this. 1 4 9 16 25 36 49 ?

+3 +5 +7 +9 +11 +13 (+15)


Quite often, you can find the right answer in number series by taking the difference between consecutive pairs
of numbers, which form a logical series. As in the above example, the differences between succeeding pairs of
numbers are 3, 5, 7, 9, 11, 13. Logically speaking, the next difference must be 15. This 15, if added to the last
number 49, gives you 64, which is the correct answer. But thinking from a different viewpoint, all these
numbers given in the questions are squares of natural numbers 1, 2, 3, 4, 5, 6, 7. So the next number in the
series should be the square of 8, which is 64. So we can get the same correct answer in series, even if we
adopt two different logical approaches. Overall, finding the differences between two consecutive numbers or
two alternate numbers (if there are two parallel series running) remains the most helpful approach to solving
such questions.

Example No. 2: 2 6 21 881 445 ?


1 1

x 2+2 x 3+3 x 4+4 x 5+5

Solution: The above series involves two operations, multiplication and addition of a number by the same
number. The series runs like this 2 x 2 + 2 = 6, 6 x 3 + 3 = 21, 21 x 4 + 4 = 88, 88 x 5 + 5 = 445. The next
number, following this logic should be 445 x 6 + 6 = 2676. This is one case where the idea of differences will
not help us.
0 6 24 60 120 210 ?
Example No. 3

+6 +18 +36 +60 +90

+12 +18 +24 +30


Solution: Obviously, the first differences do not appear to contain any logic. But if we take the differences
among differences as ill-starred above, the logic becomes clear. By the same logic, the last addition to the last
number i.e. 30, should be 6, which makes it 36. Moving backwards, 36+90+210=336, which is the correct
answer. But this method proves to be time-consuming. Adopting a more sophisticated approach, these
numbers move in this fashion. 13 - 1 , 23 - 2, 33 - 3, 43 - 4, 53 - 5, 63 - 6. Following the same logic, the next
number in the series has to be 73 - 7 =336.
Bulls Eye
6 www.hitbullseye.com

Example No. 4: 3, 5, 7, 11, 13, 17, ?


Solution: This series is a series of prime numbers, it can be clearly seen that every number given is
prime in the same numerical order. Next prime number would be 23 and that would be the answer.

Example No. 5: 4, 6, 10, 16, 24, 34, ?, 60


Solution: Here in this case the addition in the numbers is increasingly 2. I.e. firstly 2 is added, then 4, then 6
and so on. It can be seen that before the questions mark 10 has been added, therefore the next addition would
be 12. And the number would be 34 + 12 = 46.

Example No. 6: 17, 23, 29, 35, ?, 47


Solution: In this case 6 has been added in all the numbers, to generate the next number. Firstly 17 + 6 = 23,
then 23 + 6 = 29 and so on. In order to get the next number after 35, 6 would be added in that. Therefore
question mark should be replaced by 35 + 6 = 41.

Example No. 7 : 1, 2, 5, 10, 17, 26, ?


Solution : In this example, the odd numbers have been added in the increasing order. Firstly 1, then
3, then 5 and 9 in the end. Then, in order to obtain the next number, 11 should be added. Therefore
the next number would be 26 + 11 = 37.

Example No. 8 : 4, 4, 8, 24, 96, 480, ?


Solution : In this questions every number is multiplied by the next natural number starting with 1. You can
understand this from the following diagram.
4 4 8 24 96 480 ?

x 1 x2 x3 x4 x5
The next number, as is clear from the above illustration, would be obtained by multiplying the last number
480 by 6. So the answer is 480 * 6 = 2880

Example No. 9 : 2, 10, 30, 68, ?, 222.


Solution : In this question, the method of adding same/different number in the perfect cubes of the natural
numbers, is applied. Here in this case, the cube of a natural number is added, in the number. I.e. these numbers
are 13 + 1, 23 + 2, 33 + 3, 43 + 4, and the answer would be 53 + 5 130.

Example No. 10 : 0, 2, 6, 12, ?, 30.


Solution: In this question from the square of a natural number, the same number has been subtracted. The
numbers are obtained as 12 - 1, 22 - 2, 32 - 3, 42 4. Therefore the next number would be 52 5 = 20.
Double Series : Much more common in tests are questions on double number series. In such questions, two
series are mixed up alternately. Have a look at the following example.

Example No. 11 : 5, 5, 15, 7, 45, 11, 135, 13, 405, ?.


Solution: In this question two series are going on. Whenever two series are going naturally every alternate
term would be belonging to the other series. You may analyze this from the following diagram.
Bulls Eye 7
www.hitbullseye.com

PRIME NUMBERS

5 5 15 7 45 11 135 13 405 ?

x3 x3 x3 x3

It could be clearly seen that one series out of these is of prime numbers and the series is number
multiplied by 3. Apparently, the missing number belongs to the category of prime number, so the
next prime number after 13 is 17 and that would become the answer.

Example No. 12: 6 6.5 7.5 9 11 13.5 ?


Solution: In this question the addition factor is increasing continuously by 0.5. The first addition done is 0.5,
the second is 1, the third is 1.5 and so on. In the last the addition to be done is 2.5 + 0.5 = 3. So the answer
would become 13.5 + 3 = 16.5.

Example No. 13: 4 9 17 35 69 139 ?


Solution: In this question the logic applied is 2 + 1 and 2 1 alternatively. In the end what is done is 2
+ 1 and in order to get the next term 2 1 is to be done. So the answer would be 139 2 1 = 277.

Example No. 14: 2 2 3 6 15 45 ?


Solution: In this question the multiplying factor is increasing by 0.5. Firstly multiplied by 1, then by 1.5, then
by 2, then by 2.5 and so on. In the question the second last given number has been multiplied by 3 to get the
last number. Now in order to obtain the next number we should multiply the last number by 3.5. Then answer
would be 45 3.5 = 157.5.

Example No. 15: 23, 35, 57, 711, 1113, ?.


Solution : In this question, the logic is very simple, two consecutive prime numbers have been combined and
represented as a single number i.e. 2 and 3 , then 3 and 5 35, then 5 and 7 57 and so on. Every number
starts with the same prime number with which the last prime number ended. The next prime numbers to be
taken are 13 & 17 and the number would become 1317.

Wrong Number : Sometimes, the question is asked to find out the wrong number in a series. This is a bit
difficult task. Because from the point where you get a wrong number after that all other numbers would also
be looking wrong only. You have to apply two logics while solving such questions, one is to recognize the
series and then try to recognize the series. If you are not able to do this start applying the logic from the back
of series and you would definitely be able to get some clue. But the question relating to the wrong number,
you can only solve, if you have expertise in handling the questions on series very well. See the following.

Example No. 16 : Find the wrong number in the following series.


15 46 63 71 75 77 78
Solution : In this question, if you from the beginning, firstly the difference is 31, then the difference is 17,
then the difference is 8. So there is no logic. If you start backwards, you can see that firstly, 1 is added, then 2
is added, then 4 is added, then 8, 16 and 32 should have been added. So you have got the right clue. So 15 +
32 the first number should be 47 and afterwards the series becomes + 16, + 8, + 4 and so on.
Bulls Eye
8 www.hitbullseye.com

Example No. 17: Find the wrong number in the following series.
10 21 43 85 175
Solution : In this question the logic involved is 2 + 1 only. But the error is lying when 43 2 + 1 = 87
should be written and 85 is written instead. So 85 is the wrong g number.

Replacing the Wrong Number : Sometimes, the question asked is not only related with finding the wrong
number, but with finding the number which should replace that wrong number. You can see the following
example.

Example No. 18 : In the following series, a wrong number is given, find out that wrong number, and see
from the option given, which of the options should replace that wrong number.

1 4 8 9 25 16 64

Solution: In this series two consecutive series are going simultaneously relating to perfect cubes and perfect
squares respectively. Firstly 1 cube is given, then 2 cube is given, then 3 cube should have been there which
should be 27, but 9 is written at that place. And then there is cube of 4 which is 64. And the series given is
alternatively square of 2, then square of 3 and then square of 4 and so on. You may note this that in order to
increase the confusion, the wrong number is also a perfect square.

Example No. 19 : In the following series, a wrong number is given, find out that wrong number, and see
from the option given, which of the options should replace that wrong number.

2 3 4 4 7 7

5 7 10 14 18 25 32

Solution : In this question the series is related to + 2, + 3, + 4 and then there is + 4 again, then + 7, it seems
odd. Instead of second + 4 there should be + 5. This means 14 + 5 19, should be there instead of 18. And
after that the series would become + 6 and + 7 and it would be a right logical series. So 19 is the answer.
Bulls Eye 9
www.hitbullseye.com

Continues Pattern Series

Solved Examples
This type of series consists of small letters when follow a certain pattern. Some spaces are left blank in
between the series. We have to fell the blanks from given option to follow a pattern. This can be clearly
understood by following examples.

Example 1: ab_ _ baa_ _ ab _


1. aaaaa 2. aabaa 3. aabab 4. baabb
Sol. The answer is (2) option and pattern becomes aba/aba /aba/aba

Example: 2: _aa _ ba _ bb _ ab _ aab


1. aaabb 2. babab 3. bbaab 4. bbbaa
Sol:- The answer is 3rd option and pattern becomes
baab / baab / baab / baab

Example 3:- ac _ cab _ baca _ aba _ acac


1. aacb 2. acbc 3. babb 4. bcbb
Sol.- The are is 1st option and pattern becomes acac / abab / acac / abab / acac. Thus acac and abab is
repeated.
Bulls Eye
10 www.hitbullseye.com

DS 01 - Wrong Number
DIRECTIONS for questions 1-13: In each of the following number series only one number is wrong. Find
out the wrong number.
1. 4 6 14 51 220 1125 6786
1. 51 2. 14 3.6 4.1125 5.220
2. 4 7 16 46 124 367 1096
1.367 2. 7 3.16 4.46 5.124
3. 4 2 3 7.5 26.25 118.725 649.6875
1. 118.725 2.26.25 3.7.5 4.3 5.2
4. 2 11 38 197 1172 8227 65806
1.11 2. 38 3.197 4.1172 5.8227

5. 16 19 21 30 46 71 107

1.19 2.21 3.30 4.46 5.71


6. 4 2 3.5 7.5 26.25 118.125
1. 118.125 2.26.25 3.3.5 4.2 5.7.5
7. 2 3 6 18 109 1944 209952
1.3 2.6 3.18 4.109 5. 1944
8. 1 3 6 11 20 39 70
1.3 2.39 3.11 4.20 5. 6
9. 2 13 27 113 561 3369 23581
1.13 2.27 3.113 4.561 5. 3369
10. 50 51 47 56 42 65 29

1.51 2.47 3.56 4.42 5. 65


11. 3 9 23 99 479 2881 20159

1.9 2.23 3.99 4.479 5. 2881


12. 2 3 6 15 45 156.25 630
1.3 2.45 3.15 4.6 5. 156.25
13. 36 20 12 8 6 5.5 4.5
1.5.5 2.6 3.12 4.20 5. 8
Bulls Eye
www.hitbullseye.com 11

CHP 2 CODING & MEMBERSHIP


(Coding, Membership)

CODING

Almost every test on reasoning contains questions on coding. In such a question, generally one word and its
code is given and the students are asked to find the code for the other given word, applying the same logic, as
what has been applied in the given examples.
But before we proceed to discuss the various types of questions related to coding, it is better to have an idea
regarding the general types in coding. Some of the major types of coding are

1. Constant addition in the position of alphabets.

2. Constant subtraction in the position of alphabets.

3. Denoting the position of alphabets in the whole alphabetic order.

4. Addition of the positions of all the alphabets to make code for the word.

5. Constant addition and subtraction respectively in the position of all the alphabets.

6. Square of the number of letters in the word.

7. Arranging the letters in the alphabetic order.

8. Arranging the letters given in the main word, in the reverse order.

9. Interchanging each pair of the letters, in the whole word.

10. Constant addition and then reversing the letters to make the final code.

These are some of the important types of the coding, now we will discuss all of these types of coding and
much more with the help of examples.

You must have notices from the above points, that it is very important for the student to know the alphabetic
order of all the alphabets.

The following method can be applied to learn the alphabetic order.

I. THE ALPHABET : The normal English alphabet contains 26 letters in all, as shown above

(Usually, questions on alphabet are accompanied by this normal alphabet). From A to M, the alphabet
completes its first half, while the other half starts from N and ends at Z.

A-M - 1-13 (First Alphabetical Half )

N-Z 14-26 (Second Alphabetical Half)


Bulls Eye
12 www.hitbullseye.com

II. EJOTY : For purpose of convenience, it is helpful to remember this simple formula called EJOTY, with
the help of which you can easily find the position of any letter without much effort. But for practical
purposes, you should learn by heart the positions of different letters in the alphabet, where these five letters
represent the following positions.

E J O T Y

5 10 15 20 25

Solved Examples
Example No. 1 : In a certain code CSAT is written as EUCV. How is CIVIL written in that code?

Sol : In this all the letters in the word are moved two places forward.

So CIVIL will be written as EKXKN.

Example No. 2 : If the word AMITABH is coded as HBATIMA, then how will you code the word
ANUPAM ?

Sol : In this the whole of the word is written in the reverse order only, then the word ANUPAM would also be
written in the reverse order and MAPUNA would be obtained as answer.

Example No. 3 : If HEMA EHAM, Then REKHA = ?

Sol : In this the pairs of letters are interchanged, similarly the pairs in the word REKHA would be
interchanged. And ERHKA, because A is the single letter remains, and it would be written as it is.

Example No. 4 : In a certain code SERVICES is written as TFSWHBDR. How is BULLSEYE written in
that code?

Sol : In this first half of the letters are moved one place forward and second half of the letters are moved one
place backward. So, BULLSEYE will be written as CVMMRDXD.

Example No. 5 : If the word RAMESH is written as 181135198, how will you write the word SUNITA ?

Sol : Now in this case, simply the position of the alphabet is written i.e. R = 18, A = 1, M = 13 and so on.
Similarly while making code for the word SUNITA, their alphabetic positions will be written i.e. S = 19, U =
21, N = 14, I = 9 and the code will become as 1921149201

Example No. 6 : If RAM 32, then SHAM = ?

Sol : In this question, the code for the word RAM has been made by adding the positions of all the alphabets
in the word i.e. R = 18, A = 1, M = 13 18 + 1+ 13 = 32. Similarly when the code for the word SHAM
would be made it will be S = 19, H = 8, A = 1 and M = 13 19 + 8 + 1 + 13 = 41.
Bulls Eye
www.hitbullseye.com 13

Example No. 7 : If ABHISHEK BAIHTGFJ, then HRITIK = ?

Sol : In this case, both +1 & - 1 are applied. A + 1 = B, B 1 = A, H + 1 = I, I 1 = H, S + 1 = T, H 1 = G,


E + 1 = F and K 1 = J, Similarly the code for the word HRITIK will be made i.e. H + 1 = I, R 1 = Q and
so on. The code will finally be IQJSJJ

Example No. 8 : In a certain code the word ADARSH is written as ZOZXYP, then how RADHA would be
written in the same code ?

Sol : Now in this case, the coding done can not be cracked, the simple method which is to be applied in this
case is that all the letters of the word RADHA are present in the word ADARSH, so corresponding code for
each letter would be taken. Firstly take code for R i.e. = X, then take code for A i.e. = Z. Similarly taking the
codes for other letters the word RADHA would be coded as XZOPZ.

Example No. 9 : If RAM = 9, SUDHA = 25, RAMESH = ?

Sol : Now logic applied, here is very simple. The logic is, there are 3 letters in the word RAM, so its code is
32 = 9, similarly there are 5 letters in the word SUDHA, so 52 = 25. As there are 6 letters in the word
RAMESH, so its code will be 62 = 36 and that is the answer.

Example No. 10 : If RAMESH = AEHMRS, then ANURADHA = ?

Sol : In this case, the coding is simply done in the increasing position of alphabets i.e. the dictionary order.
That is arrange the alphabets in the same order as they are in the actual alphabetic order. Firstly A comes, then
E, then H, then M, R and S respectively. Similarly when the code for the word ANURADHA will be made it
would be like AAADHNRU.

Example No. 11 : If HARISH is coded as ITJSBI, then how would REEMA be coded ?

Sol : In this case, the immediate next letter in the alphabet is taken and the code is written in the reverse order
i.e. the code for the letter H is I and it is in the end, the code for A is B and it is second from the end, similarly
the code for R is S and it is third from the end and so on. While coding the word REEMA, the code for R will
be S and it would be written in the end, similarly the codes for E, E, M and A would be F, F, N and B
respectively. And the final code would be BNFFS. This type is called as +1 and reverse order.

Example No. 12 : If SHIVANI = 574, then GANESH = ?

Sol : in this case, EJOTY of all the words has been added, then this is multiplied by the number of letters in
that word. SHIVANI S = 19, H = 8, I = 9, V = 22, A = 1, N = 14, I = 9 19 + 8 + 9 + 22 + 1 + 14 + 9 =
82 7 ( there are 7 letters in the word SHIVANI), Similarly while making code for GANESH 7 + 1+ 14
+ 5 + 19 + 8 = 54 6 ( there are six letters) = 324 would be the code.

Example No. 13 : If EVITCDNIV = HZNZJLWSG, then ABDICTION = ?

Sol : In this question, firstly + 2, then + 3, +4, +5, +6, +7 and so on. Similarly when the code for the word
ABDICTION is made firstly +2, then +3, +4, +5, + 6 and so on. The code will finally become DFIOJBNYY

Bulls Eye
14 www.hitbullseye.com

Example No. 14 : If SILVER = HROEVI, then MEENAKSHI = ?

Sol : In this case, if the letter is at nth position from the beginning then the letter at nth position from the
end is written. This can always be checked, whenever the sum of the number and its respective code is 27.
Then the method applied for the coding would be this only. As in SILVER, S is 19 and its code H is 8 and the
sum is 27. I is 9 and its code R is 18 and sum is 27. While coding MEENAKSHI, the same method coding
will be applied. M is 13, so what should be added in 13 to make it 27 (that is 14), write the 14th letter which is
N as the code for M. Similarly E is 5, find 22nd letter to make sum as 27 (V is 22nd letter) and that is the code
and so on. The code for the word MEENAKSHI will be NVVMZPSR.

Example No. 15 : If A = E, B = F, C = G and H = L, how will the word COME BACK be coded in this code?

Sol : Here in this case, it can be checked that 4th letter has been made the code for every letter, in the same
way, you can the 4th letter as the code for every letter i.e. COME BACK would become as C + 4 = G, O will
be S and the code would become GSQI FEGO.

Example No. 16 : If blue is called green, green is called yellow, yellow is called red, red is called brown and
brown is called pink, then, what is the colour of the blood?

1. red 2. brown 3. pink 4. Yellow

Sol : We know that colour of the blood is red. In the question red is coded as brown. So, our answer is brown,
i.e. 2nd option.

Example No. 17 : If bird is called Pen, Pen is called air, air is called well, well is called car, car is called sky,
sky, from where one can draw water?

1. well 2. sky 3. car 4. Pen

Sol : Water can be drawn from well. well is coded as car. So, answer is the car. i.e. 3rd option.

Example No. 18 : In a certain code language sim ma kom means bring me water. ma mo mok means
water is life. jka od sim means give me toy and mo min not means life and death. Which of the
following is representing is in that language ?

1. mo 2. ma 3. mok 4. min

Sol : Here in such questions by combining two groups, the code for one particular word can be decided.

In the first two coded sentences, the only code common is ma and the only word common is water. This
implies that ma is the code for water. Similarly in the second and fourth coded sentences the only code
common is mo and the only word common is life. This implies that the code for the word life is ma.
After this in the second coded sentence the only code remaining is mok and the only word remaining is is,
So the code for the word is is mok.

Example No. 19 : In a certain code language, 952 means order my tape 849 means buy great tape and
246 means buy my water. Which of the following digits represents great in that code ?

1. 2 2. 6 3. 8 4. 5
Bulls Eye
www.hitbullseye.com 15

Sol : Here by comparing sentence 1 and 2 get the code for the word tape which is 9. By comparing second
and third sentence get the code for the word buy, which is 4. So the remaining word in the second sentence
is great and the remaining code is 8. So the answer is 8 which is the third option.
Sometimes a very different type of questions are asked. In the directions itself four/five types of coding
methods are given, then from the question itself you have to recognize which type of coding is done.

Example No. 20 : Given below are five possible ways of coding a word. Study carefully and report which of
the methods has been used in each case.
1. GRAPES is coded as TFQBSH
2. APPLE is coded as DSSOH
3. ORANGE is coded as LOXKDB
4. COCONUT is coded as OCOCUNT
5. PINEAPPLE is coded as ELPPAENIP

A. COMPUTER is coded as OCPMTURE.


1. 1 2. 2 3. 3 4. 4
In this question, it can be seen, that simply the code is made by interchanging the pairs of letters as in the
case of 4th type. CO was made as OC, then again CO as OC, then NU as UN and the last letter was alone, so it
was just left. So the answer to this question is the 4th type of code, which is given as potion no. 4.

B. FLOPPY is coded as IORSSB.


1. 5 2. 3 3. 2 4. 1
In this question, the coding done is +3, i.e. every letter is replaced by a letter which is after three places in the
alphabet. Which is the case in the second type of coding. So second type of coding, which is given in the
option no. 3, is the answer.

C. GLOBALISATION is coded as OPJUBTJMBCPMH.

1. 4 2. 3 3. 2 4. 1

In this case, the coding done is +1 and writing it in the reverse order. i.e. as in first type of coding replacing
every letter with its next =letter in the alphabetic order and then writing the code in the reverse order (the code
for the first letter is written at the last position, the code of the second letter is written in the second last
position and the code of the last letter is written at the first position).

Bulls Eye
16 www.hitbullseye.com

MEMBERSHIP

This section deals with questions which aim at analysing a candidate's ability to relate a certain given group of
items and illustrate it diagrammatically.
Here are a few different types of Venn diagrams with their implications made clear. Suppose you are given a
group of three items. Then,

1. if the items evidently belong to three different groups, the Venn diagram representing it would be as
shown below.
Ex. Doctors, Engineers, Lawyers
These three items bear no relationship to each other. So, they are represented by 3 disjoint figures as
shown in the figure.

A B

2. if one item belongs to the class of the second and the second belongs to the class of third, then the
representation is in the form of three concentric circles, as shown in figure below.
Ex. Seconds, Minutes, Hours
Clearly, seconds are a part of minutes and minutes are a part of hours. So, the Venn diagram would be
as shown in the figure with circle A representing Seconds, circle B representing Minutes and circle C
representing Hours.
C
B

3. if two separate items belong to the class of the third, they are represented by two disjoint circles
inside a bigger circle as shown in the figure below.
Ex. Table, Chair, Furniture
Clearly, table and chair are separate items but both are items of furniture. So, they would be
represented as in the adjoining figure with circle A representing Table, circle B representing Chair
and circle C representing Furniture.

A B

C
Bulls Eye
www.hitbullseye.com 17

4. if two items belong to the class of the third such that some items of each of these two groups are
common in relationship, then they are represented by two intersecting circles enclosed within a bigger
circle.
Ex. Males, Fathers, Brothers
Clearly, some fathers may be brothers. So, fathers and brothers would be represented by two
intersecting circles. Also both fathers and brothers are males.
So, the diagrammatic representation would be as shown in the figure below, with circle A
representing Fathers, circle B representing Brothers and circle C representing Males.

A B

5. if two items are partly related to the third, and are themselves independent of each other they are re
presented by three intersecting circles in a line.
Ex. Dogs, Pets, Cats
Clearly, some dogs and some cats are pets. But, all the pets are not dogs or cats. Also, dogs and cats
are not related to each other. So, the given items would be represented as shown in figure below with
circle A representing Dogs, circle B representing Pets and circle C representing Cats.

A B C

6. if the three items are partly related to each other, they are represented as shown in the figure below.
Ex. Clerks, Government Employees, Educated Persons
Clearly, some clerks may be government employees and some may be educated. Similarly, some
government employees may be clerks and some may be educated. Also, some educated persons may
be clerks and some may be government employees. So, the given items may be represented as shown
in figure below with three different circles denoting the three classes.

A B

Bulls Eye
18 www.hitbullseye.com

7. if one item belongs to the class of second while third item is entirely different from the two, then they
may be represented by the following diagram.
Ex. Engineers, Human Beings, Rats
Clearly, all engineers are human beings. This would be represented by two concentric circles. But the
class of rats is entirely different from these two. Thus, these items would be represented as shown in
the figure below with circle A representing Engineers, circle B representing Human Beings and circle
C representing Rats.

B
C
A

8. if one item belongs to the class of second and the third item is partly related to these two, they are
represented as shown below.
Ex. Females, Mothers, Doctors
Clearly, all mothers are females. This would be represented by two concentric circles. But, some
females and some mothers can be doctors. So, the circle representing doctors would intersect the two
concentric circles. Thus, the diagram becomes as shown in the figure below with circle A
representing Mothers, circle B representing Females and circle C representing Doctors.

C
A

9. if one item belongs to the class of second and the third item is partly related to the second, they are
represented as shown below.
Ex. Grass-eating Animals, Cows, Flesh-eating Animals
Clearly, cows are grass-eating animals. So, they would be represented by two concentric circles. But
some grass-eating animals are flesh-eating also. Thus, the Venn diagram is as shown below with
circle A representing Cows, circle B representing Grass-eating Animals and circle C representing
Flesh-eating Animals.

A B C
Bulls Eye
www.hitbullseye.com 19

DS 02 - Coding-Decoding
1. In a certain code DIVISION is written as DVISIOIN. How is STATES written in that code?

1. S A T E T S 2. S T A E T S 3. S A T T E S 4. S A E T T S 5. S T T A E S

2. In certain code MONKEY is written as XDJMNL. How is TIGER written in that code?

1. S H F D Q 2. Q D F H S 3. S D F H S 4. Q D H J S 5. U J H F S

3. In certain code COURSE is written as FRXUVH. Then how would RACE be written in that
code?

1. H F D U 2. U C F H 3. U D F H 4. U D H F 5. U D F G

4. In a certain code ROAD is written as URDG. How is SWAN written in that code?

1. V X D Q 2. V Z D Q 3. U X D Q 4. V Z C Q 5. V Z C P

5. In a certain code LONDON is written as HPOEPO. Which of the following words is written as
DVOHSZ?

1. M E X I C O 2. I S L A N D 3. H O L A N D 4. H U N G R Y 5. H O N D U S

6. In certain code MILLION is written as IMLLOIN. How is HILTON written in that code?

1. I H T L O N 2. I H L O T N 3. I H T L N O 4. H I L T N O 5. I H T N L O

7. If in the English alphabet, every alternate letter from B onwards is written in small letters while others
are written in capitals, then how will the 3rd day from Monday be coded?

1. W e D N e S d A Y 2. W E d n E S d A Y 3. T H U R S d A Y
4. T h U r S d A Y 5. None of these

8. If the letters of the word RUTHLESS are arranged alphabetically, then which letter would be
farthest from the first letter of word?

1. H 2. E 3. U 4. T 5. None of these

9. In a certain code CERTAIN is coded as XVIGZRM. How is MUNDANE coded in that code?

1. N F M W Z M V 2. V M Z W M F N 3. N F M W Z M X 4. N F M X Z M V 5. None of these

10. In a certain code SEQUENCE is coded as HVJFVMXV. How is CHILDREN coded in that
code?

1. X S R M W I V M 2. X S R O W I V M 3. D S R O W I U N
4. M V I W O R S X 5. None of these

Bulls Eye
20 www.hitbullseye.com

11. In a certain code RATIONAL is written as RTANIOLA. How would TRIBAL be written in that
code?

1. T R I A L B 2. T I R A L B 3. T I R L B A 4. T I R A B L 5. T I R L A B

12. In a certain code PROSE is written as PPOQE. How would LIGHT be written in that code?

1. L I G F T 2. L L G F E 3. J I E H R 4. L G G H T 5. L G G F T

13. In a certain code FORGET is written as DPPHCU. How would DOCTOR be written in that
code?

1. B P A U M S 2. B P A U P S 3. E M D R P P 4. B P A R P P 5. None of these

DIRECTIONS for questions 14 18: The BEAT is written according to some code; using the same code the
word SOUP may be written as one of the five alternatives given for the question. Find the correct alternative.

14. BEAT is written as GIDV, SOUP may be written as

1. X S Y S 2. Y S X R 3. Y X S R 4. X S X R 5. X S Y R

15. BEAT is written as YVZG, SOUP may be written as

1. H L F K 2. K X M J 3. J P X U 4. P U K X 5. U P J X

16. BEAT is written as SZDA, SOUP may be written as

1. T O R N 2. R T O N 3. O T N R 4. T R N O 5. N O T R

17. BEAT is written as ZCYR, SOUP may be written as

1. Q M S N 2. Q N S O 3. M N S Q 4. R N T O 5. R M T N

18. BEAT is written as YHXW, SOUP may be written as

1. S R R S 2. P S S R 3. P R R S 4. S P R S 5. R P R S

19. In a certain code EASE is written as GUCG. How is CUT written in that code?

1. U V D 2. D V U 3. V W E 4. E W V 5. W E V

20. If MOBILITY is coded as 46293927, then EXAMINATION is coded as

1. 6 7 2 5 0 6 2 3 0 7 6 2. 5 6 1 4 9 5 1 2 9 6 5
3. 4 5 0 3 8 4 0 1 8 5 4 4. 5 7 1 5 9 4 1 3 9 5 5 5. None of these

21. If PROGRESS is coded as OQQSNPFHQSDFRTRT, then what is the last letter in the code for
HINDER?

1. Q 2. T 3. S 4. P 5. None of these
Bulls Eye
www.hitbullseye.com 21

22. If LOYAL is coded as JOWAJ, then PRONE is coded as

1. O R M N C 2. N R M N D 3. Q R P N F 4. N R M N C 5. None of these

23. If OPTION is coded as UKXFQM, then CHOICE is coded as

1. H L R K D F 2. I C K F E D 3. W M K L A F 4. I M R L E F 5. I C S F E D

24. If ORAL is coded as 3196, then WRITTEN is coded as

1. 5 9 9 2 2 5 5 2. 5 5 2 2 9 9 5 3. 5 8 9 3 3 5 5 4. 5 1 9 2 2 5 6 5. None of these

25. In a certain code CLOCK is written KCOLC. How is STEPS written in that code?

1. S P E S T 2. S P S E T 3. S E P S T 4. S E P T S 5. S P E T S

26. In a certain code QUESTION is written NXBVQLLQ. How is REPLY written in that code?

1. O B M I V 2. U H S O B 3. O H M O V 4. O F M M V 5. Y H M O V

27. In a certain code SCRIPT is written as TCQIQT. How is DIGEST written in that code?

1. E I H E T T 2. T I H E T T 3. E I F E T T 4. T I F E T T 5. E I F E R T

28. In a certain code PRAMOD is written SODJRA. How is KESHAV written in that code?

1. N B U E D S 2. N B V E C S 3. N B V F D S 4. N B V E D S 5. None of these

29. In SYSTEM is written as SYSMET and NEARER is written as AENRER, then FRACTION,
will be coded as

1. C A R F N O I T 2. C A R F T I O N 3. N O I T F R A C 4. F R A C N O I T 5. None of these

30. In a certain language, (1) hupa chip fu pa stands for the statement Ram is very intelligent, (2) chip
hupa kupa tik for Hari is very smart (3), Luk fu hupa for boy is intelligent and (4) fu tik dop
for smart and intelligent. Which of the following words stands for Hari?

1. Chip 2. hupa 3. tik 4. kupa 5. fu

31. Which of the statement in Q. 30 is superfluous?

1. A 2. B 3 None of these 4. D 5. C

32. If 1. Quo Cui Heer means Boy is good; 2. Lai Quo Mea means Meena is fair; 3. Ruo Lev
Mea means All are fair; 4. Si Hai Cui means Dog was good; then which of the following words
stands for Boy?

1. Q u o 2. C u i 3. H c c r 4. Lai 5. None of these


Bulls Eye
22 www.hitbullseye.com

33. Which of the statement in Q. 32 is superfluous?

1. A 2. B 3. C 4. D 5. None of these

34. If 1. Buy good oranges is coded as BDG; 2. Distribute good oranges is coded as BCD; and 3.
Oranges are red is coded as BEF, then what is the code for Red?

1. B 2. Either E or B 3. Either F or B 4. Either E or F 5. None of these

35. In a certain code language, 1. de bom means lovely flower, 2. poc til de means flower and fruit,
3. pere xas bom means lovely green pasture and 4. poc xas means green fruit. Which word in
that language means pasture?

1. pere 2. bom 3. poc 4. xas 5. de

36. In a certain code language, 1. pic vic nic means winter is cold, 2. to nic re means summer is
hot, 3. re pic boo means winter and summer and 4. vic tho pa means nights are cold. Which
word in that language represents summer?

1. to 2. nic 3. pic 4. vic 5. re

37. In a certain code language, 1. BCG means oranges are sweet; 2. KUB means buy good
oranges; and 3. BGL means red sweet oranges. Which letter in that code means good?

1. K 2. U 3. C 4. L 5. Cannot say

38. In a certain code language, 1. lew nas hsi ploy means she is bringing coffee; 2. wir sut lew ploy
means he is bringing milk; and 3. sut lim nas means milk and coffee. Which word in that
language means he?

1. sut 2. wir 3. ploy 4. lew 5. None of these

39. In a certain language, 1. La Pil Ta means Mango is sweet; 2. Na Sa Pil means Mango and
Banana and 3. Ba Ta Tik means Boy is Wise. In that language Sweet means

1. La 2. Pil 3. Sa 4. Ba 5. Ta

40. In a certain code language, TELEVISION is coded as ETLEIVSINO. How will ARITHMETIC
be coded in that code language?

1. RAITHMICTE 2. RATIMHTECI 3. RAITMHETCI


4. RATIHMTEIC 5. None of these
Bulls Eye
www.hitbullseye.com 23

DS 03 - Membership
DIRECTIONS for questions 1 to 5: In the following questions, three classes are given. Out of the four
figures that follow, you are to indicate which figure will best represent the relationship amongst the three
classes.

1. 2. 3. 4.

1. Women, Mothers, Doctors

2. Authors, Teachers, Men

3. Sparrows, Birds, Mice

4. Tea, Coffee, Beverages

5. Boys, Students, Athletes

DIRECTIONS for questions 6 to 9: Each of these questions below contain three groups of things. You are to
choose from the following five numbered diagrams, the diagram that depicts the correct relationship among
the three groups of things in each question.

1 2 3 4 5

6. Tables, Chairs, Furniture

7. Tie, Shirt, Pantaloon

8. Dogs, Pets, Cats

9. Brinjal, Meat, Vegetables


Bulls Eye
24 www.hitbullseye.com

DIRECTIONS for questions 10 to 22: Each one of the following questions contains three items. Using the
relationship between these items, match each question with the most suitable diagram. Your answer is the
letter denoting that diagram.

1. 2. 3. 4. 5.

10. Deer, Rabbit, Mammal

11. Human beings, Teachers, Graduates

12. Whales, Fishes, Crocodiles

13. Plums, Tomatoes, Fruits

14. Mountains, Forests, Earth

15. Tiger, Fox, Carnivores

16. Grams, Beans, Legumes

17. Flowers, Clothes, White

18. Uncles, Parents, Friends

19. Rohtak, Haryana, Punjab

20. Engineer, Doctor, People

21. Thieves, Lawyers, Criminals

22. Sea, Island, Mountain

DIRECTIONS for questions 23 to 28: In each of the following questions, choose the Venn diagram which
best illustrates the relationship among three given words.

1. 2. 3. 4. 5.
Bulls Eye
www.hitbullseye.com 25

23. Diseases, Leprosy, Scurvy

24. Hockey, Cricket, Games

25. Yak, Zebra, Bear

26. Sun, Moon, Stars

27. Animals, Men, Plants

28. Mercury, Mars, Planets

DIRECTIONS For questions 29 to 31: In each of the following questions, find out which of the alternatives
indicates the correct relationship between the three given words.

29. Elected house, M.P., M.L.A.

1. 2. 3. 4.

30. Triangle, Four-sided figure, Square

1. 2. 3. 4.

31. Doctor, Nurse, Human being

1. 2. 3. 4.

Bulls Eye
26 www.hitbullseye.com

DIRECTIONS for questions 32 to 40:

1. Indicates that one class is completely contained in the other but not the third.
2. Indicates that two classes are completely contained in the third.
3. Indicates that neither class is completely contained in the other but the two have common members,
forming one entity.
4. Indicates that two classes are interrelated and third one is not. Choose the Venn diagram which best
illustrates the three given classes in each question.

32. Protons, Electrons, Atoms

33. Sun, Planets, Earth

34. Dog, Animal, Pet

35. Science, Physics, Chemistry

36. Atmosphere, Hydrogen, Oxygen

37. Wheat, Grains, Maize

38. Machine, Lathe, Mathematics

39. Biology, Botany, Zoology

40. Citizens, Educated, Men

DIRECTIONS for questions 41 to 45: Each of the questions below contains three elements. These elements
may or may not have some inter linkage. A group of the elements may fit into one of the diagrams. You have
to indicate the group of elements which correctly fit in the diagrams.

1. 2. 3. 4. 5.

41. Pencil, Stationery, Jeep

42. Factory, Machinery, Product

43. Vegetable, Brinjal, Cauliflower

44. Honesty, Intelligence, Aptitude

45. Truck, Ship, Goods


Bulls Eye
www.hitbullseye.com 27

DIRECTIONS for questions 46 to 50: Choose the Venn diagram which best illustrates the three given
classes in each question:

1. 2. 3. 4.

46. Girl, Athlete, Singer

47. Window, Room, Wall

48. State, Country, City

49. Copper, Paper, Wire

50. Teacher, Graduate, Player


Bulls Eye
28 www.hitbullseye.com

CHP 3 DIRECTIONS & RELATIONS


DIRECTION SENSE
In this topic a number of instructions are given regarding the movement of a person, we have to find the final
location of the person or distance travelled by him or distance between initial or final position. Before doing
this topic the following points should be kept in mind.

1. The adjacent figure shows the four main directions i.e. North, South,

West and East Also four cardinal directions (North-East (NE), North-West (NW) South-East (SE) and South-
West (SW) are shown to help the candidates to know the direction.
N
NW NE

W E

SW SE
S

2. One should start solving the question taking the initial position as origin.

3. Keep in mind the change in direction when a person takes right or left

Direction before Direction in which


taking the time person will be money
after taking time.

Right Left

North East West

South West East

East South North

West North South

4. One should keep in mind that in finding the distance between initial final position shortest distance is
to be taken and use of by Pythagoras theorem is there.

We can understand the above said points with the help of following examples.
Bulls Eye
www.hitbullseye.com 29

Solved Examples
Example No. 1 : Anuoshka starts from home and walks 80 metres towards the south. She turns to her left and
walks 50 metres. She again turns to her right and walks 40 metres. She takes a turn towards her left and walks
60 metres. She takes a final turn towards her left and walks 120 metres. Where and at what distance is
Anuoshka from the starting point?

Solution : As usual, our starting point will be the point where the N-S and E-W lines intersect, as has been
illustrated alongside. We follow her movement as has been asked of us in the question and finally arrive at a
point in the east direction. Now the problem before us is to find her distance from the starting point. We will
have to calculate it indirectly because there is no direct way to do so. If you look carefully, the two distances
i.e. 50 and 60 metres, if joined together give us the distance traveled by her on the eastern side, taken from the
starting point. If you transpose this line connecting 50 and 60 metres, you will see that this is exactly the
distance traveled by her on the eastern side. So the answer in this case is 110 metres east.
N

W E

80 m 120 m
50 m
S 40 m
60 m
Much more common is a type involving the application of the Pythagorean theorem. Lets examine such a
case.

Example No. 2 : Bhanu starts from her office in the morning at 9 a.m. to meet her friend Kanu. She walks 40
metres towards the west, then takes a right turn and walks 30 metres. What is her distance from the starting
point and in which direction is she now?

Solution : Now this, is posing a bit difficult question. As you can see for yourself, Bhanu is somewhere in the
middle of the western and the northern direction. This is where the Pythagorean Theorem comes into play.
Look carefully, these two distances, 40 and 30 metres, are forming a right-angled triangle, and by applying
the Pythagoras theorem we can get the third side.

Bulls Eye
30 www.hitbullseye.com

30
W E
40

Just for review, the Pythagorean Theorem states that the square of the hypotenuse of a right-angled triangle is
equal to the sum of the squares of the perpendicular and the base.

Let us assume that the third (unknown) side is designated X. Applying the theorem,

X2 = (30)2 + (40)2

X2 = 900 + 1600 = 2500

Therefore, X = 2500 = 50 metres. As you can see, she is in the middle of the northern and the western sides.
Therefore, combing these two sides, we say she is at present in the north-west 50 metres away from her
starting point. Keep in mind that it is not necessary for the person to be exactly in the middle of the two sides
for you to say that he is in the north-east direction. Even if he is just 10 degrees in the east and mostly in the
north, even then it should be stated that he is in the north-east direction.

RELATIONS
Now in relation questions, the following technique is applied. This technique is known as moving backwards
or backtracking.

Backtracking means starting with the last information and proceed backwards. For example, if it is given
that he is the son of the mother of my grandfathers daughter, then we proceed with the last information :
grandfathers daughter. Grandfathers daughter means aunt (sister of father). Now mother of my
grandfathers daughter becomes mother of my aunt. Mother of my aunt is my grandmother. Hence, he is
the son of the mother of my grandfathers daughter becomes son of my grandmother, which is father or
uncle.

As in the above given question a) Anupam says to Kapil, my fathers wife, means mother, her son (either
yourself or your brother, his two sisters (means your sisters), so the girl playing with hockey is Anupams
sister.

b) Sudha says in the end, only son of my grandfather which means father only, he is the father of a mans
brother means father of Sudhas brother. So Sudha is sister of the man in the photograph.
Bulls Eye
www.hitbullseye.com 31

Solved Examples
Example No. 1 : Pointing to a photograph Rajesh said, she is the mother of my mothers mothers
daughter. How is Rajesh related to the woman in the photograph?

Solution : My mothers mothers daughter means either my mother or my mothers sister. Mother of my
mother or my mothers sister means my maternal grandmother. Therefore the man is the maternal grandson of
the woman in the photograph.

Example No. 2 : Pointing to a boy in the photograph, Minakshi said,His sister is the only daughter of my
father. How is the boy related to Minakshis father?

Solution : My fathers only daughter means Minakshi herself. She is the sister of the boy in the photograph.
This means the boy is the brother of Minakshi. Therefore he is the son of Minakshi s father.

Example No. 3 : Introducing Madhuri, Anil said, She is the wife of my mothers only son. How is
Madhuri related to Anil?

Solution : My mothers only son means Anil himself. Now Madhuri is the wife of that person, means she is
the wife of Anil.

Example No. 4 : Pointing to Kajol, Ajay said, Her mothers only daughter is my mother. How is Ajay
related to Kajol?

Solution : Here in this statement Ajays mother is the only daughter of his maternal grandmother.

His maternal grandmother is the mother of Kajol (because Ajay points to Kajol and says her mother. Means
Kajol is Ajays mother.

Note : Unless otherwise mentioned all relations are to be taken from the paternal (fathers) point of view i.e. if
grandfather has been mentioned it would be taken as fathers father and not as mothers father unless maternal
grandfather is mentioned.

This technique of backtracking is applied in such questions where a single statement is given. But sometimes
there are questions in which the information is provided in the form of points. Then it is advisable to arrange
the information in the form of a family tree.

The following rules can be kept in mind, to solve the questions relating to relation in an easy way.

a) Vertical or diagonal lines should be used to represent parent-child relationships i.e. if the generations are
different then these should be represented vertically.

b) If the relationship is of the same generation then it should be represented horizontally.

c) A single arrow should be used to mark a relationship of brother or sister i.e.

d) A double horizontal line (like ) should be used to represent marriages.

e) Put a + sign before someone who is a male and a sign before someone who is a female.

Bulls Eye
32 www.hitbullseye.com

f) Whenever something is not known put a ? mark or some such symbol before it.

Here is an illustration to elaborate :

Y(+)
X() Z()

T(?) ?(+)

Q()
P(+)

J() ?(+)

The above example tells us that :

(i) Y and Z are a couple ; Y is the husband while Z is the wife.

(ii) Y has a sister X.

(iii) The couple, Y and Z, has three children : T, P and another son, whose name is not known. P is also son
while the sex of T is not known.

(iv) T and the other unknown son are unmarried while P is married to Q.

(v) The couple, P & Q, has a daughter J and a son whose name is not known.

Given the process, how to solve such questions quickly

1. Read the information given quickly, it will give you the idea of the problem.

2. Use the actual information given to make a family tree. It is better to start from the relation of a single
degree i.e. no third person is involved in that. The relation should be a direct one, instead of taking uncle
and nephew, it is better to take father and son or brother and sister.

3. When you started making tree then try and attach the further information given to the already made tree
only.

4. Then combined all the information and answer the questions accordingly.

Example No. 5: P is brother of Q, R is sister of Q. How P is related to R?

Solution :
P(+) Q(?) (R)()
Bulls Eye
www.hitbullseye.com 33

It can be clearly seen, that P is brother of R

Example No. 6: X and Y are brothers. C and D are sisters. As son is Ds brother. How is Y related to C ?
Solution : It has been clearly indicated that Y is the brother of X, Xs son is Ds brother. This implies that D
is the daughter of X. Since C and D are sisters, C is also the daughter of X. SO Y is the uncle of C.

Example No. 7 : M and N are a married couple. O and P are brothers. O is the brother of M. How is P related
to N ?

Solution : M and N are husband and wife. Since O and P are brothers, and O is the brother of M, P is also the
brother of M. Thus, P is the brother-in-law of N.

Example No. 8 : A is brother of B, C is the father of A, D is a brother of E, E is a daughter of B. Who is the


uncle of D ?

Solution : C

A B (?)

(-) E D

It can be seen from the diagram that the uncle of D is A.

Bulls Eye
34 www.hitbullseye.com

DS 04 - DIRECTIONS & RELATIONS


DIRECTIONS for questions 1 10: These are questions pertaining to movement of a person or a vehicle in
a given direction. Using sense of direction, you are required to determine the location of the person or vehicle,
after the person or vehicle has covered a certain distance, taking turns towards right to leave.

1. Sham travels 7 km North, then turns right and walks 3 km. He again turns to his right hand side and
moves 7 km forward. How many km is Sham away from the place of his starting the journey?

1. 7 km 2. 3 km 3. 6 km 4. 14 km 5. None of these

2. Reeta drives to North of her place of stay at A and finds after travelling 25 km that she has driven in
the wrong direction. She then turns to the right and travels 2 km and then again turns right and drives
straight another 25 km. How much distance she has now to cover to go back to the point from where
she started?

1. 25 km 2. 22 km 3. 4 km 4. 50 km 5. 2 km

3. Rana travels 10 km North turns left and travels 4 km and then again turns right and covers another 5
km. He then turns to right hand side and travels another 4 km. How far is he from the point of starting
his journey?

1. 15 km 2. 4 km 3. 5 km 4. 10 km 5. None of these

4. Seeta and Ram both start from a point towards North. Seeta turns to left after walking 10 km. Ram
turns to right after walking the same distance. Seeta waits for some time and then walks another 5 km,
whereas Ram walks only 3 km. They both then return to their respective South and walk 15 km
forward. How far is Seeta from Ram?

1. 15 km 2. 10 km 3. 8 km 4. 12 km 5.3 None of these

5. A taxi driver commenced his journey from a point and drove 10 km towards North and turned to his
left and drove another 5 km. After waiting to meet one of his friends, he turned to his right and
continued to drive another 10 km. He has covered a distance of 25 km so far but in which direction he
now may be?

1. North 2. East 3. West 4. South 5. None of these

6. There is a ring road connecting points A, B, C and E. The road is in a complete circular form but
having several approach roads leading to the centre. Exactly in the centre of the ring road there is a
tree which is 20 km from point A on the circular road. You have taken a round of the circular road
starting from point A and finish at the same point after touching points B, C and E. You then drive 20
km interior towards the tree from point A and from there reach somewhere in between B and C on the
ring road. How much distance you have to travel from the tree to reach the point between B and C on
the ring road?

1. 30 km 2. 15 km 3. 80 km 4. 40 km 5. 20 km
Bulls Eye
www.hitbullseye.com 35

7. A tourist drives 10 km towards East and turns to right hand side and takes a drive of another 3 km. He
then drives towards West (turning to his right) another 3 km. He then turns to his left and walks
another 2 km. Afterwards, he turns to his right and travels 7 km. How far is he from his starting point
and in which direction?

1. 10 km East 2. 9 km North 3. 8 km West 4. 5 km South 5. None of these

8. Rahul walks 30 metres towards south. Then turns to his right and starts walking straight till he
completes another 30 metres. Then again turning to his left he walks for 20 metres. He then turns to
his left and walks for 30 metres. How far is he from his initial position?

1. 50 metres 2. 30 metres 3. 10 metres 4. 60 metres 5. 70 meters

9. Vandana drove her car for 30 kms due North. Then she turned left and drove for 40 kms. She then
turned left again and drove yet another 30 kms. Again she turned left and drove her car 50 kms. How
far do you think she actually drove her car from the initial position?

1. 10 kms 2. 50 kms 3. 30 kms 4. None of these 5. Cant Say

10. Shalloo ran 20m to the east, then he turned left and walked for 15 m, then turned right and went 25 m
and then turned right again and went 15 m. How far was Shalloo from the starting point?

1. 45 m 2. 35 m 3. 25 m 4. 15 m 5. 50 m

DIRECTIONS for questions 11 14: If you start running from a point towards north and after covering 4
kms you turn to your left and run 5 km, and then again turn to your left and run 9 km and then turn to left
again and run another 6 km and before finishing you take another left turn and run 1 km, then answer these
questions.

11. How many km are you from the place you started (approximately)?

1. 1 km 2. 2 km 3. 3 km 4. 4 km 5. None of these

12. In which direction will you be running while finishing?

1. East 2. West 3. North 4. South 5. None of these

13. After taking the second turn, in which direction will you be running?

1. East 2. West 3. North 4. South 5. None of these

14. From the finishing point if you have to reach the point from where you started, in which direction will
you have to run?

1. East 2. North West 3. North-South 4. South-East 5. None of these


Bulls Eye
36 www.hitbullseye.com

DIRECTIONS for questions 15 17: Study the given information and answer the following questions.
1. There are 6 check-posts, A, B, C, D, E and F.
2. Check-post D is 15 kms to the North of F which is 25 kms to the North-East of B.
3. Check-post A is 5 kms west of E and 15 kms to the South-West of C.
4. B, A and E are in straight line in the same order.
5. The check posts B and E are 30 kms apart from each other.

15. Which check post is the farthest to the South-West of D?

1. A 2. E 3. C 4. D 5. B

16. Which post is the nearest and to the North-East of E?

1. A 2. B 3. C 4. D 5. None of these

17. If a jeep moves from E to D via A, B and F, how much distance it will have to cover?

1. 70 km 2. 120 km 3. 100 km 4. 80 km 5. None of these

DIRECTIONS for questions 18 22: Ram walks 2 km towards North and turn to his right and walks 4 km
more. He then turns to his right and walks 4 km and turns again to his right and walks another 4 km. Here he
meets Renu coming from the opposite directions. They both stop here.

18. After taking the first turn, in which direction was Ram going?

1. East 2. West 3. North 4. South 5. None of these

19. If the starting point is marked A and finishing point is marked B, what will be the distance
between these points?

1. 10 km 2. 8 km 3. 6 km 4. 20 km 5. 2 km

20. From which direction was Ram coming in the end?

1. South 2. North 3. South-east 4. East 5. None of these

21. After taking the second turn, in which direction was Ram walking?

1. West 2. North 3. South-east 4. South 5. None of these

22. If Ram is to again reach the point from where he started in which direction will he have to go from
where hes standing now?

1. East 2. North 3. South-east 4. North-east 5. None of these


Bulls Eye
www.hitbullseye.com 37

DIRECTIONS for questions 23 26: Refer to the following diagram and answer the questions that follow:

Government employees
Urban people
Graduates
Teachers

23. Which of the following statements is true?

1. All Urban peoples are graduates.


2. All graduates are urbans.
3. All Urban government employees are graduates.
4. All teachers are urban people.
5. None of these

24. Choose the correct statement.

1. There are some urban teachers who are government employees as well as graduates.
2. No teacher is a government employee.
3. All graduates are government employees.
4. All government employees are urban people.
5. None of these

25. Mark the correct statement.

1. All non-urban teachers are government employees.


2. All urban government employees are teachers.
3. There are some non-urban graduates who are neither teachers nor government employees.
4. All urban government employees are graduates.
5. None of these

26. Which of the following statements is not true?

1. Some government employees are rural.


2. All teachers are urban.
3. Teachers who are government employees are urban.
4. All government employees are urban people.
5. None of these

DIRECTIONS for questions 27 30: These questions pertain to the following information. Amit is the son
of Rahul. Sarika, Rahuls sister has a son Sonu and a daughter Rita. Raja is the maternal uncle of Sonu.

27. How is Amit related to Sonu?

1. Nephew 2. Cousin (Brother) 3. Uncle 4. Brother 5. None of these

28. How is Rita related to Raja?

1. Sister 2. Daughter 3. Niece 4. Aunt 5. None of these


Bulls Eye
38 www.hitbullseye.com

29. How many nephews does Raja have?

1. 1 2. 2 3. 3 4. 4 5. None of these

30. What is the relationship of Raja with Rita?

1. Uncle 2. Brother 3. Maternal Uncle 4. Nephew 5. None of these

DIRECTIONS for questions 31 33: These questions pertain to the following information.

There are six persons S1, S2, S3, S4, S5, and S6,
S3 is the sister of S6.
S2 is the brother of S5s husband.
S4 is the father of S1 and grandfather of S6.
There are 2 fathers, one mother and 3 brothers in the family.
31. Who is S5s husband?
1. S2 2. S3 3. S1 4. S4 5. None of these

32. Who is the mother?

1. S1 2. S2 3. S3 4. S5 5. None of these

33. How many male members are there?

1. 1 2. 2 3. 3 4. 4 5. None of these
Bulls Eye
www.hitbullseye.com 39

CHP 4 SYMBOPERATION

SYMBOPERATION
As is clear from the name, operations are done, by following the directions given by symbols. These questions
do check the analytical skills of the candidate. One has to be very careful while attempting these problems. A
greater than or equal to B is different from A greater than B.

Solved Examples
Example: 1 In these questions symbols @, #, $, % and are used with different meanings as follows:

A @ B' means 'A is smaller than B.'

A # B' means 'A is greater than B.'

A $ B' means 'A is either smaller than or equal to B.

A % B' means 'A is either greater than or equal to B.'

A B' means 'A is neither greater than nor smaller than B.'

In each question, three statements showing relationships have been given, which are followed by two
conclusions I & II. Assuming that the given statements are true, find out which conclusion(s) is/are definitely
true.

Mark answer (1), if only conclusion I is true; mark answer (2), if only conclusion II is true; mark answer (3),
if either conclusion I or II is true; mark answer (4), if neither I nor II is true and mark answer (5), if both
conclusions I and II are true.

Statements : L $ K, K @ M, J%M

Conclusions : I. L @ M

II. K @ J

L K (i) ; K < M (ii) ; J M (iii)

Combining all, we get L K < M J.

Hence L < M and I follows.

Again, K < J and II follows.

Statements : E $ W, W @ Q, Q % H

Conclusions : I. E $ Q
II. E H

Bulls Eye
40 www.hitbullseye.com

E W (i); W < Q(ii) ; J H(iii)

from (i) and (ii), E W < Q or E < Q(iv).

Hence I does not follow. Again, from (iv) and (iii), E and H cant be compared. Hence II does not follow.

Statements : J T, T # W, W%R

Conclusions : I. J # R

II. T % R

; J = T.(i) ; T > W.(ii) ; W R .(iii)

Combining all, we get J = T >W H .(iv)

Hence J > R and so I follows.

Also, T > R and so II does not follow.

Statements : T # R, R % H, H@F

Conclusions : I. H @ T

II. F T

T > R (i) ; R H (ii) ; H < F (iii)

From (i) and (ii), T > R H or H < T. Hence I follows.

But from I and (ii) F and T cant be compared.

Hence II does not follow.

Statements : F D, D # V, V@P

Conclusions : I. F P

II. V # F

F = D (i); D > V . (ii) ; H < F (iii)

From (i) and (ii), F = D > V or V < F.

Hence II follows.

But from I and (iii) F and T cant be compared. Hence I does not follow.
Bulls Eye
www.hitbullseye.com 41

Example: Symbols @, %, *, $ and # are used with different meanings as explained below:

' A$B' means 'A is not greater than B.

'A#B' means' A is not smaller than B'.

'A@B' means 'A is neither greater than nor equal to B'.

'A % B' means 'A is neither smaller than nor equal B'.

'AB' means 'A is neither smaller than nor greater than B'.

In each question, three statements showing relationships have been given, which are followed by two conclusions
I and II. Assuming that the given statements are true, find out which conclusion(s) is/are definitely true.

Give answer 1): if only conclusion I is true.

Give answer 2): if only conclusion II is true.

Give answer 3): if either conclusion I or II is true.

Give answer 4): if neither conclusion I nor II is true.

Give answer 5): if both conclusions I and II are true.

Statements : H%L, LG, G#M


Conclusions : I. H#M II. EL H%G

H > L... (i); L - G ... (ii); G 2 M... (iii)

Combining all, we get H > L = G M

Hence, H > M. This does not lead us to conclusion 1. And H > G This is conclusion II.

Statements : A$J, J%R, P@R

Conclusions : I. J%P II. PA

A J ... (i). J > R ... (ii); P < R ... (iii)

Combining (ii) and (iii), we get J > P. Hence

conclusion I is true.

But A and P can't be compared from (i) and I. Hence II is not true.
Statements : K%N, N#U, U$M
Conclusions : I. N M II. N%M
K >N ...(i); N U...(ii); U S M... (iii)
N and M can't be compared from (ii) and (iii). Nor are I and II exhaustive because they are
silent on the possibility N < M. Hence neither is definitely true.

Bulls Eye
42 www.hitbullseye.com

Statements : E%I, 1K, K@J


Conclusions : I. K%E II. J%I
E > I... (i); I - K ... (ii); K < J ... (iii)
Combining (i) and (ii), we get
E > I = K or E > K.
Hence conclusion I (K > E) is not true.
Combining (ii) and (iii), we get
I = K< J or I < J.
Hence conclusion II (J > I) is true.
Statements : BS, S@D, U#D
Conclusions : I. S@U II. D%B

B = S ... (i); S < D ... (ii); U ^ D ... (iii)


Combining (ii) and (iii), we get
S < D U or S < U. Hence conclusion I is true.
Combining (i) and (ii), we get B = S < D or B < D
Hence conclusion II (D > B) is true.
Bulls Eye
www.hitbullseye.com 43

DS 05 - Symboperation
DIRECTIONS for questions 1 to 25: Now in each of the following sets of questions, assuming the three
statements to be true, state which of the two conclusions I and II given below is definitely true. Mark
1. If only conclusion I is true
2. If only conclusion II is true
3. If either I or II is true
4. If neither I nor II is true
5. If both I and II are true

DIRECTIONS for questions 1 to 5: Symbols @, @, =, , b are used with the following meaning.
A @ B means A is greater than B.
A @ B means A is either greater than or equal to B.
A = B means A is equal to B.
A b B means A is smaller than B.
A B means A is either smaller than or equal to B.

1. Statements: SbM M@L L@Z


Conclusions: I. S = Z II. S L

2. Statements: C=D DbM NC


Conclusions: I. N b M II. C M

3. Statements: J@K Lb J L@B


Conclusions: I. K = L II. J @ B

4. Statements: C@D D=E EbF


Conclusions: I. C @ F II. D @ F

5. Statements: S@T MK T=K


Conclusions: I. T @ M II. T = M

DIRECTIONS for questions 6 to 10: In the following questions, the symbols , , =, @ and @ have been
used as follows.
A B is A is greater than B.
A B means A is either greater than or equal to B.
A = B means A is equal to B.
A @ B means A is smaller than B.
A @ B means A is either smaller than or equal to B.

6. Statements: Z@Y YX Z=X


Conclusions: I. Z = Y II. X = Y

7. Statements: A@B C@D B@C


Conclusions: I. A D II. A @ D

8. Statements: DA BC A=B
Conclusions: I. B D II. C @ A

Bulls Eye
44 www.hitbullseye.com

9. Statements: PQ Q@R RP
Conclusions: I. R @ Q II. Q R

10. Statements: M@P M=N PN


Conclusions: I. N @ P II. P M

DIRECTIONS for questions 11 to 15: In the following questions the symbols @, @, $, and * are used with
the following meaning:
@ means greater than.
@ means either greater than or equal to.
$ means smaller than.
means either smaller than or equal to.
* means equal to.

11. Statements: P@Q Q$C CD


Conclusions: I. P * D II. P $ D

12. Statements: XY Y$Z X*T


Conclusions: I. T @ Z II. Y $ T

13. Statements: A@K KL A*M


Conclusions: I. K $ M II. L @ M

14. Statements: BN C@T N*T


Conclusions: I. N $ C II. C @ B

15. Statements: C$M J@C C@N


Conclusions: I. J * N II. J @ N

DIRECTIONS for questions 16 to 20: In the following questions, the symbols are used as illustrated as
follows:

A @ B means A is smaller than B.


A @ B means A is either smaller than or equal to B.
A = B means A is equal to B.
A B is A is greater than B.
A B means A is either greater than or equal to B

16. Statements: B@K K@M M@Z


Conclusions : I. B @ Z II. B = Z

17. Statements: RB B@M


Conclusions: I. R M II. R @ M

18. Statements: M@R QP P=R


Conclusions: I. M Q II. M = Q

19. Statements: M@N N@R


Conclusions: I. R M II. R M
Bulls Eye
www.hitbullseye.com 45

20. Statements: P@Q QM


Conclusions: I. P @ M II. P M

DIRECTIONS for questions 21 to 25: In the following question Symbol @, @, =, and used with the
following meaning:

P @ Q means P is greater than Q.


P @ Q means P is either greater than or equal to Q.
P = Q means P is equal to Q.
P Q means P is smaller than Q.
P Q means P is either smaller than or equal to Q.

Now in each of the following questions assuming the given statements to be true, find which of the two
conclusions I and II given below them is/are definitely true.

21. Statements: M@N N T T=P


Conclusions: I. P @ N II. P = N

22. Statements: H@W W S M@S


Conclusions: I. M = H II. M @ H

23. Statements: G S F@S T G


Conclusions: I. F @ T II. T = S

24. Statements: P Q Q R T Q
Conclusions: I. R @ T II. R @ P

25. Statements: M=N N@B B P


Conclusions: I. P = N II. B M
46 Bulls Eye
www.hitbullseye.com

CHP 5 INPUT - OUTPUT


INPUT / OUTPUT
In this method, we are supposed to imagine that input is fed to a machine. Machine processes the input and
arranges the input in each step and produces the final output in the final step. We have to determine the
pattern and then solve the questions accordingly.

In this machine, arrangement can be of following types:

1) Numeric numbers arranging in increasing/decreasing order in each step.


2) Words arranging in alphabetic order from A to Z.
3) In series there is both numeric and alphabetic words, so in one step, numbers are arranged in
increasing/decreasing order and in the other step, words are arranged in alphabetic order from A to Z.
4) Numbers can be arranged as odd and even numbers. Odd/Even numbers are arranged in
increasing/decreasing order in the arrangement.
5) Words are arranged in alphabetic order in extreme left and numbers are arranged in decreasing order at
extreme right in each step and vice versa.

Some Important Tricks:

First of all, candidates should monitor the given Input Line of words or numbers carefully and the last
step of rearrangement so that they can identify the changes appeared in various steps of
rearrangement.
Candidates should observe at least two consecutive steps to identify the changes appeared in every
step.
At last compare the input, the last step and any one of the middle steps. It will enable the candidate to
identify the rule of arrangement quickly.

Solved Examples

Example 1: A word and number arrangement machine when given an input line of words and numbers
rearranges them following a particular rule in each step. The following is an illustration of input and
rearrangement.

Input: 23 64 87 24 35 68

Step 1: 87 23 64 24 35 68
Step 2: 87 68 23 64 24 35
Step 3: 87 68 64 23 24 35
Step 4: 87 68 64 35 23 24
Step 5: 87 68 64 35 24 23
Step 5 is the last step of the given input.
Here in this arrangement the numbers are arranged in the decreasing order. In each step one number is
brought to the start of the sequence.
Now based on this arrangement find the number of steps of the following input.
Bulls Eye 47
www.hitbullseye.com

Input: 28 54 75 84 65 57

Solution: We will arrange the numbers in decreasing order arranging one number in one step. So we have the
steps
Step 1: 84 28 54 75 65 57
Step 2: 84 75 28 54 65 57
Step 3: 84 75 65 28 54 57
Step 4: 84 75 65 57 28 54
Step 5: 84 75 65 57 54 28
So the step 5 is the last step of the input.

Example 2: A word and number arrangement machine when given an input line of words and numbers
rearranges them following a particular rule in each step. The following is an illustration of input and
rearrangement.

Input: good boy completes all his work

Step 1: all good boy completes his work


Step 2: all boy good completes his work
Step 3: all boy completes good his work

Step 3 is the last step of the above arrangement. You have to answer the question by following the same rules
as illustrated above.

Input: how to find the lost bag

How many steps will be needed to find the required output?

Solution: If we go through the given arrangement above, we can see that in the last step, that the words are
arranged in alphabetical order as arranged in the dictionary. Also in each step, one word is brought to the start
of the line alphabetically.
So we have

Input: how to find the lost bag

Step 1: bag how to find the lost


Step 2: bag find how to the lost
Step 3: bag find how lost to the
Step 4: bag find how lost the to

Hence 4 steps are required to find the output.


48 Bulls Eye
www.hitbullseye.com

Input: how to find the lost bag

Which of the following will be the 4th word from the right in the third step?

1. lost 2. to 3. the 4. bag 5. how

Solution: We have
Input: how to find the lost bag

Step 1: bag how to find the lost


Step 2: bag find how to the lost
Step 3: bag find how lost to the

Hence the 4th word from the right in the third step is how.

Example 3: A word and number arrangement machine when given an input line of words and numbers
rearranges them following a particular rule in each step. The following is an illustration of input and
rearrangement.

Input: quick 39 fox 23 jumps 34 dog 42

Step 1: 23 quick 39 fox jumps 34 dog 42


Step 2: 23 quick 34 39 fox jumps dog 42
Step 3: 23 quick 34 jumps 39 fox dog 42
Step 4: 23 quick 34 jumps 39 fox 42 dog

In this arrangement, the numbers are arranged in increasing order and the words are arranged in reverse
alphabetical order as per the dictionary with numbers and words being arranged in alternate steps.

Based on the arrangement given above, find the number of steps required to complete the arrangement.

Input: like 24 56 green through 45 maid 64

Solution: Here we will arrange the numbers in increasing order and the words in reverse alphabetical order.
We have

Input: like 24 56 green through 45 maid 64


Step 1: 24 like 56 green through 45 maid 64
Step 2: 24 through like 56 green 45 maid 64
Step 3: 24 through 45 like 56 green maid 64
Step 4: 24 through 45 maid like 56 green 64
Step 5: 24 through 45 maid 56 like green 64
Step 6: 24 through 45 maid 56 like 64 green

Hence step 6 is the last step of the arrangement.


So once you have completed the arrangement, you will be able to solve the questions.
Bulls Eye 49
www.hitbullseye.com

DS 06 - Input Output
DIRECTIONS for questions 1 5: Study the following information carefully to answer the questions given
below.
In a toy exhibition, a machine processes a given input by following rule. Participants are shown one by one
till it reaches its last step. Following is an illustration of the working of this machine.
Input : sui me ato fe zen u no
Step I : fe sui me no ato zen u
Step II : no fe sui u me ato zen
Step III : u no fe zen sui me ato
Step IV : zen u no ato fe sui me
Step V : ato zen u me no fe sui and so on.

1. What will be the step IV for the following input? Input may sen to cry if not hell

1. cry may sen to if not hell 2. if not hell to cry may sen
3. sen to if may not hell cry 4. not hell cry if may sen 5. None of these

2. Which of the following correctly describes the machine logic in generating various steps based on
the given input?

1. Each step is generated on random basis


2. Words/letters are finally arranged in dictionary order
3. The seventh word occupies the fourth position in the immediate next step every time
4. Data inadequate
5. None of these

3. If the Step III of an input is to men chi from yel as know, which of the following would be its input?

1. Data inadequate 2. from the men know chi yet as


3. men chi yet to as know from 4. chi as yet men know from to 5. None of these

4. If the step V of an output is so cd rom lay is nor it, which of the following would be step II?

1. Is nor it rom lay so cd 2. Nor it lay is so cd rom


3. Lay socd it rom is nor 4. Data inadequate 5. None of these

5. Which of the following steps would read as not you only say wise yet are for the input say not you
are only wise yet?

1. III 2.V 3. VI 4. VII 5. None of these


50 Bulls Eye
www.hitbullseye.com

DIRECTIONS for questions 6 10: Study the following information carefully to answer the questions given
below.
A word and number arrangement machine when given an input line of words and numbers rearranges them
following a particular rule in each step. The following is an illustration of input and rearrangement
Input: say dry 42 96 get 39 kite 67
Step I: 96 say dry 42 get 39 kite 67
Step II: 96 dry say 42 get 39 kite 67
Step III: 96 dry 39 say 42 get kite 67
Step IV: 96 dry 39 say 67 42 get kite
Step V: 96 dry 39 say 67 get 42 kite
Arrangement in Step V is the final arrangement and Step V is the last step.
You have to answer the question by following the same rules as illustrated above.

6. Which step will be the last step of an input for which third step is 91 go 28 mock pet 43 lead 37?

1. Eighth 2. Seventh 3. Sixth 4. Fifth 5. None of these

7. If the second step of an input is 52 at deep follow 4116 road 32, what will be the fifth step?

1. 52 at 16 road 32 deep follow 41 2. 52 at 16 road 41 deep follow 32


3. 52 at 16 road 32 follow 41 deep 4. There will be no such step. 5. None of these

8. If the third step of an input is 65 daily 12 tie 42 23 foreign urgent what will be definitely the input?

1. foreign 65 tie urgent 12 42 23 daily 2. foreign 65 urgent tie 42 daily 2312


3. foreign 65 12 urgent tie 42 daily 23 4. Cannot be determined 5. None of these

9. If the second step of an input is 76 from 48 super itself 56 18 went, how many more steps will be
required to complete the arrangement?

1. Five 2. Six 3. Four 4. Three 5. None of these

10. What will be the third step if the input is thirty days from now 32 56 87 24?

1. 87 thirty days from now 3 56 24 2. 87 days thirty from now 32 56 24


3. 87 days 24 thirty from now 32 56 4. 87 thirty 24 days 32 from now 56 5. None of these

DIRECTIONS for questions 11 16: Study the following information carefully and answer the questions.

An exhibition is open for public since 9 am till 3 pm and again since 4 pm till 10 pm. In a day there are 12
batches of 1 hour each. The entry ticket bears a pass code made up of seven words, which changes every hour
following a particular rule. The pass codes for 4 pm to 10 pm are the same as those for respective hours
during 9 am till 3 pm, i.e. the pass code for 4 pm to 5 pm is same as of 9 am to 10 am and so on. Following is
an illustration of the code and steps of rearrangement for subsequent clock hours.

First batch 9 am to 10 am Dig more and you will find water


(4 pm to 5 pm)
Second batch 10 am to 11 am And dig find you water will more
(5 pm to 6 pm)
Third batch (11 am to 12 noon) Find and will you more water dig.
(6 pm to 7 pm)
Bulls Eye 51
www.hitbullseye.com

11. If the pass-code for 7 pm to 8 pm batch is pen with write pencil nice time day, what will be the pass-
code for 11 am to 12 noon?

1. day nice with pencil write pen time 2. day with nice pencil write pen time
3. nice day with pencil write pen time 4. nice day pencil with write time pen
5. None of these

12. If the pass-code for the batch 4 pm to 5 pm is go to office in time every day, what will be the pass
code for 2 pm to 3 pm batch?

1. day to go in every office time 2. to day go in every office time


3. to go day in every office time 4. to go in day every office time 5. None of these

13. If the pass-code for third batch is any cold and at is hill station, which batch will have the pass-code
station is cold at and any hill?

1. Fifth 2. Fourth 3. Second 4. Sixth 5. First

14. If the pass-code for the second batch is do not play the near water dirty, what will be the pass code
for 2 pm to 3 pm batch

1. dirty near play the not do water 2. near dirty not the play do water
3. dirty near not the play do water 4. near dirty not the play water do 5. None of these

15. If the pass-code for third batch is at study sleep and night child good, which batch will have the
pass-code child sleep night and study good at?

1. Second 2. Fourth 3. Sixth 4. Fifth 5. None of these

16. If the pass-code for 5 pm to 6 pm is out in above over field the and, what will be the pass-code for 1
pm to 2 pm?

1. field and the over out in above 2. the field and over out in above
3. field the and over out above in 4. the field and over out above in 5. None of these
52 Bulls Eye
www.hitbullseye.com

CHP 6 SYLLOGISM

Basics

Syllogism has been defined as A form of reasoning in which a conclusion is drawn from two given or
assumed propositions. It is deductive reasoning rather than inductive reasoning.
For Example the two Propositions could be
1. All short men are fat.
2. All fat men wear caps.
We can conclude on the basis of the above two statements that All short men wear caps
However the validity of the conclusion is important. Not all conclusions need be valid. Only those two
propositions which lead to a valid conclusion are relevant to our cause.
For Example
1. All trains are long.
2. Some buses are long.
The Conclusion that Some buses are trains is invalid.

You might be facing some difficulties regarding which type of statement can lead to what type of
conclusion. If you understand the rules given below and their proper use, you are likely to commit no
mistakes.

Classes of Statements

A class is defined to be the collection of all objects that have some characteristics in common. Classes
can be related to each other in many ways. If every member of one class is also a member of a second
class, then the first class is said to be included or contained in the second. If some but perhaps not all
members of one class are also member of another, then the first class may be said to be contained
partially in the second class. Of course, there are pairs of classes having no members in common, such as
the class of all triangles and the class of all circles.
Now let us take 4 statements
1. All politicians are liars
2. No politician is a liar
3. Some politicians are liars
4. Some politicians are not liars.
The first statement is about two classes and clearly defines that the first class is included in the second.
The second statement says that the first class is wholly excluded from the second.
The third statement says that at least one member of class 1 is also a member of class 2. The fourth
similarly implies that at least one politician is not a liar.
Note: it is important to understand the above four statements completely before proceeding further.
Bulls Eye 53
www.hitbullseye.com

1. Universal Positive Statement: First of all universal means something which is applicable to all and
positive means, this is of yes format. Universal positive statement indicates something positive
applicable to all the items of that category. This is represented by the letter A. This kind of statements
begins with all, every & each.
Some Examples are
All boys are cute.
All girls are hardworking.
All Indians are brave.
Every fan is a cooler.
Each of the schools is a college.
All these are A type of statements, because they all are conveying universal positive meanings.

2. Universal Negative Statement: Again in this case, the only difference from the last category is that, in
this case, the statement conveys a negative meaning. It implies that it refers to that kind of statements,
which are universal and giving negative impression. These types of statement begin with No, None of
the, Not a single etc. and are represented by the letter E. Some examples of these types of statements
are:
No S is P.
No computer is intelligent.
No Hema is Rekha.
No girl is lazy.
None of the boys is crazy.
Not a single sheet is paper.

3. Particular Positive Statement: In this case, the statement given gives a positive impression but it covers
only some items and not all. This kind of statement begins with some, any, a few and are represented by
the letter I.
Some examples of I type of statements are
Some actors are comedians.
Some vegetables are potatoes.
Few colleges are schools.
Some telephones are shoes.

4. Particular Negative Statement: Here, the statement again covers only some items, but it gives a
negative impression. These kinds of statements are represented by the letter O. Some examples of this
are
Some boys are not cute.
Some buses are not circular.
Some x are not y.
Some Kapils are not Sunils.
Few breads are not butters.
The definitions of the A, E, I, O statement are very important and the student must be able to
immediately recognize the statement. With this need in mind, given below is a list of all the four types of
statements.
54 Bulls Eye
www.hitbullseye.com

Sr No. Type of statement Represented by the letter


1. Universal Positive A
2. Universal Negative E
3. Particular Positive I
4. Particular Negative O

As far as the questions of syllogism are concerned, there are so many varieties of questions; it could be
two statements and two conclusions, three/four statement and three/four conclusions and it could also be
a question having six statements. In order to solve questions, first the candidate should try to check the
conclusions, by reversing the individual statements given and then by combining the two/three
statements given. First of all, we will start with, what can be derived from a single statement. Before we
start with that, let us understand some universal principles.

Rules for Conclusions

1. With two particular statements no universal conclusion is possible.


2. With two positive statements no negative conclusion is possible.
3. With two negative statements no positive conclusion is possible.
4. With two particular statements no conclusion is possible, except when an I type of statement is
given and then by reversing it an I type of conclusion is given.

Venn Diagram Approach

No A statement of Type When reversed gives a conclusion of type


Type Example Venn Type Conclusion
Diagram
1. E No cat is E No dog is cat
dog C
&

D O Some dogs are not cats.


O is a part of E

2. I Some S I Some helmets are scooters


scooters
are H Here, the conclusion some helmets are not scooters is
helmets seemingly right, but simply remember the universal
principal no. 2. So this can not be true.
3. A All I Some rats are mouses.
mouses R
are rats. M In this case, if you are also taking the conclusion some
rats are not mouses to be true, again remember the
second universal principal. (no negative with positive is
possible)
Bulls Eye 55
www.hitbullseye.com

4. O Some No With O type of statement, you can not get any conclusion.
TV are T concl Even you can not make the conclusion as some
not usion computers are not TVs. The representation of the main
computer C is statement could be given in the following way.
possib
le T
C

Because the statement only signifies that some part of TV


must be outside the circle of computers, and all the
diagram, which are representing that are valid diagram.

Some Examples

You may have some confusion regarding I and O types of statements. Each of the statements should be
taken literally i.e. nothing should be assumed.

Let us take an example of a statement of type I.

Some Indians are brave. This information just indicates that some part of the circle of Indians must be
inside the circle of brave, may be all. Please note, the stated words are may be, therefore this statement
can be represented in three different diagrammatic ways, which are

I B B I

I B

All these three diagrams are valid ones. You should always make the first type of diagram, but always
keep in mind that the second and third types of diagram are also possibilities. The second diagram is
indicating that, if you conclude that Some Indians are not brave, then it is wrong. Similarly the third
diagram indicates that if you conclude Some brave are not Indians, even which is wrong. Because the
statement given is positive, you can not conclude anything negative.

Let us try to find all the possible conclusions from the statements given. Let us take some examples.

Example No. 1: Statements: I. Some printers are scanners.


II. Some scanners are not clocks.

Solution : Now, in this case, the possible conclusion is Some scanners are printers (I to I), as the
universal principal no. 4 says, that with two particular statements only I to I is possible, therefore only 1
conclusion is possible.

Example No. 2 : Statements : I. Some houses are not homes


. II. Some theatres are not houses.
Solution : It can be seen, here that both the statements given are particular and none of the statements is
of I type, so no conclusion is possible.
56 Bulls Eye
www.hitbullseye.com

Example No. 3 : Statements : I. Some bags are helmets.


II. All helmets are telephones.
Solution : Here the possible conclusions are
1. Some helmets are bags. ( I to I)
2. Some telephones are helmets. ( A to I)

These two conclusions have been derived from the individual statements alone, but if you combine these
two statements, then the diagram becomes
B

H
T

By combining these two statements, the third conclusion possible is Some telephones are bags.
Similarly the fourth conclusion is some bags are telephones. So four conclusions are possible.

Example No. 4 : Statements : I. All X are Y.


II. All Y are Z.

Solution : Here by these statements, individually, the following conclusions can be drawn.
1. Some Y are X. 2. Some Z are Y.

If you combine these two statements then the diagram would be


Z
Y
X

From this diagram, the conclusions, which can be drawn are


3. All X are Z. 4. Some Z are X.

Again, if you are getting the conclusion, some Z are not X, then simply remember the universal principal
no. 2, with two positive statements, no negative conclusion is possible.

Example No. 5 : Statements : I. All A are B


. II. No B is C.
Solution : Now individually, the conclusions are
1. Some B are A ( A to I) 2. No C is B ( E to E)
Again if you combine these two statements, then the diagram would be like
B
A
C

The conclusions, which can be drawn from this diagram are


3. No A is C 4. No C is A.
Bulls Eye 57
www.hitbullseye.com

DS 07 - Syllogisms
DIRECTION for questions 1 to 5: In each question below are given three statements followed by four
conclusions numbered I, II, III and IV. You have to take the two given statements to be true even if they seem
at variance from commonly known facts. Read all the conclusions and then decide which of the given
conclusions logically follows from the two given statements, disregarding commonly known facts.

1. Statements: Some dice are slates. All states are apples. No apple is car.
Conclusions:
I. Some cars are slates II. Some dice are cars.
III. Some apples are dice IV. No car is dice

1. No any conclusion follows 2. Only II follows 3. Only III follows


4. Either II or IV and III follow 5. None of these

2. Statements: Some films are clouds. All rats are clouds. Some clouds are chairs.
Conclusions:
I. No film is chair II. Some rats are films
III. Some clouds are rats IV. Some chairs are rats

1. Only I and III follow 2. Either II or IV follows


3. None follows 4. Only IV follows 5. None of these

3. Statements: No candle is bell. Some shoes are bells. All tables are shoes.
Conclusions:
I. Some tables are bells II. No table is bell
III. Some shoes are candle IV. No shoes is candle.

1. Only I and IV follow 2. Only I and II follow


3. Only III and IV follow 4. Only I and III follow 5. None of these

4. Statements: All gates are flowers. Some gates are fruits. Some flowers are clips.
Conclusions:
I. Some flowers are fruits II. Some clips are fruits
III. Some clips are gates. IV. No flower is fruit

1. Only I follows 2. Only I and IV follow


3. Only II and IV follow 4. Only I and III follow 5. None of these

5. Statements: Some ice are rings. No ring is paint. Some rings are gold
Conclusions:
I. No gold is paint II. No ice is gold
III. Some rings are paints. IV. All golds are rings.

1. Only I and III follow 2. Only I and II follow


3. Only III and IV follow 4. Only II and III follow 5. None of these
58 Bulls Eye
www.hitbullseye.com

DIRECTION for questions: 6 to 10: In each of these questions, you are given three statements followed by
three conclusions numbered I, II and III. You have to take the given statements to be true even if they seem to
be at variance from commonly known facts. Read all the conclusions and then decide which of the given
conclusions logically follows from the given statements, disregarding commonly known facts.

6. Statements:
All locks are keys. All keys are bats. Some clocks are bats
Conclusions:
I. Some bats are locks. II. Some clocks are keys.
III. All keys are locks.
1. Only I and II follow 2. Only II and III follow 3. Only I follows
4. Only II follows 5. I, II and III follow

7. Statements:
Some cups are pots. All pots are tubes. All cups are bottles.
Conclusions:
I. Some bottles are tubes. II. Some pots are bottles.
III. Some tubes are cups.

1. Only I and II follow 2. Only II and III follow


3. Only I and III follow 4. I, II and III follow 5. None follows

8. Statements:
All papers are books. All bags are books. Some purses are bags.
Conclusions:
I. Some papers are bags. II. Some books are papers.
III. Some books are purses.

1. Only I follows 2. Only II and III follow


3. Only I and III follow 4. Only I and II follow
5. I, II and III follow

9. Statements:
Some cars are jeeps. All boxes are jeeps. All pens are cars.
Conclusions:
I. Some cars are boxes II. No pen is jeep
III. Some boxes are cars.

1. None follows 2. Only I and II follow 3. Only II follows


4. Only II and III follow 5. Only I follows

10. Statements: Some rats are cats. Some cats are dogs. No dog is cow.
Conclusions
I. No cow is cat. II. No dog is rat.
III. Some cats are rats.

1. None follows 2. Only I and II follow


3. Only II and III follow 4. Only III follows
5. I, II and III follow
Bulls Eye 59
www.hitbullseye.com

DIRECTIONS for questions 11 to 15: In each question below there are three statements followed by four
conclusions numbered I, II, III and IV. You have to take the given statements to be true even if they seem to be
at variance from commonly known facts. Read all the conclusions and then decide which of the given con-
clusions logically follows from the given statements, disregarding commonly known facts.

11. Statements:
No Cloud is Bird. Some Goats are Birds. All Cars are Goats.
Conclusions:
I. No Car is Cloud. II. Some Cars are Birds.
III. No Bird is Car. IV. Some Clouds are Goats.

1. Only III follows 2. Only either II or III follows


3. Only I follows 4. Only I and either II or III follow
5. None of these

12. Statements:
All Grapes are Bananas. All Potatoes are Bananas. Some Bananas are Mangoes.
Conclusions:
I. No Grape is Mango. II. Some Potatoes are not Mangoes.
III. Some Grapes are Potatoes. IV. All Mangoes are Grapes

1. Only I follows 2. Either I or IV follows


3. Only II and III follow 4. Only I, II and III follow 5. None of these

13. Statements:
Some Cats are Rats. Some Rats are Ants. Some Ants are Flies.
Conclusions:
I. Some Flies are Cats. II. Some Flies are not Ants.
III. No Rat is Fly. IV. No Cat is Fly.

1. Only I and IV follow 2. Only II follows


3. Only I and III follow 4. Either I or IV follows 5. None of these

14. Statements:
All chalks are Dusters. Some Chalks are Boards. Some Dusters are Pens.
Conclusions:
I. Some Pens are Chalks. II. Some Dusters are Boards.
III. Some Pens are Boards. IV. All Chalks are Pens.

1. Either I or IV follows 2. Only II and III follow


3. Either I or IV and II follow 4. Only II follows 5. None of these

15. Statements:
Some Bags are Books. All Books are Boxes. No Box is Board.

Conclusions:
I. Some Bags are not Boards. II. Some Bags are not Boxes.
III. All Bags are Boxes. IV. No Bag is Board.
1. Only I follows 2. I and either II or III follow 3. Only IV follows
4. Only II follows 5. None of these
Bulls Eye
60 www.hitbullseye.com

CHP 7 LOGICAL CONNECTIVES


In Logic, we deal with statements that are essentially sentences in the English language. However, in Logic
we are not interested in or worried about the factual correctness of the sentence. We are interested only in the
Logical "truthfulness" of the statements.

Statements like "I will go for shopping", "It is a rainy day", etc are called simple statements. When two or
more such simple statements are connected together to form a single statement, such a statement is called a
compound statement.

The simple statements are combined using logical connectives to form compound statements. We should
know some of the important logical operators/connectives to be able to effectively tackle questions that
involve compound statements and logical operations on compound statements.

Negation ("NOT")

Any statement can be negated by using the words "not" or "no". In layman's language, negation is like the
opposite of a statement.

For example, the negation of the statement "I will drink water" is "I will not drink water." The negation of the
statement "He will pass the exam" is "He will not pass the exam."

Having defined simple statements, we shall now study about a few common operators (also called
connectives) that can be used to combine (or operate upon) two or more simple statements and arrive at more
complicated or compound statements.

Logical Connective OR

Two or more statements can be connected using the connective OR. The following is an example using OR.
It is raining or I will go to movie.

The same statement can also be written as: Either it is raining or I will go to movie.

Both the statements above mean the same. The additional word "either" does not change the meaning of the
statement.
When two (or more) statements are connected using OR, that implies exactly one of them is true and the other
is false.
Suppose we have a statement "Either p or q", that implies if p is not happening, then q must happen and if q is
not happening, then p must happen.

Logical Connective AND

Two or more statements can be connected using the connective AND. The following is an example using
AND. It is raining and I will go to movie.

The two statements connected by and have to be true for the compound statement to be true. In general, if
we have a statement "p and q", then we can conclude that p should be true as well as q, that is, both the
statements should be true. Even if one of the two statements is false, the compound statement is false. Here
p and q are simple statements.
Bulls Eye
www.hitbullseye.com 61

IMPLICATION AND NEGATION TABLE

Number Logical Connectives Implications Negations


1. If p, then q pq p & -q
2. Whenever p, then q -q -p -q & p
3. Either p or q -p q -p & -q
-q p -q & -p
p -q p&q
q-p q&p
4. Unless p, then q -p q -p & -q
-q p -q & -p
5. Only if p, then q qp q & -p
-p -q -p & q

Implication and Negation


Let us take all the statements one by one and understand their meanings as well the implications we can draw
from the same. Here we will also do the negation of a particular statement. But let all of us be clear about the
same, that negation does not mean opposite of the statement. It means where the condition given in the
question gets broken i.e. it is not fulfilled e.g. the negation of the statement India will win the match is India
will not win the match. The negation of this statement would not be India will lose the match. While
negating we have to keep in mind that the condition given in the question is not fulfilled. Let us understand all
the above types in detail.

1. If p, then q: The implication of this is that if p happens, q happens. Also the reverse with negation of
each of them is true. It means if negation q happens, implies negation p happens. In simple language
if q does not happen, p does not happen. These two are the valid conclusions or implications.

Example: If it is raining, there would be slippery.


Sol: The implications of this statement are:
It is raining, there is slippery.
There is no slippery, it is not raining.

Negation: The negations or wrong implications of the statement, If p, then q are: 1) q implies p
2) negation p implies negation q.

Example: If it is raining, there would be slippery.


Sol: The negation or wrong implications of this statement are:
There is slippery, it is raining.
It is not raining, there is no slippery.

2. Whenever p, then q: This second type of connectives gives exactly the same implications and same
negations. Hence for all the practical purposes, the same implications and the same negations as
explained in the first type should be considered. Let us just take one more example to get some more
clarity.
Example: Whenever I study, I get good marks.
Sol: The implications of this statement are:
I studied implies I would get good marks.
I do not get good marks imply I do not study.
Bulls Eye
62 www.hitbullseye.com

The negations of this statement are:


I get good marks do not imply I studied.
I do not study does not imply I do not good marks.

3. Either p or q: The implication of this is that if negation p happens, implies q happens. One more
implication is there, that is: if negation q happens, implies p happens. Besides that there are two more
implications, which are: 1) p happens implies q does not happen 2) q happens implies p does not
happen.

Example: Either Mohan would sleep or Sohan would play the game.

Sol: Its implications are:


Mohan did not sleep implies Sohan plays the game.
Sohan did not play the game implies Mohan slept.
Mohan sleeps implies Sohan would not play the game.
Sohan plays the game implies Mohan would not sleep.

Negation: The negation of this is if p does not happen, implies q does not happen. One more implication is
there, that is: if q does not happen, implies p does not happen. Besides that there are two more negations, i.e.
when p & q both are happening simultaneously.

Negations of this statement are:


Mohan did not sleep implies Sohan did not play the game.
Sohan did not play the game implies Mohan did not sleep.
Mohan sleeps implies Sohan plays the game.
Sohan plays the game implies Mohan sleeps.

4. Unless p, then q: The implications of this is that if negation p happens, implies q happens. One more
implication is there, that is: if negation q happens, implies p happens.

Example: Unless he works hard, he would fail.

Sol: Its implications are:


He did not work hard implies he failed.
He did not fail implies he worked hard.

Negation: The negation of this is if p does not happen, implies q does not happen. One more implication is
there, that is: if q does not happen, implies p does not happen. Any other negation would be incorrect.

Negations of this statement are:

He did not work hard implies he did not fail.


He did not fail implies he did not work hard.

5. Only if p, then q:: The implication of this is exactly reverse of the 1st case. The implication of this is
that if q happens, implies p happens. Also the reverse with negation of each of them is true. It means
if negation p happens, implies negation q happens.
Bulls Eye
www.hitbullseye.com 63

Example: Only if she performs well, she would get promotion.

Sol: Its implications are:


She got promotion implies she performed well.
She did not perform well implies she did not get promotion.

Negation: The negation of this is exactly reverse of the 1st case. The negation of this is that if q
happens, implies p does not happen. Also one more negation is true which is, if negation of p
happens, implies q happens. Any other negation would be incorrect.

The negations of this statement are:


She got promotion and she did not perform well.
She did not perform well and she got promotion.

Negation of compound statements formed with OR, AND

A compound statement formed with OR or AND can be negated in the following manner:

"Negation (p OR q)" is the same as "Negation p AND Negation q."


"Negation (p AND q)" is the same as "Negation p OR Negation q."

As described above, when a compound statement consisting of two simple statements (connected within OR
or AND) is negated, the result will consist of each of the individual statements negated in the same manner
i.e. (i) OR will become AND (ii) AND will become OR

Example: Mohan would go to cinema or would read a book.


Sol: The negation of this statement is: Mohan would not go to cinema and would not read a book.

Example: Seema would go to party and Sneha would clean the house.

Sol: The negation of this statement is: Seema would not go to party or Sneha would not clean the house.

Example:
If it is raining, then I will go to a mall or I will visit my uncle's place.

(A). It is not raining, means that I will not go to a mall or I will not visit my uncle's place.
(B). It is not raining, means that I will not go to a mall and I will not visit my uncle's place.
(C). I will not go for a mall or I will not visit my uncle's place, means that it is not raining.
(D) I will not go for a mall and I will not visit my uncle's place, means that it is not raining.

Sol: All the options are having it is not raining. Considering that condition that it is not raining, the basic
question has two conditions with a conjunction or, In order to negate this both the implications given should
be individually negated with and. Hence the answer will be second option, which states it is not raining,
means that I will not go to a mall and I will not visit my uncles place.
Bulls Eye
64 www.hitbullseye.com

DS 08 - Logical Connectives
DIRECTIONS for questions 1 to 5: In each question, there is a main statement followed by four statements
(a), (b), (c) and (d). From the choices, choose the ordered pair where the first statement implies the second
statement and the two are logically consistent with the main statement.

1. Mohan would be selected in the company, if he has an excellent academic record.

(a) Mohan is selected in the company.


(b) Mohan is not selected in the company.
(c) Mohan has an excellent academic record.
(d) Mohan does not have an excellent academic record.

1. ac 2. bd 3. db 4. ad

2. Only if Suman has good knowledge of Maths, she would become a Mathematician.

(a) Suman does not have good knowledge of Maths.


(b) Suman has become a Mathematician.
(c) Suman has not become a Mathematician.
(d) Suman has good knowledge of Maths.

1. ca 2. db 3. bd 4. ba

3. Unless the Government seals the borders, illegal migration will not stop.

(a) Government sealed the border.


(b) Illegal migration has stopped.
(c) Government had not sealed the borders.
(d) Illegal migration will not stop.

1. cd 2. ab 3. cb 4. ad

4. Whenever Aman receives an sms from Aarti, he is on top of the world.

(a) Aman did not receive an sms from Aarti.


(b) Aman is on top of the world.
(c) Aman is not on top of the world.
(d) Aman received an sms from Aarti.

1. ba 2. bd 3. cd 4. ca

5. If the inflation is low, the GDP will touch a record high.

(a) The inflation is low.


(b) The inflation is not low.
(c) The GDP will touch a record high.
(d) The GDP will not touch a record high.

1. cb 2. ad 3. ca 4. db
Bulls Eye
www.hitbullseye.com 65

DIRECTIONS for questions 6 to 11: Each question given below is a statement followed by four different
statements. Choose the one which is the correct negation of the given statement.

6. Whenever Viru and Basanti go for movie, Jai follows them.

1. Jai follows Viru and Basanti but they are not going for movie.
2. Viru and Basanti are going for movie and Jai follows them.
3. Jai does not follow Viru and Basanti even when they go for movie.
4. None of these

7. Ajeet will attend the function, only if his father allows him to go by car.

1. Ajeet is not attending the function even his father allows him to come by car.
2. Ajeet's father did not allow him to go by car but he was attending the function.
3. Ajeet, is not attending the function because his father did not allow him to go on car.
4. None of these

8. Unless the meeting takes place, the problem will not be solved.

1. The problem is solved and the meeting did not take place.
2. The meeting took place but the problem is not solved.
3. The meeting took place and the problem is solved.
4. The problem is solved implies, the meeting took place.

9. Unless Mohit learns the concepts, he cannot solve LR.

1. Mohit learned the concepts but he could not solve LR.


2. Mohit did not learn the concepts, but he could solve LR.
3. Mohit learned concepts and solved LR.
4. Mohit did not learn concepts and he did not solve LR.

10. He either goes to cinema or he will study.

1. He went to cinema and did not study.


2. He went to cinema but studied.
3. He did not go to cinema and studied.
4. He did not go to cinema and did not study.

11. If you share your problems with your elders, you will be happy.

1. You did not share your problems with your elders and you are happy.
2. You shared your problems with your elders but you are not happy.
3. You did not share your problems with your elders and you are not happy.
4. You shared your problems with your elders, so you are happy.
Bulls Eye
66 www.hitbullseye.com

DIRECTIONS for questions 12 to 18: Each question below consists of a main statement followed by four
numbered statements. From the numbered statements, select the one that logically follows the main statement.

12. If it is a Thursday, then I will go to cinema or I will sleep.

1. It is not a Thursday, implies that I am not going to cinema and I am not sleeping.
2. It is a Thursday and I am not sleeping means that I am going to cinema.
3. I am not going to cinema and I am sleeping, means that it is a Thursday.
4. None of these

13. If the number of retail outlets is increased, then the prices will decrease or the consumption will
increase.

1. The prices did not decrease or the consumption did not increase implies that the number of retail
outlets is not increased.
2. The number of retail outlets is not increased means that the prices will not decrease and the
consumption will not increase.
3. The prices decreased or the consumption increased means that the number of retail outlets was
increased.
4. The prices did not decrease and the consumption did not increase implies that the number of retail
outlets is not increased.

14. Unless you return soon, you will be affected by sunstroke and the health will deteriorate.

1. You did not return soon, you will be affected by sunstroke and the health will deteriorate.
2. You are not affected by sunstroke or your health did not deteriorate, implies that you did not
return soon.
3. You are affected by sunstroke and your health deteriorated implies that you did not return
soon.
4. You are not affected by sunstroke and your health deteriorated means that you returned soon.

15. If the faculty is in the class, then the students will either study or keep quiet.

1. The students will not study or will not keep quiet, implies that the faculty is not in the class.
2. If the faculty is not in the class, then the students will not study and will not keep quiet.
3. The students will not study and will not keep quiet, implies that the faculty is not in the class.
4. If the faculty is not in the class, then the students will not keep quiet or will not study.

16. Whenever an accident occurs, either an injury or death takes place.

1. If a death or an injury takes place, then an accident must have occurred.


2. If a death did not take place and an injury did not occur, then an accident did not occur.
3. If an accident does not occur, neither an injury nor a death takes place.
4. If an accident occurs and death takes place, then injury does not occur.

17. If it is a working day, then I will be busy and be restless all day.

1. I was not busy or I was not restless all day, means that it was not a working day.
2. It is not a working day, means that I will be not be busy and I will not be restless all day.
3. I was not busy and I was not restless all day, means that it was not a working day.
4. I was busy and I was restless all day, means that it was a working day.
Bulls Eye
www.hitbullseye.com 67

18. Whenever pollution is on the rise, vehicles will be stopped and their emission levels will be checked.

1. Vehicles are not stopped or their emission levels are not checked, means that the pollution is not on
the rise.
2. If vehicles are not stopped but pollution is on the rise, then the emission levels of vehicles will
definitely be checked.
3. If vehicles are stopped but their emission levels are not checked, it means that the pollution is not
on the rise.
4. Both (1) and (3) above
Bulls Eye
68 www.hitbullseye.com

CHP 8 BINARY LOGIC


DS 09 Binary Logic
DIRECTIONS for questions 1 to 3: Select the correct alternative from the given choices.

Sameer, Sameep and Sumer participated in a quiz contest and each one of them received exactly one title
amongst the Winner, the First Runner-up and the Second Runner-up. When asked, "Who among you three is
the winner?", following were their replies:

Sameer: I am the winner.


Sameep is not the 1st Runner-up.
Sameep: I am the winner,
Sameer is the 2nd runner up.
Sumer: I am the winner.
Sameep is the 2nd runner-up.

It is also known that one among them always tells the truth, one always lies and one alternates between the
truth and lie (not necessarily in that order).

1. Who can never be the first runner-up?

1. Sameer 2. Sameep 3. Sumer 4. Cannot be determined

2. Each person out of A, B and C had exactly one different title amongst the Good, the Bad and the
Ugly. Also, each person always gave two answers to any question. Exactly one among them always
spoke the truth, another always lied and the last person always alternates between truth and lie (in any
order). When asked about their titles, following were their replies:

A: B is 'the Good'.
I am 'the Ugly'.
B: C is 'the Bad'
A is not 'the Good'.
C: B is 'the Ugly'.
A is not 'the Bad'.

Which among the following choices has the names of the persons who had the title the Good, the Bad
and the Ugly, respectively?

1. A, B, C 2. C, A, B 3. B, C, A 4. A, C, B

3. John, Johny and Janardan participated in a race and each won a different medal among Gold, Silver
and Bronze, not necessarily in that order. Each person among them gives two replies to any question,
one of which is true and the other is false (in any order). When asked about the details of the medals
obtained by them, the following were their replies:

John: I won the Gold medal.


Johny won the Bronze medal
Johny: John won the Silver medal.
Bulls Eye
www.hitbullseye.com 69

I won the Gold medal.


Janardan: Johny won the Silver medal.
I won the Gold medal.
Which among the following is the correct order of the persons who won the Gold medal, the Silver
medal and the Bronze medal respectively?

1. John, Johny, Janardan 2. Janardan, John, Johny


3. Johny, Janardan, John 4. Janardan, Johny, John

DIRECTIONS for questions 4 to 6: These questions are based on the following information.

Each of the three persons - Ramu, Raman and Rajan-belonged to exactly one of different cities amongst
Delhi, Mumbai and Chennai. Each person always gives two replies to any question asked. Out of these three,
exactly one person always speaks the truth, other always lies and the another always alternates between truth
and lie, in any order. When each was asked "Which city do you belong to?", the following were their replies:

Ramu: I am from Delhi. Raman is from Mumbai.


Raman: I am from Delhi. Rajan is from Chennai.
Rajan: Ramu is from Mumbai. Raman is from Delhi.
Based on the above, answer the following questions.

4. Who among the three must be from Chennai?

1.Ramu 2. Raman 3. Rajan 4. Cannot be determined

5. Which city did Raman belong to?

1. Delhi 2. Mumbai 3. Chennai 4. Chennai or Delhi

6. If there are exactly two persons who always tell the truth, and the third person either always lies or
alternates between truth and lie, then which of the following statements must be false?

1. Rajan is not from Mumbai 2. Ramu is not from Delhi.


3. Rajan is not from Chennai. 4. Raman is from Delhi

DIRECTIONS for questions 7 to 9: These questions are based on the following information.

There are four persons - A, B, C and D - each of whom plays exactly one game from amongst Cricket,
Football, Table-Tennis and Tennis. No. game is played by two persons. Each person gives two replies to any
question asked to them. At least one person among them always speaks the truth and at least one person
always tells lies. There is at least one person who always alternates between the truth and lie in any order.
When asked about the names of the persons and the respective games played by them, following were their
replies:

A : I play Cricket. C plays Cricket.


B : I play Tennis. D plays Tennis.
C : A plays Table-Tennis. B plays Cricket.
D : C plays Football. I play Table-Tennis.
It is also known that D plays Tennis and a definite arrangement can be obtained from the statements given
by each person.
Bulls Eye
70 www.hitbullseye.com

7. Who among the following plays Cricket?

1. A 2. B 3. C 4. D

8. Who among the following always speaks the truth?

1. A 2. B 3. C 4. D

9. Who are the two persons who always alternate between the truth and lie?

1. A and D 2. B and C 3. D and B 4. A and C

DIRECTIONS for questions 10 to 12: These questions are based on the following information.

P, Q and R when asked a question give three statements as reply in the following manner.
P - always replies in only one type of statements - truth or lie.
Q - never replies in the same type of statements as P.
R - is neither consistent in all his statement types nor alternates between the two types of statements.
At the bus - stop, I asked: "Which bus goes to the airport?"
Their replies were as follows:

P: Take the North - bound bus.


The next bus arrives in 15 minutes.
It takes 45 minutes to reach the airport.
Q: The north - bound bus comes from the airport.
You have to wait for 20 minutes for the bus.
It takes 30 minutes to reach the airport.
R: Take the South - bound bus.
You have to wait for another quarter of an hour for the bus.
It takes anywhere between 1 hour to 1 hour 30 minutes to reach the airport.

10. Which of the following represents the reply that R gave?

1. Truth - Truth - Lie 2. Lie - Truth - Truth 3. Lie-Lie-Truth 4. Truth-Lie-Lie

11. Which of the following is definitely true?

I. P always lies. II. Q always lies. III. The North - bound bus goes to the airport.

1. Only I 2. Only II 3. Only I and II 4. Only II and III

12. How long does it take before I reach the airport from the bus-stop?

1. 45 minutes 2. 50 minutes
3. Between 1 hour and 1 hour 30 minutes 4. 30 minutes
Bulls Eye
www.hitbullseye.com 71

DIRECTIONS for questions 13 to 15: These questions are based on the data given below.

There are four executives A, B, C and D each working in a different organization amongst Microsoft, Oracle,
Infosys and IBM, not necessarily in the same order. Each of them gives three replies to any question asked,
which are alternately true or false, in any order. When asked to mention the name of the executive and the
organization that he works for, they gave the following replies:

A: C works with Microsoft.


D works with IBM.
B works with Infosys.

B: A works with IBM.


C works with Infosys.
D works with Oracle.

C: D works with Microsoft.


B works with Oracle.
A works with Infosys.

D: B is not working with Infosys.


A is working with IBM.
C is not working with Microsoft.

A definite arrangement can be obtained from the above information. Answer the questions based on the above
data.

13. Who among the following works with Microsoft?

1. A 2. B 3. C 4. D

14. B works with

1. IBM 2. Infosys 3. Microsoft 4. Oracle

15. Which of the following statements must be false?

1. D is working with IBM.


2. The first statements made by A, B and C are false.
3. C is working in Infosys.
4. The second statements made by A, B and C are false.
Bulls Eye
72 www.hitbullseye.com

CHP 9 DATA CASELETS


Caselets are basically a set of linked questions -- there are two to five questions based on the same
information and the questions are related in some manner. Very often, caselets provide data within a
paragraph or a set of statements, rather than in the form of a table or graph. Caselets vary in length, as the
amount of information contained in provided in different sentences and paragraphs. While reading a caselet, it
is always advisable to underline the important facts and figures and if necessary, one can make his/her own
table/chart/graph for solving the questions.

Preparing the table can be time consuming and effort intensive. Hence, it is strongly advised that students
regularly practice a lot for these types of questions.

Hence the solution to the caselet often lies in making the correct table/chart for the data. There will be missing
numbers in the table, and so you will have to consider various possibilities for the possible values of these
missing numbers. Caselet questions are not easy to manage at times as the interpretation of required data in a
meaningful manner is left to student's ability to comprehend the hidden facts. In CAT and other competitive
exams, caselets can be asked either/both in Quant and Data Interpretation section and it is similar to the
Reading Comprehension part in English Usage section. The best way to master caselets is to practice and
practice. The below given examples would help you to understand the kind of questions, which appear in
exams and approach you should apply to solve those questions.

Illustration 1: Mohan took voluntary retirement in Dec. 2011 and received a certain amount of money as
retirement benefits. On Jan 1, 2012, he invested the entire amount in shares. At the end of the month, he sold
all his shares and realized 25% profit. On Feb 1, he reinvested the entire amount in shares which he sold at the
end of the month at a loss of 20%. Again, he invested the entire amount on Mar 1 in a new company. At the
end of the month, he sold the shares of the new company to a friend and realised a profit of 20% in the
process. He invested the entire amount in shares on Apr 1, which he sold at the end of the month for Rs.
1,08,000 incurring a loss of 10%.

Q1. What is the amount of retirement benefits received by Mohan?

1. Rs.1,08,000 2. Rs.1,25,000 3. Rs. 1,20,000 4.Rs. 1,00,000

Sol: Now as in this data caselet the information starts from the amount he received at the time of his
retirement, the question should start from there. So, let the amount received by Mohan in Dec. 2011
be Rs. 100, as retirement benefits:
Therefore, investment in the month of Jan 2012 = 100.
Profit of 25% at the end of Jan 2012.
Hence, investment in the month of Feb 2012 = 125.
Loss of 20% at the end of Feb 2012.
Hence, investment in the month of March 2012 = 100.
Profit of 20% at the end of March 2012.
Hence, investment in the month of April 2012 = 120.
Loss of 10% at the end of April 2012
Therefore the amount left at the end of April 2012 = 108. Now Rs. 100 received initially becomes Rs.
108 at the end of April. What sum of money will amount to Rs. 1,08,000 108000 100/108 =
1,00,000.
Bulls Eye
www.hitbullseye.com 73

Q2. The percentage profit received by Mohan between Jan 1 and Apr 30 is:

1. 8.00% 2. 15.00% 3. -10.00% 4. None of these

Sol: As we calculated above, that Rs. 100 received would have become Rs. 108 i.e. he gets a profit of Rs.
8 on his initial investment of Rs. 100. The percentage profit earned by him will be 100 8/100 = 8%.

Q3. The amount of loss incurred by Mohan based on his operation in Apr 2012 is:

1. Rs. 25,000 2. Rs. 12,000 3. Rs. 20,000 4. Rs. 8,000

Sol: As solved in the first question, he invested Rs. 120 in the month of April. Now as per the solution of
this equation, Rs. 120 would be Rs. 120000 in total as 100 corresponds to Rs. 100000. When he
invested Rs. 120000, he gets a loss of 10% in the month of April. So the loss made by him is 120000
10/100 = Rs. 12000.

Q4. The maximum amount invested by Mohan in any one month was in:

1. January 2. February 3. March 4. April

Sol: Now as in this data caselet the information starts from the amount he received at the time of his
retirement, the question should start from there. So, let the amount received by Mohan in Dec. 2011
be Rs. 100, as retirement benefits:

Therefore, investment in the month of Jan 2012 = 100.


Profit of 25% at the end of Jan 2012.
Hence, investment in the month of Feb 2012 = 125.
Loss of 20% at the end of Feb 2012.
Hence, investment in the month of March 2012 = 100.
Profit of 20% at the end of March 2012.
Hence, investment in the month of April 2012 = 120.
Loss of 10% at the end of April 2012.

Now as per this hypothetical value, he invested the largest amount in the month of February, which was Rs.
125. Let us calculate the corresponding value. If Rs. 108 in this assumed value is actually Rs. 108000, so Rs.
125 will be 125 108000/108 = Rs. 125000.

Illustration 2: An institute offers MBA with specialization in Marketing, Finance and HR. Among the total
number of students in the institute, 45% are girls. Number of boys studying Marketing is 30% of the total
number of boys in the institute which is 297. 40% of the girls are studying HR. Number of boys and girls
studying Marketing are in the ratio of 3: 2. 50% of boys are studying Finance.

Q1. How many girls are studying Finance?

Sol: Number of boys studying Marketing = 297. It is given that 30% of the boys in the institute study
Marketing. Let us calculate the total number of boys in the institute = (297 100)/30 = 990. It is
given in the question, that there are 45% girls in the institute, which implies there are 55% boys. Now
if the total number of boys is 990, the total number of girls will be (45/55) 990 = 810. Number of
girls studying HR are 40% of the total number of girls, which becomes = 810 40/100 = 324. The
ratio of number of boys and number of girls studying Marketing is given to be 3 : 2. The number of
Bulls Eye
74 www.hitbullseye.com

boys is given to be 297. So the number of girls studying Marketing would be = 297 2/3 = 198.
Number of boys studying Finance are half of the total (i.e. 50%, given in the question) = (1/2) 990
= 495. Number of boys studying HR can be found by subtracting the number of boys studying
Marketing and Finance from the total number of boys and there are three kinds of specialization are
available only i.e. = 990 297 495 = 198. Number of girls studying Finance can also be found in
the same manner = 810 324 198 = 288.

Q2. Number of girls studying Marketing is what percent of the boys studying Finance?

Sol: As calculated above the number of girls studying Marketing is 198 and the number of boys studying
Finance is 495. The required percentage is = (198/495) 100 = 40%.

Q3. Number of boys studying Finance is what percent of the total number of the students in the Institute?

Sol: Total number of the students in the Institute = 990 + 810 = 1800.
The number of boys studying Finance = 495. Now the percentage will be 100 495/1800 = 27.5%.

Q4. What is the respective ratio between number of boys and girls studying HR?

Sol: The number of boys studying HR is 198 and the number of girls studying HR is 324.
Required ratio = 198 : 324 = 11 : 18.
Bulls Eye
www.hitbullseye.com 75

DS 10 - Data Caselets
DIRECTIONS for the questions 1 to 3: Read the information given below carefully and answer the
questions that follow

Michael starts going to Church 100 days before the selection of final team for the tournament. Michael lights
candles to Christ on each day. Christ accepts these candles on the next day and leaves three of them, but
before Michael lights candles on that day. Michael takes the three candles left, and then lights candles for that
day. The number of candles accepted by Christ on the third day is 18 and it is known that on any day, the
difference between the number of candles lighted by Michael and that accepted by Christ is constant (No
candle is accepted by Christ on the first day)

1. How many candles does Christ accept in the first 40 days?

1. 7020 2. 7380 3. 6669 4. None of these

2. On which of the following days Michael lights 435 candles?

1. 46th day 2. 47th day 3. 48th day 4. 49th day

3. How many candles does Michael lights in the first 25 days?

1. 2400 2. 3200 3. 2800 4. 3000

DIRECTION for the questions 4 to 7: Read the information given below carefully and answer the questions
that follow.

In 2012, the sales of Tipla Ltd. witnessed a growth of 30% when compared to the previous year and reached
Rs. 1,716 crore. The growth percentage had tripled when compared to the corresponding figure of the
previous year. But, the same trend was not evident when profit was considered where there was a decrease of
25 percentage points in the percentage growth of profit from 2011 to 2012, when compared to that from 2010
to 2011. Profit as a percentage of sales for the company had still increased when compared to 2011, when it
was only 10%, which in turn was more when compared to 2010, when profit was a mere Rs. 30 crore. The
growth in the sales in 2012 (over 2011) was mainly owing to a 25% increase in the sales of cosmetics which
formed 50% of the sales in 2012. The only other two products Health supplement and beauty equipments
showed a percentage change of +40% and -25% respectively, when compared to 2011, and both of these had
a share of 30% and 20% respectively in the total sales in 2012. Coincidentally, in 2012, the sales in Q1 were
the same as in Q3 and the percentage growth in the sales from Q1 to Q2 was 50% and from Q3 to Q4 is 40%.
The profit in Q4 was the highest and the profits in Q1, Q2 and Q3 were 60%, 70% and 40% of that in Q4. Note:
Q1, Q2, Q3 and Q4 are the four quarters of any given year.

4. What was the value of the sales of Tipla Ltd. in 2010?

1. Rs. 1200 crore 2. Rs. 1,500 crore 3. Rs. 1,000 crore 4. Rs. 1,150 crore

5. In which quarter of 2012, was the profit, as a percentage of the sales, the highest?

1. Q1 2. Q2 3. Q3 4. Q4
Bulls Eye
76 www.hitbullseye.com

6. What was the value of the sales in Q2, in 2012?

1. Rs. 350.2 crore 2. Rs. 287.7 core 3. Rs. 525.3 crore 4. Rs. 421.8 crore

7. In 2011, the percentage share of the sales of beauty equipment in the total sales was approximately

1. 28.12% 2. 34.67% 3. 18.57% 4. 29.12%

DIRECTIONS for questions 8 to 11: These questions are based on the following information.

Each of the 600 students (either male or female) studying in exactly one of the two colleges: DAV or SD,
play exactly one of the two games Tennis or Cricket. Following information is also available regarding the
students:

45 male students studying in SD College play Cricket.


285 students play cricket.
150 students studying in SD College play Tennis.
150 male students studying in DAV College play either of the two games.
150 students who play Tennis are either females studying in SD College or males studying in DAV
College.
180 female students who study in SD either play cricket or who study in DAV play tennis.
90 male students who study in DAV College play cricket.

8. What is the number of female students who study in SD College play cricket?

1. 105 2. 75 3. 90 4. 60

9. What is the number of female students who study in DAV College play Tennis?

1. 105 2. 75 3. 90 4. 60

10. What is the total number of female students?

1. 270 2. 345 3. 255 4. 285

11. What is the number of male students who study in DAV College play Tennis?

1. 45 2. 90 3. 60 4. 30

DIRECTIONS for the questions 12 to 15: Read the information given below carefully and answer the
questions that follow.

P, Q, R and S were distributed 12 lottery tickets each bearing a different number among 1 to 12. No two
persons got the same number of lottery tickets. Both the lottery tickets owned by P bear even numbers. Also,
the number on any of the lottery tickets that R got is not the square of a prime number. Neither the lottery
ticket numbered 1 nor the lottery ticket numbered 9 is with Q. R received lottery ticket number 12. The sum
of the numbers on two lottery tickets of R is the same as the sum of the numbers on all the lottery tickets of P.
All the prime numbered lottery tickets are owned by Q.
Bulls Eye
www.hitbullseye.com 77

12. If a lottery ticket numbered 14 is given to anyone except to the person having the maximum number
of lottery tickets without violating any of the earlier conditions and the sum of numbers on all the
lottery tickets of R is more than that of Q, then, what is the sum of all the numbers on the lottery
tickets of P?

1. 16 2. 12 3. 18 4. Cant be determined

13. What is the minimum possible difference between the sum of numbers on all the lottery tickets of any
two persons?

1. 1 2. 3 3. 4 4. 5

14. Which of the following conditions is required to get the unique solution for the distribution?

1. Twice the total of numbers on all lottery tickets of P is equal to the total of numbers on all lottery
tickets of Q.
2. The sum of all the numbers on all the lottery tickets of one of the persons is a perfect cube.
3. The sum of numbers on all lottery tickets of R is 19.
4. Any one of the above

15. What is the maximum possible difference between the sum of numbers on all the lottery tickets of any
two persons?

1. 24 2. 21 3. 22 4. 23

DIRECTIONS for questions 16 to 20: Answer the questions on the basis of the information given below.

The following are the marks of five students - A, B, C, D and E taken three at a time - 250, 254, 256, 264,
266, 270, 270, 272, 276 and 286. The marks of each student is a distinct integer. It is known that E has the
maximum marks and A, B and D together have 32 marks less than C, D and E together.

16. What is the average marks of the five students?

1. 88.8 2. 88 3. 90.4 4. 92.6

17. Which student has third highest marks?

1. A 2. D 3. C 4. Cant be determined

18. What is the maximum marks of a student?

1. 100 2. 98 3. 94 4. 96

19. What is the difference in the marks of B and D?

1. 20 2. 10 3. 14 4. Cant be determined

20. The maximum difference between the marks of any two of the given students is

1. 18 2. 20 3. 22 4. 16
Bulls Eye
78 www.hitbullseye.com

CHP 10 DATA & REASONING


This variety is appearing in different MBA exams quite frequently these days. In this form of Data
Interpretation, the data would be provided in the form of Table, Bar Graph, Line Graph or Data Paragraph and
then you have to apply different level of reasoning to crack the set or to solve the questions. This form of data
requires a though understanding of the data given. In this kinds of sets, the questions require the thorough
knowledge of the concept of Sets and Venn diagram, Logical Connectives, Logical Deductions (Syllogism)
etc. Some of the major types of questions asked in Data and Reasoning are Maxima & Minima, Games &
Tournaments questions etc. The basic understanding of all the types of graphs and a deep understanding of the
various logics needed in reasoning are pre-requisites to solve these kinds of questions. The examples given
below would help you to get the idea about the kind of graph, which may appear under this category and the
approach to solve these kinds of questions.

Illustration 1: Read the information given below and answer the questions that follow.

A team of 5 players A, B, C, D and E participated in a tournament and played four matches (1 to 4). The
following table gives partial information about their individual scores and the total runs scored by the team in
each match.

Each column has two values missing. These are the runs scored by the two lowest scorers in that match. None
of the two missing values is more than 10% of the total runs scored in that match.

Match-1 Match-2 Match-3 Match-4


A 100 53
B 88 65 52
Runs scored by player C 110
D 72 75 20 56
E 60 78
Total 270 300 240 200

1. What is the maximum possible percentage contribution of A in the total runs scored in the four
matches, approximately?

1. 19.7% 2. 19.9% 3. 21.2% 4. 20.2%

Sol: As the score of A is already given in two of the matches i.e. match number 2 & 4, it means we only
need to guess the score in the remaining two matches.
Now his maximum possible score in Match 1 = 27 (i.e. 10% of the total of 270)
Similarly his maximum score in the Match 3 = 24 (i.e. 10% of 240). But if we carefully go through
the details of Match 3, we can say that, since A and B are the two lowest scorers, so each one of them
cannot score more than 19. So maximum score that A can contribute in Match 3 is 19.

Hence Maximum possible percentage contribution: 100 (27 +100 + 19 + 53)/(270+ 300 +240+200)
% = 100 199/1010 = 19.7%. So the answer is 1st option.
Bulls Eye 79
www.hitbullseye.com

2. If the absolute difference between the total runs scored by B and D in the four matches is minimum
possible, then what is the highest possible total score of A in the four matches?

1. 192 2. 194 3. 217 4. 198

Sol : Total score of D in the four matches is = 72 + 75 + 20 + 56 = 223.


Now as want to minimize the difference in D and B, we should try to make the total score of B as
close as possible to D.

The given score of B in three matches is = 88 + 65 + 52 = 205.


Now in the Match 3, the score of B can be taken as 18, so as to make the difference as 0.
Now the score of A in Match 3 will be = 240 (18 + 110 + 20 + 78) = 14.
The maximum score of A in Match1 can be taken as 27 (10% of 270).
The total score of A now is = 14 + 27 + 100 + 53 = 194.

3. The players are ranked 1 to 5 on the basis of the total runs scored by them in the four matches, with
the highest scorer getting Rank 1. If it is known that no two players scored the same number of total
runs, then how many players are there for whom ranks can be exactly determined?

1. 0 2. 1 3. 3 4. 5

Sol: The sum of three scores given in Match 1 is 88 + 72 + 60 = 220. The total of Match 1 is 270. The
remaining score is 270 220 = 50. As per the question 10% of the total i.e. 27 is the maximum score
one player can get. This means the minimum that the fifth player can get is 50- 27 = 23. This implies
that the score of A and C in Match 1 will be in between 23 27, inclusive.

The sum of three scores given in Match 2 is 100 + 65 + 75 = 240. The total of Match 2 is 300. The remaining
score is 300 240 = 60. As per the question 10% of the total i.e. 30 is the maximum score one player can get.
This means both the players C and E, will have a score of 30 each in the second match.

The sum of three scores given in Match 3 is 110 + 20 + 78 = 208. The total of Match 3 is 240. The remaining
score is 240 208 = 32. As per the question 10% of the total i.e. 24 is the maximum score one player can get.
But since these two people are the two lowest scorers, so each one of them cannot score more than 19. This
implies that the score of A and B in Match 3 will be in between 13 19, inclusive.

The sum of three scores given in Match 4 is 53 + 52 + 56 = 161. The total of Match 4 is 200. The remaining
score is 200 161 = 39. As per the question 10% of the total i.e. 20 is the maximum score one player can get.
This means the minimum that the fifth player can get is 39 - 20 = 19. This implies that the score of C and E in
Match 4 will be in between 19 20, inclusive.

As discussed above, Individual ranges for total score now will be:

A = 23 27, 100, 13 19, 53 = 189 199


B = 88, 65, 13 19, 52 = 218 224.
C = 23 27, 30, 110, 19 20 = 182 187.
D = 72, 75, 20, 56 = 223.
E = 60, 30, 78, 19 20 = 187 188.

C and E can have the same total score. As per the question, two players cannot have the same total score, as
the minimum of E is the same as the maximum of C that implies C will the total lesser than E. Hence C will
Bulls Eye
80 www.hitbullseye.com

have rank 5, E will have rank 4. A will have rank 3. Also we cannot determine the exact ranks of B and D.
Thus the rank of three persons can be found and two persons cannot be found.

Illustration 2: Read the information given below and answer the questions that follow.
There are 140 employees in an organization across five departments. The following table gives the
department-wise distribution of average age, average basic pay and allowances. The gross pay of an employee
is the sum of his/her basic pay and allowances.

Department Number of Average Age Average Basic Allowances (%


Employees (Years) Pay (Rs.) of Basic Pay)
HR 9 50 10000 80
Marketing 40 40 12000 80
Finance 30 35 13000 70
Business 35 47 15000 70
Development
Maintenance 26 40 11000 60

There are limited numbers of employees which are considered for transfer/ promotion across departments.
Whenever a person is transferred/promoted from a department of lower average age to a department of higher
average age, he/she will get an additional allowance of 10 of basic pay over and above his/her current
allowance. There will not be any change in pay if a person is transferred/ promoted from a department with
higher average age to a department with lower average age.
1. There was a mutual transfer of an employee between Marketing and Finance departments and transfer
of one employee from Marketing to HR. As a result, the average age of Finance department increased by
one year and that of Marketing department remained the same. What is the new average age of HR
department?

Sol: As the average age of the finance department has increased by one year that means the total increase in
the finance department is 30 years as there are 30 persons. This increase of 30 years in the finance department
implies a decrease of 30 years in the marketing department. Now one person from marketing has been
transferred to HR department. After this the average of marketing department remains the same, this means
the total decrease in marketing should be equal to the original average age, which was 40 years. There is a
decrease of 30 years due to mutual transfer between finance and marketing. This means the person transferred
from marketing to HR must have an age of 10 years. The total of HR without this person is 50 9 = 450. The
total of 10 persons, after this person is added will be 450 + 10 = 460. The final average age of the 10 persons
is 460/10 = 46 years.

2. What is the approximate percentage change in the average gross pay of the HR department due to
transfer of a 50-year old person with basic pay of Rs.15000 from the Marketing department?

Sol: Total pay of employee working in Marketing having age 50 years when he is transferred to HR
department will be equal to 15000 + 90% of 15000 = Rs 28500. Total pay of HR Department will be
(10000 + 80% of 10000) 9 = Rs 162000. Total pay of HR Department after inclusion of employee
from Marketing = 162000 + 28500 = Rs 190500. Average gross salary of HR Department =
190500/10 = 19050. Change in average salary = 19050 18000 = Rs 1050. Hence percentage change
1050 5
= 100 5 % .
18000 6
Bulls Eye 81
www.hitbullseye.com

3. If two employees (each with a basic pay of Rs. 10000) are transferred from Maintenance department
to HR department and three employees (with a basic pay of Rs. 12000) are transferred from
Marketing department to HR department, what will be the change in average basic pay of HR
department?

Sol: 2 persons transferred from Maintenance department to HR department will contribute 2 10000 = Rs
20000. Three persons transferred from Marketing department will contribute = Rs 36000. Total basic
salary of HR department = 10000 9 + 20000 + 36000 = Rs 146000. Average basic salary of HR
department = 146000/14 = Rs. 10428. Change in average salary is = Rs. 10428 10000 = Rs 428.

428
Hence the percentage change = 100 4.28%.
10000
Bulls Eye
82 www.hitbullseye.com

DS 11 Data & Reasoning


DIRECTIONS for questions 1 to 5: These questions are based on the following information.

Two professional badminton players Tanu and Manu participated in different annual tournaments held in
different countries in the years 2009, 2010, 2011 and 2012. In any tournament the winner is awarded 4 points
and the runner up is awarded 2 points. The following table gives the total number of points awarded to players
Tanu and Manu every year from 2009 to 2012.

2009 2010 2011 2012


Tanu 416 480 640 720
Manu 504 624 752 1048

Further it is known that the number of finals reached by 'Tanu' in 2010 is '160'. Also the number of times
'Tanu' stood as the runner up in a given year is equal to the number of times he stood as the winner in the
previous year. In any given year the number of times 'Tanu' reached finals is twice that of the number of times
'Manu' stood as the runner up in the same year.

1. How many times did Tanu reach finals in the years 2009 and 2010 together?

1. 240 2. 264 3. 312 4. 288

2. How many times did 'Manu' stand as the runner up in 2011 and 2012 together?

1. 210 2. 220 3. 240 4. 264

3. What was the total number of finals reached by 'Tanu' in 2011 and Manu in 2010?

1. 432 2. 292 3. 396 4. 286

4. How many times had 'Tanu' emerged as the winner throughout the given period?

1. 392 2. 398 3. 396 4. 400

5. What was the difference between the number of finals reached by Tanu and Manu in the given
period?

1.186 2. 198 3. 190 4. 192

DIRECTIONS for questions 6 to 10: These questions are based on the information given below.

Top 16 teams, A through P, participated in a knock out badminton tournament. There were four knock out
rounds - First round, quarter finals, semi finals and finals. A is the highest ranked (rank 1) player, B is the
second ranked (rank 2) player and so on P is the lowest ranked (rank 16) player. The following table gives the
information about the matches played in each round and the players playing the match.
Bulls Eye 83
www.hitbullseye.com

Round I

Match 1 2 3 4 5 6 7 8
Players Rank 1 Rank 2 Rank 3 Rank 4 Rank 5 Rank 6 Rank 7 Rank 8
Vs Rank Vs Rank Vs Rank Vs Rank Vs Rank Vs Rank Vs Rank Vs Rank
16 15 14 13 12 11 10 9

Quarter Final

Match 1 2 3 4
Players 1 vs 8 2 vs 7 3 vs 6 4 vs 5

Semi Final
Match 1 2
Players 1 vs 4 2 vs 3

Final
Match 1
Players 1 vs 2

In the table for the matches played in quarter final, semi final and final the number represents the winner of
that match number in the previous round. For example, in quarter final, the numbers 1 and 8 represents the
winners of the match number 1 and 8 of the previous round respectively.
An upset is said to be taken place, if a lower ranked player defeats a higher ranked player.

6. What is the maximum possible number of upsets in the tournament?

1. 14 2. 13 3. 15 4. 7

7. What is the minimum possible number of upsets in the tournament for G to be the winner of the
tournament?

1. 2 2. 3 3. 1 4. 4

8. Who among the following cannot be the players playing the finals in the tournament?

1. O, E 2. H, C 3. A, G 4. B, J

9. If A and K are playing in the final, then who among the following cannot be the other two players
playing the semi final?

1. D, B 2. M, J 3. E, O 4. N, D

10. If L is the winner of the tournament who among the following can be the runner up in the
tournament?

1. P 2. I 3. G 4. D
Bulls Eye
84 www.hitbullseye.com

DIRECTIONS for questions 11 to 14: These questions are based on the following information.

The table below gives the number of employees in a company in the years 2012 13 and 2013 14. The
company has 6 departments from A to F in this order and employees are recruited only in department A and
employees get transfer to a new place only if they qualify the personal interview after working in department
F. Every year the employees who pass the promotional exam are promoted to the next higher department
while employees who fail in the promotional exam have to work in the same department the next year also.

Deptt. 2012 - 13 2013 - 14


A 84 94
B 72 82
C 100 72
D 94 100
E 86 92
F 90 98

It is known that 76 employees got transfer to a new place in the year 2012-13.

11. How many employees joined the company in the year 2013-14?

1. 94 2. 92 3. 90 4. 88

12. In how many departments did exactly four employees fail in the exam in the year 2012-13?

1. 5 2. 3 3. 4 4. 2

13. How many employees in the company failed in the exam in the year 2012-13?

1. 34 2. 30 3. 36 4. 32

14. In which department was the pass percentage (number of employees who passed as a percentage of
total number of employees) the highest in the year 2012 - 13?

1. A 2. B 3. E 4. D

DIRECTIONS for questions 15 to 18: Answer these questions on the basis of the information given below.

The table gives the break-up of percentage of Salts and Minerals namely Sodium, Potassium, Magnesium and
Calcium contained in four foods - A, B, C and D.

Minerals & Sodium(S) Potassium(P) Magnesium(M) Calcium(C)


Salts
Food Item
A 20 30 10 40
B 10 40 30 20
C 15 25 35 25
D 40 15 35 10
The cost of food items A, B, C and D are respectively Rs. 100, Rs. 250, Rs. 200 and Rs.150 per serving. Two
or more of the items can be mixed in any ratio to produce an item containing the minerals and salts in the
required ratio.
Bulls Eye 85
www.hitbullseye.com

15. What is the least cost/litre of an item which contains at least 25% of Sodium?

1. Rs. 150 2. Rs. 125 3. Rs. 130 4. Rs. 112. 50

16. If an item which contains at least 20% of each of the minerals and salts S, P, M and C is to be
prepared, then what would be the least cost/serving incurred?

1. Rs. 100 2. Rs. 120 3. Rs. 125 4. Rs. 140

17. If an item which contains 25% of Magnesium is to be prepared at the least cost/serving, then the ratio
in which any two items have to be mixed is __________.

1. 1:2 2. 3:4 3. 2:3 4. 4:1

18. If not more than two servings of each item is available, then what can be the maximum percentage of
Potassium in an item obtained by mixing two or more foods, if the cost/serving of the resulting item
should not exceed Rs. 175?

1. 22.5 2. 25 3. 27.5 4. None of these

DIRECTIONS for questions 19 to 22: Those questions are based on the following data.

A renowned coaching institute Cats Eye is conducting an exam for a reputed NBA exam. The data of certain
students namely Ahmed, Bheem, Cheery, Dhawan, Guru and Farukh is given. The test had 50 questions in
total. Each correct answer is given 3 marks, while for each wrong answer 2 marks are deducted. 1 mark is
deducted for each question left unanswered. The following table gives details of the students who wrote the
test.

Student Attempted Un attempted Correct Net score


Ahmed 65
Bheem 28
Cherry 25
Dhawan 15
Guru 21
Farukh 32

19. What is the maximum net score of Bheem?

1. 60 2. 62 3. 72 4. 84

20. If Dhawan had a net score of 55, then which of the following is the number of correct answers he
had?

1. 25 2. 24 3. 28 4. Data Insufficient

21. If Farukh had the highest net score, then at most how many mistakes did he have?

1. 5 2. 4 3. 3 4. None of these
Bulls Eye
86 www.hitbullseye.com

22. Which of the following represents the correct range of the net score of Guru?

1. 34 to 63 2. 10 to 34 3. 5 to 63 4. None of these

DIRECTIONS for questions 23 to 27: Answer these questions on the basis of the information given below.

Company ABC is in the process of making a product Z. The production of Z requires the production of the
intermediate products P through Y. The following table gives the time required to produce the intermediate
products. These products can be produced simultaneously or in any order as long as the other products which
are required to be produced before them are produced. The final product will be produced when all the
intermediate products are produced.

Duration (in Other products to be produced before starting the production


Product
minutes) of new product
P 4 -
Q 2 -
R 6 P
S 4 Q, T
T 4 P, R
U 10 S, W
V 4 Q, R
W 14 P, R
X 8 S, T
Y 4 U

23. What is the shortest possible time, from the start in which product S can be produced?

1. 12 minutes 2. 10 minutes 3. 18 minutes 4. 15 minutes

24. The shortest time, from the start, in which product U can be produced is ________.

1. 24 minutes 2. 32 minutes 3. 30 minutes 4. 34 minutes

25. At most how many of the given products can be produced in ten minutes from the start?

1. 5 2. 8 3. 7 4. 3

26. What is the minimum number of products that must be produced, before product U can be
produced?

1. 4 2. 5 3. 6 4. 7

27. What is the minimum time, from the start, in which the last intermediate product will be produced?

1. 38 minutes 2. 28 minutes 3. 34 minutes 4. 40 minutes


Bulls Eye 87
www.hitbullseye.com

DIRECTIONS for questions 28 to 31: These questions are based on the following information.

Five students - Bhavesh, Chander, Dipankar, Lalita and Malti wrote a test comprising four Subjects
English, Hindi, Math and Science. The marks obtained by each of them are given along with their roll
numbers though the roll numbers are not in the same order in which the names are given.

Section Roll Numbers


1111 1112 1113 1114 1115
English 24 16 24 18 22
Hindi 24 28 26 30 20
Math 16 28 18 28 26
Science 28 32 20 22 24
Total 92 104 88 98 92

Further, it is known that


(i) Malti obtained the same marks in two of the subjects.
(ii) In one of the subjects the marks obtained by Chander and Lalita are the same,
(iii) Dipankar obtained his maximum marks in the Hindi.

28. What of can be said regarding the following two statements?

I. Dipankar is the person who scored the highest marks in the Hindi..
II. Malti is the person who scored the least marks in the Math.

1. If statement I is true, then statement II is definitely true.


2. If statement I is false, then statement II is false
3. If statement I is true, then statement II is false.
4. Both the statements are definitely false.

29. How many of the following statements are true, if Malti got the same marks in the English and the
Hindi?
I. Lalita got her least marks in the English.
II. Dipankar got his least marks in the English.

1. Exactly two 2. Exactly one 3. At most one 4. At least one

30. If the total marks obtained by Bhavesh and Chander are the same, then who got the least total marks?

1. Malti 2. Lalita 3. Bhavesh 4. Dipankar

31. Which of the following statements can be concluded from the given data?

I. The roll number of Malti is 1112. II. The roll number of Bhavesh is 1115.
III. The roll number of Dipankar is 1113.

1. Only l 2. Only II or III 3. Only II 4. Only I or II


ANALYTICAL & LOGICAL REASONING
(PREVIOUS SNAP QUESTIONS)
DIRECTIONS for questions 1 and 2:

Each question consists of a set of numbered statements. Assume that each one of these statements is
individually true. Each of the four choices consists of a subset of these statements. Choose the subset as
your answer where the statements therein are logically consistent among themselves:

1. A. Only if the water level in the coastal areas rises, then the people change their lifestyle.
B. People change their lifestyle only if they are rewarded.
C. If people are rewarded, then they will not change their lifestyle.
D. If the temperature rises, then the water level in the coastal areas rises.
E. Whenever the water level in the coastal areas rises, then the temperature rises.
F. Unless the people change their lifestyle, temperature rises.
G. People are rewarded.
H. Water level in the coastal area does not rise.

1. C, D, F, G and H 2. G, F, D, B and H 3. E, F, G, H and B 4. None of the above

2. A. If Kumar sings, then the audiences sleep.


B. If Kumar sings, then the audiences dance.
C. Unless audience do not dance, the concert will be successful.
D. Only if audience dance, the concert will be successful.
E. If Vina dances, then Kumar sings.
F. Kumar sings, only if Vina dances.
G. Vina dances.
H. The concert is successful.

1. C, F, G, B and H 2. A, C, F, G and H 3. E, C, G, B and H 4. Both B and C

DIRECTIONS for question 3 to 5: These questions are based on the data given below:

The letters of the English alphabet are numbered from 26 to 1 such that 26 stands for A, 25 stands for B,
and so on. The assigned numbers are used to write the letters of the original alphabet.

3. What will be the sum total of all vowels in the sequence?

1. 78 2. 84 3. 76 4. None of these

4. Which of the following Sequences denotes a valid word?

1. 6-12-17-23 2. 5-11-18-22 3. 5-12-18-23 4. 5-12-17-23

5. If each of the alphabets stands for the number which denotes it, what will be the next term in the
following sequence?
Z, W, R, K, ______

1. A 2. B 3. C 4. D
DIRECTIONS for questions 6 to 8: These questions are based on the following information.

A cube of 7 cm x 7cm x 7 cm is kept in the corner of a room and painted in three different colours, each
face in one colour. The cube is cut into 343 smaller but identical cubes.

6. How many smaller cubes do not have any face painted?

1. 125 2. 180 3. 144 4. 216

7. How many smaller cubes have exactly one colour on them?

1. 108 2. 72 3. 36 4. 24

8. How many smaller cubes have at the most two faces painted?

1. 343 2. 342 3. 256 4. 282

DIRECTION for question 9-10: In the diagram below, the circle stands for educated, square stands for
hard working, triangle for urban people and rectangle for honest. The different regions of the
diagram are numbered from 1 to 12. Study the diagram carefully and answer the questions:

5
3
1 6
11 2
4
12 9 8
10

9. Uneducated urban hard-working and honest people are indicated by:

1. 3 2. 11 3. 9 4. 4

10. Non-urban educated people who are neither hard-working nor honest are indicate by:

1. 5 2. 7 3. 10 4. 11

DIRECTIONS for questions 11 to 15: Refer to the following data and answer the questions that follow:

A numerical machine accepts two values X and Y. Then it updates these values as X = XY and Y = Y + l
in every step. The machine stops at X N.

11. For X = 3, Y = 2 and N = 100, how many steps are performed before the machine stops?

1. 2 2. 3 3. 4 4. 5
12. In the above question (11), what is the final value of X?

1. 6 2. 20 3. 72 4. 360

13. In the above question (11), what is the final value of Y?

1. 4 2. 5 3. 6 4. 20

14. If the value of N is changed to 500, what would be the final value of X ?

1. 360 2. 500 3. 560 4. 2160

15. If X = 2 and Y = 3, what should be the minimum value of N such that final value of Y is 7?

1. 300 2. 360 3. 720 4. 860

DIRECTIONS for questions 16 to 20: Refer to the following statements and answer the questions:

Seven students Priya, Ankit, Raman, Sunil, Tony, Deepak and Vicky take a series of tests. No two
students get similar marks. Vicky always scores more than Priya. Priya always scores more than Ankit.
Each time either Raman scores the highest and Tony gets, the least, or alternatively Sunil scores the
highest and Deepak or Ankit scores the least.

16. If Sunil is ranked sixth and Ankit is ranked fifth, which of the following can be true?

1. Vicky is ranked first or fourth 2. Raman is ranked second or third


3. Tony is ranked fourth or fifth 4. Deepak is ranked third or forth

17. If Raman gets the highest, Vicky should be ranked not lower than:

1. Second 2. Third 3. Fifth 4. None of these

18. If Raman is ranked second and Ankit is ranked first, which of the following must be true?

1. Sunil is ranked third 2. Tony is ranked third


3. Priya is ranked sixth 4. None of these

19. If sunil is ranked second, which of the following can be true?

1. Deepak gets more than Vicky 2. Vicky gets more than Sunil
3. Priya gets more than Raman 4. Priyagets more than Vicky

20. If Vicky is ranked fifth, which of the following must be true?


1. Sunil scores the highest 2. Raman is ranked second
3. Tony is ranked third 4. Ankit is ranked second
21. In 2002, according to a news poll, 36 % of the voters had leaning towards party "Y". In 2004, this
figure rose to 46%. But in another survey the percentage was down to 40%. Therefore, the party
"Z" is likely to win the next election. Which of the following, if true, would seriously weaken the
above conclusion?
1. People tend to switch their votes at the last minute
2. It has been shown that 85% of the voters belonging to the party "Y" vote in an election as
compared to 80% of the voters belonging to party "Z"
3. 35% of people favour party "Z"
4. No one can predict how people will vote

22. Inflation rose by 5% over the second quarter, by 4% during the first quarter and higher than 3%
recorded during the same time last year. However, the higher price index did not seem to alarm
National stock index, as stock prices remained steady.
Which of the following, if true, could explain the reaction of National stock Index?
1. RBI announced that it will take necessary corrective measures
2. Stock prices were steady because of a fear that inflation would continue
3. Economist warned that inflation would continue
4. Much of the quarterly increase in the price level was due to a summer drought effect on food
price

DIRECTIONS for question 23: Pick up the appropriate analogy.

23. Birth: Dirge

1. sunset : sunrise 2. security check : arrival


3. marriage : alimony 4. welcome address : vote of thanks

Beautiful beaches attract people, no doubt about it. Just look at this city's beautiful beaches, which
are among the most overcrowded beaches in the state.

24. Which of the following exhibits a pattern of reasoning most similar to the one exhibited in the
argument above?

1. Moose and bear usually appear at the same drinking hole at the same time of day. Therefore,
moose and bear must grow thirsty at about the same time.
2. Children who are scolded severely tend to misbehave more often than other children. Hence if
a child is not scolded severely that child is less likely to misbehave.
3. This software program helps increase the work efficiency of its users. As a result, these users
have more free time for other activities,
4. During warm weather my dog suffers from fleas more than during cooler weather. Therefore,
fleas must thrive in a warm environment.

25. No national productivity measures are available for underground industries that may exist but
remain unreported. On the other hand, at least some industries that are run entirely by self-
employed industrialists are included in national productivity measures.
From the information given above, it can be validly concluded that
1. There are at least some industries run entirely by self-employed industrialists that are
underground industries
2. No industries that are run entirely by self-employed industrialists operate underground
3. There are at least some industries other than those run entirely by self-employed industrialists
that are underground industries
4. There are at least some industries run entirely by self-employed industrialists that are not
underground industries

26. Nilu has never received a violation from the Federal Aviation Administration during her 16-year
flying career. Nilu must be a great pilot.

Which of the following can be said about the reasoning above?


1. The definitions of the terms create ambiguity.
2. The argument uses circular reasoning.
3. The argument is built upon hidden assumptions.
4. The argument works by analogy.
27. Many people argue that the death penalty deters murder. However, the notorious killer Tom
Hanks deliberately moved to a state that imposes the death penalty just before embarking on a
series of ferocious murders. Thus, it seems clear that the existence of the death penalty does not
serve as a deterrent to murder.
The argument above may best be characterized as:

1. An appeal to emotion.
2. A flawed analogy.
3. A general conclusion based on a specific example,
4. Circular reasoning.

28. What number should replace the question mark?

4 3 2
5 3 5 1 1
6 1 2 8 3 3 1
7 2 8 4 3
9 ? 3

1. 1 2. 4 3. 12 4. 6

29. The fewer restrictions there are on the advertising of legal services, the more lawyers there are
who advertise their services, and the lawyers who advertise a specific service usually charge less
for that service than lawyers who do not advertise. Therefore, if the state removes any of its
current restrictions, such as the one against advertisements that do not specify fee arrangements,
overall consumer legal costs will be lower than if the state retains its current restrictions.

If the statements above are true, which of the following must be true?

1. Some lawyers who now advertise will charge more for specific services if they do not have to
specify fee arrangements in the advertisements.
2. More consumers will use legal services if there are fewer restrictions on the advertising of legal
services.
3. If the-restriction against advertisements that do not specify fee arrangements is removed, 'more
lawyers will advertise their services.
4. If more lawyers advertise lower prices for specific services, some lawyers who do not advertise
will also charge less than they currently charge for those services.

30. Which of the following, if true, would most seriously weaken the argument concerning overall
consumer legal costs?

1. The state is unlikely to remove all of the restrictions that apply solely to the advertising of legal
services.
2. Lawyers who do not advertise generally provide legal services of the same quality as those
provided by lawyers who do advertise.
3. Most lawyers who now specify fee arrangements in their advertisements would continue to do
so even if the specification were not required,
4. Most lawyers who advertise specific services do not lower their fees for those services when
they begin to advertise.

31. Four children A, B, C & D are having some chocolates each.


A gives B as many as he already has, he gives C twice of what C already has and he gives D
thrice of what D already has.
Now, D gives 1/8th of his own chocolates to B.
Then A gives 10% chocolates he now owns to C and 20% to B.
Finally, all of them have 35 chocolates each.
What is the original number of chocolates each had in the beginning?

1. A-110, B-10, C-10, D-10 2. A-90, B-20, C-20, D-10


3. A-70, B-25, C-25, D-20 4. A-125, B-5.C-5, D-5

32. There are two similar figures below with some numbers. The left one is complete whereas one
number is missing in the right one. Find a suitable number to fill in place of the question mark.

102 203

09 90 07 70

201 ?

1. 280 2. 303 3. 362 4. 382

33. Complete the following series by replacing the '?:(TBLD. VEPI, XHTN, ? )

1. ZJVP 2. ZVJP 3. ZKXS 4. ZKXP

34. In a cricket team, three batsmen Ricky, Sachin and Brian are the top three run-scorers in any
order. Each of them gives two replies to any question, one of which is true and the other is false,
again, in any order. When asked about who the top scorer was, following were the replies they
gave:
Sachin: I got the top score. Ricky was second.
Brian: I got the top score. Sachin was second.
Ricky: I got the top score. Sachin was third.

Which of the following is the correct order of batsmen who got the top score, second best and
third best score respectively?

1. Brian, Ricky, Sachin 2. Brian, Sachin, Ricky


3. Ricky. Sachin, Brian 4. Sachin, Brian, Ricky

35. 60 employees in an office were asked about their preference for tea and coffee. It was observed
that for every 3 people who prefer tea, there are 2 who prefer coffee. For every 6 people who
prefer tea, there are 2 who drink both of tea and coffee. The number of people who drink both is
the same as those who drink neither. How many people drink both tea and coffee?

1. 10 2. 12 3. 14 4. 16

36. A clock strikes once at 1 o'clock, twice at 2 o'clock and so on. If it takes 6 seconds to strike at 3
o'clock, how much time will it take to strike at 9 o'clock?

1. 24 seconds 2. 18 seconds 3. 20 seconds 4. None of these

DIRECTIONS for questions 37 & 38: Read the following information and answer the questions that
follow.

E-1, E-2 and E-3 are three engineering students writing their assignments at night. Each of them starts at a
different time and completes at a different time. The digit in their name and the order of their starting and
completing the assignment is certainly not the same. The last student to start is the first to complete the
assignment.

37. Who is the first student to start writing the assignment?

1. E-1 2. E-2 3. E-3 4. Cannot be decided

38. Who is the last student to complete the assignment?

1. E-1 2. E-2 3. E-3 4. Cannot be decided

DIRECTIONS for questions 39 & 40: Read the following information and answer the questions that
follow.

A, B and C are three students from Don School and P, Q and R are three students from Elite School. Q is
brighter than R but duller than the Don School student who is brighter than A. The same Don School
student is duller than P but is brighter than C.

39. Who is brightest amongst all?

1. B 2. P 3. R 4. Cannot be decided

40. Who is the dullest amongst the three students from Elite School?

1. P 2. Q 3. R 4. Cannot be decided
41. When Rafael entered the class, there were already 10 students in the class. 5 students entered the
class between Roger and Rafael. Total 10 students entered after Roger. Exactly how many
students are in the class finally?

1. 15 2. 25 3. 27 4. Cannot be decided

DIRECTIONS for questions 42 to 44: Read the following information and answer the questions that
follow.

Arijit, Biplab, Chintan, Debashish, Elangovan, Frederick, Gautam and Himadri are sitting around a
circular table. Some information about the order in which they are sitting is available as follows:
1. Debashish is sitting opposite to Himadri and to the immediate right of Gautam.
2. Elangovan is sitting to the immediate right of Biplab.
3. Arijit is sitting opposite Chintan who is not immediately next to Frederick on either side.

42. Who is sitting to the immediate right of Himadri?

1. Arijit 2. Debashish 3. Elangovan 4. Frederick

43. Who is sitting opposite Biplab?

1. Arijit 2. Debashish 3. Frederick 4. Himadri

44. Who is to the immediate right of Chintan?

1. Arijit 2. Biplab 3. Elangovan 4. Himadri

45. Select the alternative that logically follows the two given statements:
Some rocks are not tables
Some rocks are balloons

1. Some tables are not balloons 2. Some tables are balloons


3. Some balloons are not tables 4. None of the above

DIRECTIONS for Questions 46 & 47: Read the following information and answer the questions that
follow.

A, B, C, D and E sit on a long bench. C does not sit next to A or E. A and E have three persons sitting
between them.

46. Who is sitting in the middle of the bench?

1. B 2. C 3. D 4. None of these

47. Who are sitting at the extreme ends of the bench?

1. A&E 2. B&D 3. C&E 4. None of these


DIRECTIONS for questions 48 & 49: Observe the chart below and answer the following questions.

I Yearly Commission Earned by Five Salesmen.


Figures in
Rupees
Year 1990 1991 1992 1993 1994 1995
Salesman
A 27350 28500 25200 29800 24600 27000
B 26850 27900 27400 28000 28500 29000
C 26200 27900 28200 29100 29400 30000
D 27850 30040 29800 30060 29800 32000
E 28640 29000 28750 30000 29750 29700

48. In the year 1994, the commission earned by salesman D was approximately what per cent more of
the commission earned by A?

1. 18 2. 82.5 3. 21 4. 17

49. In the year 1993, the commission of B was approximately what per cent of the total commission
earned by five salesmen that year?

1. 30 2. 20 3. 40 4. 80

50. Find the Missing Numbers in the following set

2 4 6 8 10
2 14 34 ?? 98

1. 30 2. 62 3. 42 4. 78

51. There are 6 volumes of books on a rack kept in order (such as vol. 1, vol.2 and so on).
After some readers used them, their order got disturbed. The changes showed as follows:

Vol.5 was directly to the right of vol. 2


Vol.4 has vol. 6 to its left and both were not at Vol.3's place
Vol.3 has Vol.4 on right and Vol.5 on left
An even numbered volume is at Vol.5's place

Find the order in which the books are kept now, from the 4 given alternatives :

1. 6, 3.5, 1, 4, 2 2. 4, 6, 3, 5, 1, 2 3. 3, 4, 1, 6, 5, 3 4. 2, 5, 1, 3, 6, 4

52. All German philosophers, except for Marx, are idealists.


From which of the following can the statement above be most properly inferred?

1. Except for Marx, if someone is an idealist philosopher, then he or she is German.


2. Marx is the only non-German philosopher who is an idealist.
3. If a German is an idealist, then he or she is a philosopher, as long as he or she is not Marx.
4. Marx is not an idealist German philosopher.
53. Ramaswami was studying for his examinations and the lights went off. It was around 1:00 a.m.
He lighted two uniform candles of equal length but one thicker than the other. The thick candle is
supposed to last six hours and the thin one two hours less. When he finally went to sleep, the
thick candle was twice as long as the thin one.
For how long did Ramaswami study in candle light?

1. 2 hours 2. 3 Hours 3. 2 hours 45 minutes 4. 4 hours

54. The numerator and denominator of a fraction is in the ratio 2:3. If 6 are subtracted from the
numerator the value of the fraction becomes 2/3 of the original fraction. The numerator of the
original fraction is,

1. 16 2. 21 3. 18 4. 30

55. A person wanted to withdraw X rupees and Y paise from the bank. But cashier made a mistake
and gave him Y rupees and X paise. Neither the person nor the cashier noticed that. After
spending 20 paise, the person counts the money. To his surprise, he has double the amount, he
wanted to withdraw.
Find X and Y. (1 Rupee = 100 Paise)

1. X = 3, Y = 6 2. X = 26, y = 53 3. X = 15, Y = 30 4. X = 9, Y = 36

56. A drawer contains 10 black and 10 brown socks which are all mixed up. What is the fewest
number of socks you" can take from the drawer without looking and be sure to get a pair of the
same color?

1. 7 pairs 2. 7 pieces only 3. 10 pieces only 4. 3 pieces only

57. A placement company has to assign 1000 SW personnel who are skilled in Java and Dot Net to a
prospective outsourcing company. He finds that 750 have Dot Net skills and 450 have Java skills.
Some have skills in both Java and Dot Net. Find the numbers who have skills in both Java and
Dot Net.

1. 250 2. 200 3. 350 4. 100

58. All good athletes who want to win are disciplined and have a well balanced diet. Therefore
athletes who do not have well balanced diets are bad athletes.
Based on the sentence above which of the statement below strongly supports the view:

1. No bad athlete wants to win


2. No athlete who does not eat a well balanced diet is good athlete
3. Every athlete who eats a well balanced diet is good athlete
4. All athletes who want to win are good athletes.
59. The numbers in these series are arranged in a triangle which has a logic as shown below. Find the
missing numbers shown as (?) from the choices given below:
2
2 2
2 4 2
2 8 ? 2
2 16 64 16 2
2 32 1024 ? ? 2

1. {16, 32, 64} 2. {8, 1024, 32} 3. {24, 1024, 64} 4. {16, 32, 64, }

60. If for a particular value of the variable x, the following holds good, 17 = 17x/(1-x), then compute
the value of (2x)*x.

1. 17 2. 1 3. 2 4.

61. A band passes around all the wheels so that they can all be turned by the driving wheel. When the
driving wheel turns in the direction shown, which way will the wheel B turn?
A
B

G
D E F

1. Clockwise 2. Cannot move 3. Anti-clockwise 4. Either way

62. In a certain language, (A) Sun shines brightly is written as ba lo sul; (B) House are brightly
lit as kado udo ari ba; and (C) Light comes from sun as dapi kup lo nro. What words will be
written for sun and brightly?

1. Lo, ba 2. ba, lo 3. snl, lo 4. ba, sul

63. The Drawing shows a cross section where the land meets the sea. The section covered is 5
kilometers. On a hot day, in which direction, indicated by four arrows, is the wind most likely to
blow?
A B

Land D Sea

1. A 2. B 3. C 4. D
64. Each child in family has at least 4 brothers and 3 sisters. What is the smallest number of children
the family might have?

1. 7 2. 8 3. 9 4. 10

65. In the following question two statements are followed by two conclusions numbered I and II.
Assume the two statements are true even if they are at variance with commonly known facts.
Then pick the correct answer from the choices given below.

A. Only conclusion I follows B. Only conclusion II follows


C. Both conclusions I & II follows D.Neither conclusion I nor conclusion II follows

Statements: Some doctors are fools. Joshi is a doctor


Conclusions: I. Joshi is a fool.
II. Some fools are doctors.

1. A 2. B 3. C 4. D

66. Debu walks towards east then towards North and turning 450 right walks for a while and lastly
turns towards left. In which direction is he walking now?

1. North 2. East 3. South-East 4. North-West

DIRECTIONS for question 67 69: Read the following instructions and answer.

I. There is a rectangular wooden block of length 4 cm, height 3 cm and breadth 3cm.
II. The two opposite surfaces of 4 cm 3 cms are painted yellow on the outside.
III. The other two opposite surfaces of 4 cm 3 cm are painted red on the outside.
IV. The remaining two surfaces of 3 cm 3 cm are painted green on the outside.
V. Now, the block is cut in such a way that cubes of 1 cm 1 cm 1 cm are created.

67. How many cubes will have only one colour?

1. 10 2. 12 3. 14 4. 18

68. How many cubes will have no colour?

1. 1 2. 2 3. 4 4. 8

69. How many cubes will have any two colour?

1. 32 2. 24 3. 16 4. 12

70. Read the following about the grid given and answer.
The cells in this grid contain the digits 1 to 9 in random order.
Column A contains no odd digits.
Cell C3 minus Cell C2 equals 4.
The sum of three digits in Row 1 is 17.
Number 7 is in column B; its left hand neighbor is 4
The digits of Column C add upto 14.
2 is not in the same horizontal row as 8; and 9 is not immediately below 3.

Which cell holds the number 9?

1. B1 2. B3 3. C2 4. C1

A B C
1
2
3

71. Replace the question mark with the right option.


4, 32, 288, ? , 31680

1. 25600 2. 2880 3. 7420 4. 10000

72. In the Sunday bazzar, Jamuna sells her lemons at Rs. 0.50 for two. Her neighbor Seema has a
little smaller lemons; she sells hers at Rs. 0.50 for three. After a while, when both ladies have the
same number of lemons left, Seema is called away. She asks her neighbor to take care of her
goods. To make things simple, Jamuna puts all lemons in one big pile, and starts selling five
lemons per one rupee. When Seema returns, at the end of the day, all lemons have been sold. But
when they start dividing the money, there appears to be a shortage of Rs. 3.50. Supposing they
divide the money equally, how much does Jamuna lose with this deal?

1. Rs. 10.50 2. Rs. 11.50 3. Rs. 42.00 4. Rs. 52.50

73. There are two cups, one containing orange juice and one containing an equal amount of
lemonade. One teaspoon of the orange juice is taken and mixed with the lemonade. Then a
teaspoon of this mixture is mixed back into the orange juice. Is there more lemonade in the
orange juice or more orange juice in the lemonade?

1. More orange juice in the lemonade?


2. More lemonade in the orange juice
3. Equal amount of each juice between the two cups
4. None of the above

74. Consider the statement and decide which of the assumptions are implicit:-

In the present period of economic hardships, education and small family norm may lead the
nation to progress and prosperity.

Assumptions: I. Education and small family norms are directly related to nations progress.
II. big families find it difficult to bear the cost of education.

1. Only I is implicit 2. Only II is implicit


3. Both I & II are implicit 4. Neither I nor II is implicit
75. Fill in the blanks to find two words that are synonyms

T I T I
M N

E E

D O C E

1. KS, ST 2. MS, NT 3. ST, DN 4. MS, DN

76. Beautiful beaches attract people, no doubt about that. Just look at the citys most beautiful
beaches, which are amongst the most overcrowded places in the state.
Which of the following exhibits a pattern of reasoning similar to the one exhibited in the
argument above?

1. Moose and bear usually appear at the same drinking hole at the same time of day. Therefore,
moose and bear must be feeling thirsty at about the same time.
2. Children who are scolded severely tend to misbehave more often than other children. Hence if
a child is not scolded severely, that child is less likely to misbehave
3. During warm weather my dog suffers more fleas than during cool weather. Therefore, fleas
must thrive in a warm environment
4. Tally accounting software helps increase the work efficiency of its users. As a result, these
users have more time for other activities.

77. Abdul, Mala, and Chetan went bird watching. Each of them saw one bird that none of the others
did. Each pair saw one bird that the third did not. And one bird was seen by all three. Of the birds
Abdul saw, two were yellow. Of the birds Mala saw, three were yellow. Of the birds Chetan saw,
four were yellow. How many yellow birds were seen in all? How many non-yellow birds were
seen in all?

1. 7 yellow birds and 3 non yellow birds 2. 5 yellow birds and 2 non yellow birds
3. 4 yellow birds and 2 non yellow birds 4. 3 yellow birds and 2 non yellow birds

78. Consider the following statements and answer the question.


M, N, O and P are all different individuals
M is the daughter of N.
N is the son of O
O is the father of P.
Which among the following statements is not contradictory to the above premises?

1. P is the father of M. 2. O has three children.


3. M has one brother. 4. M is the granddaughter of O.
79. A, B, C, and D are standing on the four corners of a square field as shown in the figure. From the
positions shown in the figure, A walks to the North position and B walks to the East position
while C decides to walk two sides in anticlockwise direction. B walks to North and then changes
his mind to take the previous position. Identify the choice with correct positions.
B C
N

A D

1. A & B occupy the same position. 2. C & D occupy the same position.
3. D & B are in their original position. 4. B & C are in diagonally opposite position.

80. A gambler bet on a horse race, but the bookee wouldnt tell him the results of the race. The
bookee gave clues as to how the five horses finished which may have included some ties and
wouldnt pay the gambler off unless the gambler could determine how the five horses finished
based on the following clues:
o Penn Fe finished before Night Marvel and after Wish Bones.
o Wish Bones tied with Penn Fe if and only if Hallelujah did not tie with Sundae.
o Penn Fe finished as many places after Sundae as Sundae finished after Wish Bones if and
only if Wish bones finished before Night Marvel.

The gambler thought for a moment, then answered correctly. How did the five horses finish the
race?

1. Sundae came in first. Which Bones and Hallelujah tied for second place. Penn Fe came in
fourth. Night Marvel came in fifth.
2. Wish bones came in first. Sundae and Penn Fe tied for second place. Hallelujah came in
fourth. Night Marvel came in fifth.
3. Wish Bones came in first. Sundae and Hallelujah tied for second place. Penn Fe came in
fourth. Night Marvel came in fifth.
4. Penn Fe came in first. Night Marvel and Hallelujah tied for second place. Wish bones came in
fourth. Sundae came in fifth.

81. In a school drill, a number of children are asked to stand in a circle. They are evenly spaced and
the 6th child is diametrically opposite the 16th child. How many children are made to stand in the
circle?

1. 16 2. 20 3. 22 4. None of the above

82. In this question insert the missing number at the sign of interrogation.

8 4 9 5
5 7 3 4
3 4 5 8
39 44 60 ?

1. 62 2. 72 3. 60 4. 70
83. Steel cylinders are made so that each one has a large and small hole through the middle. In the
drawing six cylinders have been stacked on top of each other. To stop the cylinders from rolling
on the smooth floor they are wedged by heavy blocks at each side of bottom row. If the heavy
blocks are removed what would be the position of cylinders when they stopped rolling?

A B C D

1. A 2. B 3. C 4. D

DIRECTIONS for question 84 86: Use the information given below to answer.

I. There is a group of 5 persons A, B, C, D and E


II. In the group there is one badminton player, one chess player and one tennis player
III. A and D are unmarried ladies and do not play any games
IV. No lady is a chess player or a badminton player
V. There is a married couple in the group of which E is the husband
VI. B is the brother of C and is neither a chess player nor a tennis player.

84. Which of the groups has only ladies?

1. ABC 2. BCD 3. CDE 4. None of the above

85. Who is the tennis player?

1. B 2. C 3. D 4. E

86. Who is the wife of E?

1. A 2. B 3. D 4. None of the above


Bulls Eye
www.hitbullseye.com

Answer keys

DS 01 DS 02 DS 03 DS 04 DS 05 DS 06
1. 3 1. 1 1. 4 1. 2 1. 4 1. 2
2. 4 2. 2 2. 1 2. 5 2. 1 2. 3
3. 1 3. 3 3. 2 3. 1 3. 2 3. 5
4. 4 4. 2 4. 3 4. 3 4. 4 4. 1
5. 1 5. 4 5. 1 5. 1 5. 3 5. 3
6. 3 6. 3 6. 4 6. 5 6. 4 6. 2
7. 4 7. 5 7. 3 7. 4 7. 2 7. 2
8. 2 8. 3 8. 2 8. 1 8. 2 8. 4
9. 1 9. 1 9. 5 9. 4 9. 4 9. 3
10. 4 10. 2 10. 2 10. 1 10. 5 10. 3
11. 3 11. 3 11. 3 11. 4 11. 4 11. 5
12. 5 12. 5 12. 1 12. 3 12. 4 12. 2
13. 1 13. 1 13. 5 13. 4 13. 1 13. 5
14. 4 14. 2 14. 2 14. 5 14. 3
15. 1 15. 2 15. 5 15. 2 15. 4
16. 3 16. 2 16. 3 16. 1 16. 5
17. 1 17. 4 17. 1 17. 4
18. 3 18. 1 18. 1 18. 4
19. 3 19. 5 19. 5 19. 2
20. 2 20. 2 20. 4 20. 4
21. 3 21. 5 21. 4 21. 3
22. 4 22. 5 22. 2 22. 4
23. 5 23. 1 23. 4 23. 1
24. 2 24. 1 24. 1 24. 4
25. 5 25. 2 25. 3 25. 2
26. 3 26. 3 26. 4
27. 3 27. 2 27. 2
28. 4 28. 1 28. 3
29. 1 29. 2 29. 2
30. 4 30. 3 30. 3
31. 3 31. 1 31. 3
32. 3 32. 2 32. 4
33. 3 33. 1 33. 4
34. 4 34. 3
35. 1 35. 2
36. 5 36. 2
37. 5 37. 2
38. 2 38. 1
39. 1 39. 2
40. 3 40. 3
41. 1
42. 4
43. 5
44. 4
45. 4
46. 3
47. 2
48. 1
49. 4
50. 3
Bulls Eye
www.hitbullseye.com

DS 07 DS 08 DS 09 DS 10 DS 11
1. 4 1. 2 1. 2 1. 1 1. 4
2. 5 2. 3 2. 4 2. 3 2. 2
3. 5 3. 1 3. 2 3. 4 3. 3
4. 1 4. 4 4. 3 4. 1 4. 4
5. 5 5. 4 5. 2 5. 4 5. 1
6. 3 6. 3 6. 3 6. 3 6. 3
7. 4 7. 2 7. 2 7. 2 7. 2
8. 2 8. 1 8. 3 8. 2 8. 4
9. 1 9. 2 9. 3 9. 1 9. 4
10. 4 10. 4 10. 4 10. 2 10. 3
11. 2 11. 2 11. 1 11. 3 11. 4
12. 5 12. 2 12. 2 12. 3 12. 2
13. 4 13. 4 13. 1 13. 1 13. 1
14. 4 14. 1 14. 4 14. 4 14. 3
15. 2 15. 3 15. 4 15. 4 15. 4
16. 2 16. 1 16. 2
17. 1 17. 3 17. 3
18. 4 18. 2 18. 4
19. 4 19. 2
20. 3 20. 3
21. 4
22. 4
23. 3
24. 4
25. 4
26. 3
27. 1
28. 3
29. 2
30. 2
31. 3

Actual Previous Year SNAP


1. 2 30. 4 59. 2
2. 3 31. 1 60. 4
3. 2 32. 1 61. 2
4. 3 33. 3 62. 1
5. 2 34. 1 63. 4
6. 4 35. 2 64. 3
7. 1 36. 1 65. 2
8. 2 37. 3 66. 4
9. 4 38. 1 67. 1
10. 2 39. 2 68. 2
11. 3 40. 3 69. 3
12. 4 41. 4 70. 2
13. 3 42. 1 71. 2
14. 4 43. 3 72. 1
15. 3 44. 2 73. 3
16. 4 45. 4 74. 1
17. 3 46. 2 75. 4
18. 4 47. 1 76. 3
19. 1 48. 3 77. 2
20. 1 49. 2 78. 4
21. 3 50. 2 79. 4
22. 4 51. 4 80. 3
23. 3 52. 4 81. 2
24. 4 53. 2 82. 2
25. 4 54. 3 83. 3
26. 3 55. 2 84. 4
27. 3 56. 4 85. 2
28. 4 57. 2 86. 4
29. 3 58. 2
Bulls Eye
www.hitbullseye.com

14. Each letter moves +4, +3, +2, ......


Explanations 15. Alphabet is written in reverse
16. Letters are reversed and then move 1.
17. Each letter moves 2
DS 01 18. The letters move 3, +3, .....
1. Logic is 41 + 1 = 5, 5 2 + 4 =14, 14 3 + 9 = 51, 51 4 19. Letters are written in reverse and move +2.
+ 16 = 220. So in place of 6 it should be 5 20. Letters are written in series of 9 i.e. A = 1, B = 2, C = 3,
2. The logic is 4 3 5 = 7,7 3 5 = 16, 16 3 5 = 43, ..... I = 9, J = 1, ....
So in place of 46 it should be 43 21. O(P)Q, QS, .....
3. Logic is 4 .5 = 2, 2 1.5 = 3,3 2.5 = 7.5, 22. Alternate letters move +2.
7.5 3.5 = 26.25, 26.25 4.5 = 118.125, 118.125 5.5 = 23. Series is +5, -4, +3,......
649.6875, In place of 118.725 it should be 118.125 25. Letters are reversed
4. Logic is 2 3 + 5 = 11, 11 4 6 = 38, 38 5+7 = 197, 26. Series is 3, +3, ......
197 6-8 = 1174, 1174 7 + 9 = 8227, Thus the wrong 27. Alternate letters move +1, -1, +1....
number is 1172 , it should be 1174 28. Series is +3, -3, ......
5. Starting from the back 29. In every triplet, first and third letters are interchanged.
107 71 = 36, 71- 46 = 25, 46 30 = 16, 30 21 = 9, 21 30. hupa chip = is very from (1). Hari = Kupa or
17 = 4, 17 16 = 1, Every difference is perfect square tile. But tile is in 4th statement. Hence Hari = Kupa.
starting from 36 and decreasing. 31. From the above.
Thus the wrong number is 19, it should be 17. 32. Quo = is from (1) and (2). Cui = good from (1) and
6. Logic is 4 .5 = 2, 2 1.5 = 3, 32.5 = 7.5, 7.53.5 = (4). Hence boy = heer.
26.25, 26.25 4.5 = 118.125. Thus the wrong number is 33. From the above.
3, it should be 3.5. 34. Oranges = B. Hence Red = E or F.
7. Logic is 2 3 = 6,6 3 = 18, 18 6 = 108, 108 18 = 35. xas = green. Bom = lovely. Hence pasture = pere.
1944 Thus the wrong number is 109 , it should be 108 36. pic = winter. Hence summer = re from(2) and (3).
8. Logic is 1 2 + 1 = 3, 3 2 + 0 = 6, 6 2 1 = 11, 11 2 37. B = oranges. Good = K or U.
- 2 = 20, 20 2 3 = 37, 37 2 4 =70, Thus the wrong 38. he = wir, since all other words figure in the given
number is 39, it should be 37. statements except (2).
9. Logic is 2 2 + 7 = 11, 11 3 6 = 27, 27 4 + 5 = 113, 39. Mango = Pil. Ta = is. Hence sweet = la.
Thus the wrong number is 13 , it should be 11 40. Every pair of letters is written in the reverse order.
10 Logic is 50 + 1 = 51, 51 4 = 47, 47 + 9 = 56, 56 -16 = 40,
. 40 + 25 = 65. Thus the wrong number is 42, it should be 40. DS 03
11 Logic is 3 2 + 3 = 9, 9 3 4 = 23, 23 4 + 5 = 97, 97
. 5 6 = 479 1. Mother is subset of women and doctors could be
Thus the wrong number is 99, it should be 97. mothers, other women and could be males also. So 4th
12 Logic is 2 1.5 = 3, 3 2 = 6, 6 2.5 = 15,15 3=45, 45 figure.
. 3.5 = 157.5157.5 4 = 630. Thus the wrong number is 2. All intersect each other.
157.25, it should be 157.5. 3. Sparows are subset of birds, nice are separate.
13 36 20 = 16,20 12 = 8, 12 8 = 4, 8 6 = 2, 6 5 = 1 , 4. Both are subset of a larger set.
. 5 - 4.5 = .5, The difference is halved every time. 5. All intersect each other.
Thus the wrong number is 5.5, it should be 5. 6. Furniture is a layer set which includes both.
7. All are different.
DS 02 8. Pets include dogs and cats.
9. Brinjal is vegetable, meat is separate.
1. First and last letter remain same. The others interchange
10. Larger set includes both.
their positions.
11. Human beings include both teachers and graduates.
2. Reverse the word and move each letter 1.
12. There is no intersection among the three.
3. Each letter moves +3.
13. Plum are fruit, tomatoes are different.
4. Each letter moves +3.
14. Earth has both mountains and forests.
5. Each letter moves +1, -1, .... except for L, which is 4.
15. Both belong to a larger set.
6. Letters are interchanged in each pair.
16. Both belong to a larger set.
7. The small letters are b, d, e, g, i, k, m, o, q, s.....
17. There may both white flower and clothes.
8. Last letter is U.
18. All are disjoint sets.
9. Alphabet is written in reverse: i.e. ABC.... becomes
19. Rohtak is in Haryana, while Punjab is separate.
ZYX...
20. Both engineers and doctors are people.
10. Same as above.
21. Theves are criminals while lawyers are not.
11. Letters are interchanged in each 3-letters : RAT = RTA,
etc. 22. Island is in sea, while mountain is different.
12. The alternate letter moves 1, -2. 23. Both leprosy and scurvy are diseases.
13. The letters move 2, +1,..... 24. Both are games.
25. All are disjoint sets.
Bulls Eye
www.hitbullseye.com

26. Sun is a star and moon is different. Hence 3rd option. 4. 5 3


27. All are disjoint sets.
28. Mercury and Mars are planets.
29. Both MP and MLA are elected. 10
30. Triangle is disjoint while square is join sides. 15 15
31. Both doctor and nurse are human beings.
32. Protons and electrons are included in atoms.
33. Earth is in planets, while sun is different.
34. Dog is an animal but all animals are not pets. A B
AB = 5 + 3 = 8
35. Physics and Chemistry are science.
5. He is facing North.
36. Hydrogen and Oxygen is in atmosphere.
37. Both are grams.
38. Lathe is machine, but mathe is entirely separate. 10 5
40. All are separate but may have common characteristics.
41. Pencil is stationary but jeep is different.
42. All are disjoint.
10
43. Both are vegetables.
44. All are disjoint sets.
45. All are disjoint sets. 6. He has to move a distance of 1 radius
46. Some girls are athletes and sugers. 7.
47. Room contains wall and window. 10
48. City is included in state and country. B
3
49. Copper and wire are related, but paper is not.
50. Player may be graduate and teacher. 2
A
DS 04
1. Distance AB = 3
3 AB = 3 + 2 = 5 South
8. A
7 7 30
30
20
A B
B
AB = 30 + 20 = 50 30
2. AB = 2 2 9.
40

25 25
30 30

A B
50
3.
4
B Distance from initial position = 50 40 = 10.
4 10.
5 25

15 15
10 20
B A
AB = 10 + 5 = 15.
A
AB = 20 + 25 = 45.
Bulls Eye
www.hitbullseye.com

11- 31.
5 4
14.
4
9 A
B
Couple brother
1 5 1 2
6
AB = 16 1 17 approx 4.
12. He is facing N
13. SOUTH sister son
14. Direction is North-West 3 6
15-
D
17. S5s husband is S1.
32. The mother is S5.
15 33. There are 4 males.

F
C DS 05
1. Given: S < M, M > L Z. Hence S = Z and S L are
25 both wrong.
15
2. N C = D < M. Hence N < B is true but C is not
true (C < M).
B 25 A 5 E 3. J K, L < J, L > B. Hence K L but J > B is true.
4. C > D = E < F. Hence C > F may or may not be true,
16. See above figure. but D F is false.
17. See above figure. 5. S > T, M K = T. Hence T > M or T = M.
18- 6. Z < Y, Y > X and Z = X. Hence both Z = Y and X = Y
22. 4 E
are false.
AB = 2 A B C D. Hence A D.
2 7.
4 8. D > A = B > C. Hence B > D is false but C < A is true.
A 9. P Q, Q R, R P. Hence R < Q and Q R are
both false.
B 10. M P, M = N P. Hence N P and P N are
both true.
19. See above figure. 11. P > Q < C < D. Hence P = D and P < D are both false.
20. See above figure. 12. X Y < Z, T = X. Hence T > Z and Z < T are both
21. See above figure. false.
22. See above figure. 13. M = A > K L. Hence K < M is true but L > M is not.
23. We see that rectangle is in the triangle. 14. B N, C > T = N B. Hence both N < C and C > B
24. Both square and circle intersects the rectangle. are both true.
25. Some part of the circle is outside. 15. J > C < M, C > N. Hence J N but J > N is true.
26. Square is not subset of triangle. 16. B < K < M < Z. Hence B < Z is true. II is wrong.
27. sister Rahul Raja 17. R > B, B M, Hence R > M and R < M are both false.
Sarika 18. M R, Q P = R M. Hence M > Q and M = Q
(sister) are both false.
19. M < N R. Hence R M is not true but R > M is
Amit (Son) true.
Rita Sonu
20. P Q, Q M. Hence P M and P M are both
false.
Amit & Sonu are cousins. 21. M > N T = P. Hence P N is true.
22. H > W S < M. Hence M = 1+ and M > H are both
false.
23. G < S < F, T G. Hence F > T is true but T = S is
28. Rita is niece. false.
29. Amit and Sonu 24. P Q R, T Q. Hence R > T and R > P are both
30. Maternal Uncle
Bulls Eye
www.hitbullseye.com

false.
25. M = N > B < P. Hence P = N is false but B < M is true. DS 07
1.
DS 06 Appl
1-5 There is no final step, its a continuous process. In the Ca
Dice
first step the 4th word is brought at the 1st place and last Slate
word is brought at the 4th place and this process keeps on
repeating. Ca
6-10 It is a ease of simple arrangement.
Look at the last step. From last step it is obvious that From the diagram we see That some apples are
numbers and words get arranged alternately. definitely dice, and in some cases some dice are cars
Also, the numbers are arranged in the following way: and in other cases No car is dice. So either II or IV may
Largest, Smallest, Second largest, Second smallest and also follow.
so on. 2.
Chai
Also, the words are arranged in the following ways: Chai Cloud
A . Z., B. Y.. and so on.
Film
From input to step I, a number gest arranged first and the Rat
remaining elements are pushed rightward. From step I to Rat
step II, a word gest arranged and the remaining elements
are pushed rightward. The process continues and all the
elements get arranged. If an element is found already Cloud
Chai Chair
arranged, another elements gets arranged.
6. Step III: 91 go 28 mock pet 43 lead 37 Chai
Step IV: 91 go 28 pet mock 43 lead 37 From the possible cases seen from the diagram, Some
Step V: 91 go 28 pet 43 mock lead 37 clouds are definitely rats in every case. All others are
Step VI: 91 go 28 pet 43 mock lead 37 true only for some specific cases.
Step VII: 91 go 28 pet 43 lead 37 mock 3.
Hence, step VII is the last step. Sho
7. Step II: 52 at deep follow 4116 road 32 Candl
Step III: 52 at 14 deep follow 41 road 32
Step IV: 52 at 16 road deep follow 41 32 Table Tabl
Step V: 52 at 16 road 41 deep follow 32
8. It is a case of arrangement. Previous steps can't be Bell
obtained with certainty. Sho
9. Step II: 76 from 48 super itself 56 18 went
Step III: 76 from 18 48 super itself 56 went From the diagram we can clearly conclude that all of
Step IV: 76 from 18 went 48 super itself 56 the given conclusions are true for only particular cases.
Step 37: 76 from 18 went 56 48 super itself Hence None follows So option
Step VI: 76 from 18 went 56 itself 48 super 4.
Step VI: is the last step. Thus, four more steps are Clips
required to get the arrangement. Gates
10. Input: thirty days from now 32 56 87 24 Fruits
Step I: 87 thirty days from now 32 56 24
Flowers
Step II: 87 days thirty from now 32 56 24
Step III: 87 days 24 thirty from now 32 56
11- The order of words changes from one batch to the next Clearly from the diagram only I is true.
16 as follows: the first word becomes the second the second 5.
becomes the last the third becomes the first the fourth Pain
remains the same the fifth becomes the sixth the sixth
Ice Ring
becomes the third and the last becomes the fifth. For Gold
convenience, we plot the movement of each word in
each step by the numbers assigned to them in the first Pain
batch:
Gold Gold
dig-1, morc-2, artd-3, you-4, will-5, find-6 and water-7.
1st batch: 9 am to 10 am 1 2 3 4 5 6 7 (4pm to 5 pm)
2nd batch: 10 am to 11 am 3 1 6 4 7 5 2 (5 pm to 6pm) From the diagram we can clearly conclude that all of
3rd batch: 11 am to 12 noon 6 3 5 4 2 7 1 (5 pm to 6 the given conclusions are true for only particular cases.
pm) Hence None follows So option 5.
4th batch: 12 noon to 1 pm 5 6 7 4 1 2 3 (7 pm to 8 pm)
5th batch: 1 pm to 2 pm 7 5 2 4 3 1 6 (8 pm to 9 pm)
6th batch: 2 pm to 3 pm 2 7 14 6 3 5 (9 pm to 10pm)
Bulls Eye
www.hitbullseye.com

6. 11.
Bats
Clocks Cloud Cloud
Goat
Keys s

Lock Bird Cars


Clocks Cars

Clocks
We can observe from the diagram that Either Some
So, it is clear that Some Bats are Locks. Cars are Birds or No Bird is a Car has to be true. Thus
7. 2nd option.
12.
Tubes Bananas
Bottles Cups
Pots
Potatoes
Mangoes
Grapes
From the figure it is clear that Some Bottles are Tubes,
Some Pots are Bottles and Some Tubes are Cups.
8.
Books It is clear from the figure that no conclusion follows.
Purses 13.
Flies
Paper
Bag
s
Purses Cats Rats Ants Flies

Some Books are Papers and Some Books are Purses are Ants
the only two conclusions which follow.
9.
It can be observed from the diagram that no Conclusion
follows. But the conclusion I & IV form a
Pens Box
Pens Box complimentry form. Thus 4th option.
14.
Dusters Pens
Cars Jeeps

Pens
It is clear from the diagram that none of conclusions
Chalk
follow. Boards
10.

Only Some Dusters are Boards is correct conclusion.


Rats Cats Dogs 15.
Boxes
Board

Dogs Cow Bags Books


Board

It is clear from the diagram that Some Cats are Rats.


Some Bags are not Boards and either of Some Bags are
not Boxes or All Bags are Boxes follow as per the
given diagram.
Bulls Eye
www.hitbullseye.com

The negation is ~p and ~q.


DS 08 10. The statement is of the form: either p or q.
1. q The negation is ~p and ~q.
Mohan would be selected in the company, 11. The statement is of the form : If p, then q.
p The negation is p and ~q.
If he has an excellent academic record. 12. p
If It is a Thursday, then I will go to cinema
The implications of the statement are r
(i) p => q and (ii) ~ q => p or I will sleep
It can be ca or bd.
2. q The implications of the above statements are
Only if Suman has good knowledge of Maths, (i) p and ~q => r
q (iii) ~q and ~ r => ~p.
she would be become a Mathematician. Choice (2) is implication (ii).
13. p
The implications of the above statement are If The number of retail outlets is increased,
(i) q => p and (ii) ~p => -q q
It can be bd or ac. then The prices will decrease
3. p r
Unless the Government seals the borders, The consumption will increase
q = unless p
illegal migration will not stop. The change happens I
q
The implications of the above statement will The problem will not be solved
(i) ~ p => q and (2) ~q => p.
It can be cd or ba. The statement is of the form,
4. p If p then q or r.
Whenever Aman receives a message from Aarti The implications are
q (i) p => q or r
he seems to be on top of the world. (ii) ~q and -r => ~p.
(iii) p and ~q => r
The implications of the above statement are (iv) p and -r => q.
(1) p => q and (2) ~q => ~p Choice (4) is according to (ii).
It can be db or ca. 14. p
5. p Unless You return soon,
If The Inflation is low,
You will be affected by sunstroke
the GDP will touch a record high r
and The health will deteriorate
The statement is of the form: if p, then q. The statement is of the form, Unless p, then, q and r
The implications are The implication are
(i) p => q (i) ~p => q and r
It can be ac or db. (ii) ~q or ~ r => p.
6. p Choice (1) is according to (i).
Whenever Viru and Basanti go for movie, 15. P
q If the faculty is in the class
Jai followed them. q
The negation of the above statement is p and ~q. then the students will either study
7. q r
Ajeet will attend the function, Or keep quiet.

Only if his father allows his to go by car Statement: If p, then q or r.


The negation of the above statement is ~p and q. Conclusions:
8. p (i) p => q or r
Unless the meeting takes place, (iii) p & -q => r
q Conclusions:
the problem will not be solved. (i) If the faculty is in the class, then either the students
will study or they will keep quiet, (ii) The students are
The statement is of the form: Unless p, then q. not studying and they are not keeping quiet, implies
Negation for the above statement is ~p and ~q. that the faculty is not in the class.
9. The statement is of the form: Unless p, then q. (iii) The faculty is in the class and the students are not
Bulls Eye
www.hitbullseye.com

studying, implies that they are keeping quiet.


(iv) The faculty is in the class and the students are not DS 09
quiet, implies that they are studying. 1. (T True; F False)
Among the conclusions, (ii) is available as the third (i) Assuming that Sameer is the Truth-teller, we get the
choice. following arrangement;
16. p
Whenever an accident occurs Statement Statement
q I II
either Sameer T T Truth Winner
r teller
or death takes place. Sameep F F Liar 2nd
runner-
Statement: Whenever p, q or r. up
Conclusions: Sumer F F Alt. 1st
(I) p => q or r runner-
(iii) p & ~q => r up
Conclusions:
(i) An accident occurred, so an injury or death takes (ii) Assuming Sameep is the Truth-teller, we get the
place, following arrangement
(ii) An injury did not take place and a death did not Statement Statement
take place, means that an accident did not occur, I II
(iii) An accident occurred and a injury did not take Sameer F T 2nd runner-
place, means a death takes place, up
(iv) An accident occurred but a death did not take
Sameep T T Winner
place, means that a injury takes place.
Sumer F F 1st runner-
17. p
up
If it is a working day, then I will be busy
r
(iii) Assuming Sumer is the Truth-teller, we get the
and be restless all day.
following arrangement.
Statement: If p, then q and r. Statement Statement
Conclusions: I II
(i) p => q and r (ii) ~q or ~r => ~p Sameer F T Alt. 1st
(iii) ~ q & r => ~p (iv) q & ~r => ~p runner-
(v) ~q & ~r => ~p up
Conclusions: Sameep F F Liar 2nd
(i) It is a working day, means that I will be busy and runner-
restless all day. up
(ii) I was not busy or was not restless all day, means Sumer T T Truth- Winner
that it was not a working day, teller
(iii) I was not busy and I was restless all day, means
that it was not a working day, Hence, either Sameer or Sumer can be the 1st runner-up,
(iv) I was busy and I was not restless all day, means but not Sameep.
that it was not a working day. 2. If A always speaks the truth, then B Is the Good', A is
(v) I was not busy and was not restless all day, implies the Ugly', hence C is 'the Bad'; which also means that B
that it was not a working day. is also the truth-teller. Hence, neither A nor B is the
18. p truth-teller.
Whenever the pollution is on rise , Therefore, C must be the truth-teller, which gives us the
q following arrangement:
the vehicles will be stopped and A the Good;
r C the Bad;
their emission levels will be checked, B the Ugly
3. Each among John, Johny and Janardan is an Alternator.
Statement: Whenever p, then q and r. Let John's first statement be true and second be false;
Implications:
(i) p => q and r (ii) ~q or ~r => ~p I II Medal
~ p is 'the pollution did not rise', ~q is 'the vehicle will John T F Gold
not be stopped' and ~r is 'the emission levels will not
be checked'. Johny F F Silver
Implication (ii) is represented in the first choice and (i) Janardan T F Bronze
is represented in the third choice.
Bulls Eye
www.hitbullseye.com

Hence, Johny is not an Alternator, which means our So, we have the following:
initial assumption was false. Case (i)
Let John's first statement be false and second be true: I II III
I II Medal P T T T
John F T Silver Q F F F
Johny T F Bronze R F T F
Janardan F T Gold Case (ii)
I II III
Hence, Janardan Gold medal P F F F
John Silver medal Q T T T
Johny Bronze medal R T F F
4- (i) Assuming Ramu always speaks the truth:
6. I II City In case (i), R becomes an alternator which violates the
Ramu T T Delhi given condition.
Raman F T Mumbai Hence case (ii) in the correct arrangement and the south
Rajan F F Chennai bound bus goes to the airport.
This arrangement works out for answering the first two 10. R replies in the manner: Truth - lie - lie
questions. 11. Only I statement is true.
(ii) Assuming Raman always speaks the truth: 12. (20 minutes to wait for the bus) + (30 minutes of travel
I II City time) = 50 minutes to reach the airport.
Ramu F F Mumbai 13- Each person alternates between truth and lie, then for
Raman T T Delhi 15. three statements given by any person, the nature of the
Rajan T T Chennai statements could be:
True, False, True
(iiii) Assuming Rajan always speaks the truth: or
I II City False, True, False
Ramu F F Mumbai (i) Assume that the three statements given by A are in
Raman T T Delhi the order True, False, True. Then we get the following
Rajan T T Chennai arrangement:
I II III Organization
Arrangement (ii) and (iii) help in answering the third A T F T IBM
question. B T F T Infosys
4. Rajan must be from Chennai. C F F F Microsoft
5. Raman belongs to Mumbai. D Oracle
6. Rajan is not from Chennai is false.
7- It is known that D plays Tennis, which means that the Here, all the three statements given by C become false,
9. 2nd statement made by B must be true, whereas the 2nd hence, we'll try the second combination.
statement made by D must be false. As A says that he (ii) Assume that the three statements given by A are in
and C both play cricket, A cannot be the person who the order False, True, False.
always speaks the truth, as each person plays exactly
one game and exactly one game is played by each Then we get the following arrangement:
person. This means only C can always speak the truth, I II III Organization
as there must be at least one person who always speaks A F T F (x IBM) Microsoft
the truth. B F T F (x Infosys) Oracle
We take this as the basis and get the arrangement as C F T F (x Microsoft) Infosys
given below: D T F T IBM
I II Game played
A F T Table-tennis Hence, here we get a definite arrangement.
13. A works with Microsoft
B F T Cricket 14. B works with Oracle
C T T Football 15. As can be observed from the arrangement made above,
D T F Tennis it is choice (D) which is false.
B plays Cricket.
DS 10
7.
8. C always speaks the truth.
9. D and B alternate between truth and lie. 1- Let x be the candles lighted by Michael on the first
10- If P always speaks the truth, then Q always lies R. 3. day. And it is given that Christ accepts none of them
12. If P always lies R then Q always speaks the truth. on the first day.
In either case, R cannot be speaking all truths or all lies. The difference between the number of candles
R cannot alternate between truth and lies either. lighted and accepted = x- 0= x
Bulls Eye
www.hitbullseye.com

The candles offered is x more than the candles in 2012 as compared to 2011, so the share in 2011 =
accepted. 343.2
On the second day, Christ accepts all the candles that Rs 457.6 cr
were lighted.
0.75
On the first day except three i.e., x 3. The number 457.6
Hence the required %age = 100 34.67%.
of candles lighted by Michael on the second day => x - 1320
3 + x = 2x 3. On the third day Christ accepts (2x - 3) 8- Let us consider the following Venn diagram
- 3 = 2x - 6 candles. But it is given to be 18. 2x - 6 = 11. GT = 600
18 x = 12
The flowers offered and taken is as follows: Tennis SD
Day 1 2 3 4
Lighted 12 21 30 39 d b
a
Accepted 0 9 18 27
The candles lighted are in A.P with a common g
difference of 9 and also the number of candles f e
accepted is in A.P with a common difference of 9.
Candles lighted = 3 + 9n.
c
Candles accepted = 9 (n - 1), where n is the number of
that day.
1. The sum of first n terms in an arithmetic progression Male n
n
. [2a + (n - 1)d]
2 a + b + c + d + e + f + g + n = 600 ..(1)
40 In the above Venn diagram, the number inside each
Here, n = 40, a = 0 and d = 9 Sum= [(40-1)9] = circle is the number of students who belongs to that
2 group [(d + b + g + e) represents students of SD
20 [39 9] = 7020. College and the number of students outside each circle
2. 3 + 9n = 435 => 9n = 432 => n = 48. is the number of students who do not belong to that
3. The sum of the first n terms in an arithmetic group [(a + f + c + n) represents students of DAV
n College).
progression = [2a + (n-1)d]
2 Given e = 30 .(2)
a + d + f + g = (600 - 285) = 315 (3)
25 c = 90 .(4)
Here, n = 25, a = 12, d = 9 => Sum = [24 + 24
2 d + g = 150 ..(5)
9] = 25 120 = 3000. d + f = 150 (6)
4. Given that percentage growth from 2011 to 2012 = c + f = 150 .(7)
30%. Percentage growth from 2010 to 2011 = 10%. a + b = 180 ..(8)
1716 from (4) and (7),
Sales in 2012 = 1716 cr Sales in 2010 = = f = 150 90 = 60 ..(9)
1 .3 1 .1 from (9) and (6)
Rs. 1200 crore. d = 150 60 = 90 ..(10)
5. Sales in Q1 = Sales in Q3 from (10) and (5),
Sales in Q2 = 1.5 Sales in Q1 g = 150 90 = 60 From (3)
Sales in Q4 = 1.4 sales of Q1 a = 315 (d + f + g) = 315 (90+60+60) = 105
Let sales in Q1 be X from (8) b = 180 105 = 75
Sales in Q2, Q3 and Q4 are 1.5X, X and 1.4X from the above results
respectively. Now let the profit in Q4 by Y. n = 600 (315+ 75 + 45 + 90) = 75
Profit in Q1, Q2 and Q3 and 0.6Y, 0.7Y and 0.4Y. The final Venn diagram is as follows GT = 600
Profit as a percentage of sales is
0.6Y 0.7Y 0.4Y .Y Tennis SD
, , and in Q1, Q2, Q3 and Q4
X 1 .5 X X 1.4 X
respectively. So it is the highest in Q4. 10 90 75
6. Sales in Q2 =
1.5 1.5 60
1716 1716 Rs 525.3 cr 60 45
11.5 11.4 4.9
7. In 2012, the sales = 1716 cr
Sales in 2011 = 1320 cr 90
Share of beauty equipment in 2012 = 20% of 1716 cr =
343.2 cr
As the share of beauty equipment has been decreased Male n = 75
Bulls Eye
www.hitbullseye.com

8. 75 female students who study in SD College play least and second least marks in any order.
cricket. Adding all the marks we get 6 (A + B + C + D + E) =
9. 105 female students who study in DAV College play 250 + 254 + 256 + 264 + 266 + 270 + 270 + 272 + 276
Tennis. + 286 = 2664
10. 105 + 90 + 75 + 75 = 345 students are females. A + B + C + D + E = 444
11. 60 male students who study in DAV College play For ease of calculation, let us assume that B has more
Tennis. marks than A.
12 12 lottery tickets can be distributed among 4 persons A + B + C = 250
- such that no two persons get the same number of D + E = 194
15. lottery tickets in 2 ways: Either 1, 2, 3 and 6 or 1, 2, 4 C + D + E = 286
and 5. C = 92
Consider the distribution 1, 2, 4 and 5: Q can have As we have assumed B > A, so the second highest
lottery tickets numbered 2, 3, 5, 7 and 11 only. As both group has to be B + D + E = 276
P and R have at least two lottery tickets, S must have B = 82
only one lottery ticket. It must be numbered 9 as R A = 250 (82 + 92) = 76
cannot have a square of a prime number and P cannot The second lowest group has to be A + B + D
have an odd one. Also, P should have 4. Since R A + B + D = 254
cannot have square of a prime number. D = 254 (76 + 82) = 96.
S P R Q E = 194 96 = 98
9 4, 10 1, 6, 8, 12 2, 3, 5, 7, 11 D1 The marks are A 76, B 82, C 92, D 96 and E
Now, consider the distribution 1, 2, 3 and 6. Then they 98. (A and B could also be 82 and 76 respectively)
will have 16. The total marks of all the students = 76 + 82 + 92 + 96
S P R Q + 98 = 444. Hence the average marks = 444/5 = 88.8
9 8, 10 1, 6, 12 2, 3, 5, 7, 11, 4 D2 17. C has the third highest marks.
Also the sum of numbers on all the lottery tickets of 18. The maximum marks are 98.
each person is calculated and tabulated below. 19. The difference in the marks of B and D could be 14 or
D1 D2 20.
S 9 9 20. The difference between the marks is maximum for A
(or B) and E = 98 76 = 22.
P 14 18
R 27 19 DS 11
Q 28 32 1- Let us assume 'a' to be the number of matches in which
12. If the lottery ticket numbered 14 is to be inserted in the 5. 'Tanu' stood as the winner in 2010 and 'b' to be the
distribution 1, 2, 4 and 5, it should be given to P as the number of matches in which 'Tanu' stood as the runner
other three conditions should not be violated but the up in 2010.
sum of R is not more than Q. So it is not possible. In the matches in which "Tanu reached the finals he
So, the distribution will be for 1, 2, 3 and 6 only must be the winner or the runner up. a + b = 160 and
=> S P R Q 4a + 2b = 480 => a = 80, b = 80.
9 8, 10 1,6,12,14 2,3,5,7, 11,4 Since the number of matches in which 'Tanu' stood as
The sum of all numbers on the lottery tickets of P = 8 the winner is 80 in 2010, the number of matches in
+ 10 = 18. which 'Tanu' stood as the runner up in 2011 is also 80.
13. The minimum possible difference in D1 = 1 (28 - 27) Total number of points earned by 'Tanu' in 2011 is '640'.
and D2 = 1 (19-18). => Out of 640, he earned '160' (80 x 2) points as the
14. Choice (2) refers to D1, where R = 27. runner up.
Choice (3) refers to D2, where R = 19 The remaining 480 points could have been earned by
Choice (1) refers to D1, where 2P = Q emerging as the winner in 120 matches.
So we get unique solution using any one of the above Like wise, we can find the number of matches in which
choices. Tanu' stood as the winner and the runner up.
15. The maximum possible difference in D1 = 19 (28 - 9) Year
2009 2010 2011 2012 Total
and in D2 = 23 (32 - 9). Tanu
16- It is given that the difference between the marks of A, Winner 80 80 120 120 400
20. B, D and C, D, E is 32. From the given values we Runner
48 80 80 120 328
know A + B + D = 254 and C + D + E = 286 is the Up
only option. As 254 is the second lowest among all the Total 128 160 200 240 728
marks given, it is obtained when the two smallest and
the second largest marks are added. As 286 is the It is given that the number of times 'Tanu' reached finals
maximum among all the marks so it is obtained when is twice that of the times 'Manu' stood as runner up in
the three highest marks are added. As D is common to the same year.
both the groups, D has the second highest marks and C => In 2009, Manu stood as runner up = 128/2 =64
has the third highest marks. A and B would have the In 2009, Manu was the winner = 94 times.
Bulls Eye
www.hitbullseye.com

Like wise tabulate the information C 96 4


D 90 4
Year 2009 2010 2011 2012 Total E 84 2
Manu F 76 14
Winner 94 116 138 202 550 11. As 6 employees in department A failed in the year 2012-
Runner 64 80 100 120 364 13 and as there were 94 employees in the company in
Up 2013-14, a total of 94 - 6 = 88 employees joined the
Total 158 196 238 322 914 company in 2013-14.
12. Exactly four employees failed in department B, C and
1. No of times Tanu reached finals = No. of times he stood D.
as winner + No. of times he stood as runner up = 128 + 13. The total number of employees who failed in the year
160 = 288 2012-13 was 6 + 4+4 + 4 + 2 + 14 = 34
2. Manu as runner up in 2011 and 2012 = 100+ 120 = 220 14. The pass percentage was the highest in department E.
3. Total number of finals reached by Tanu in 2011 and 15. Item D which has 40% of Sodium, costs Rs.
Manu in 2010 = 200 + 196= 396 150/serving.
4. Total number of times Tanu emerged as winner = 80 + But Item A only costs Rs. 100/litre and by mixing Items
80 + 120 + 120 = 400 A and D we can reduce the cost/serving to less than 150.
5. Difference in the number of finals reached by Tanu and To have 25% of Sodium, Items A and D can be mixed in
Manu: = (914 - 728) = 186 a ratio of 3:1.
6. In the tournament there are 15 matches. Every match 3 100 150
can be an upset. the cost/serving = 112.5
4
7. G is the 7th seeded player in the first round he plays with
16. The item which has the least cost/serving, i.e. Item A,
a lower seeded player.
have more than 20% of each of minerals and salts S, P
Hence, there will not be any upset for him in the first
and C. To make the quantity of Magnesium 20%, we
round. But in quarter final he plays with B. Hence, to
can combine items A & D in the ratio 3 : 2, to get a
reach semi final there should be at least one upset.
solution with cost/serving as
In semifinal G faces C. There should be at least one
upset in semi final. Similarly in final there should be an 100 3 150 2
Rs.120
upset in the final (As G will have to play with A as there 5
are no upsets till the matches in the finals in which G 17. To get an item containing 25% of Magnesium, we have
did not play) . In total there should be 3 upsets in the to combine Items A & D in the ratio 2 : 3
tournament. 18. The maximum quantity of Potassium occurs, when we
8. In the final one among A, P, H, I, D, M, E and L and combine the items A & B. As the cost/serving of the
one among B, O, C, N, F, K, G and J will play. Except resultant item should not exceed Rs. 175, we have to
B, J and other pairs are possible set. mix items A & B in the ratio 1 : 1 to get 35% of
9. In Semi final, one among D, M, E and L will play with Potassium.
A and one among B, O, G and J will play with K Except 19. The net score if all his attempts are correct = 28 3 - 22
choice 4, others are possible. 1 = 62
10 If L is the winner, then none among A, P, H, I, D, E and 20. As his net score was 55, the only possibility is 28
M can be the runner up as they must be defeated by L to correct and 7 mistakes.
reach the final. Hence, G can be the runner up. 21. As Cherry had 25 correct answers, his minimum net
11- It is given that 76 employees got transfer to a new place score would be 25 3 - 25 2 = 50. As Farukh had the
14. in the year 2012-13. As there were 90 employees in highest net score, his score was above 65. Since he had
department F in that year, it means that 14 employees 32 attempts, his maximum score would be 96 and for
failed in that department. Now these 14 employees each mistake his score would come down by 5 (3 marks
would have worked in the same department in 2013-14. of the correct answer and 2 penalty for the wrong
In 2013-14, as there were 98 employees in the answer). If he has 2 mistakes, he would have a net score
department it means that 98 - 14 = 84 employees who of 68, which is not given in any of the options. Thus
were in department E in 2012-13 passed and got fourth option is the answer.
promoted to department F. 22. As Guru had 21 correct attempts, he would have a net
Two employees in department E in 2012-13 failed and score of 21 3 = 63. His maximum score would be
as there were 92 employees in department E in 2013-14, when he has attempted only 21 questions in which case
it means 92 - 2 = 90 employees who were in department his score would be 21 3 - 29 1 = 34. His minimum
D in 2012-13 passed and got promoted. Similarly we score would be if he had attempted all the 50 questions
can find the corresponding values for each department. and in this case his score would be 21 3 - 29 2 =5.
His net score would be in the range 5 to 34. Thus fourth
Department Employees Employees option.
passed failed 23. Product S can be produced only after producing Q and T
A 78 6 i.e., P, Q, R and T.
B 68 4 Required time = 4 + 6 + 4 + 4 = 18 minutes.
Bulls Eye
www.hitbullseye.com

24. Product U can be produced only after products S and W


are produced. Product S can be produced in 18 minutes
from the start and product W in 24 minutes from the
start.
Product U can be produced in 24 + 10 = 34 minutes
from the start.
25. The products which can be produced in ten minutes
from the start are P, Q, R.
26. Product U can be produced only after P, Q, R, S, T and
W are produced.
27. It can be seen that the last intermediate product to be
produced would be Y and it would be produced in 4
minutes after product U is produced i.e. 34 + 4 = 38
minutes from the start.
28- From the given information, we can arrive at two
31. possibilities. As Chander and Lalita scored the same
marks in one of the subjects, they can be
(i) 1111,1113(English),
(ii) 1112, 1114 (Math)
Malti can be 1111 or 1112 and Dipankar can be 1113 or
1114.
. Bhavesh is 1115.
Roll No. Possibility 1 Possibility 2
1111 Chander / Lalita Malti
1112 Malti Chander/ Lalita
1113 Chander / Lalita Dipankar
1114 Dipankar Chander / Lalita
1115 Bhavesh Bhavesh
28. Number is 1114. From the above possibilities,
possibility 1 is correct and hence Malti is 1112 and did
not score the least mark in the Math, i.e., if I is true then
II is false.
29. If Malti gets the same marks in English and Hindi, then
possibility 2 is true. Lalita's roll number could be either
1112 or 1114. For either of the roll numbers, least marks
are in the English i.e. statements I is true. Dipankar's roll
number would be 1113 and his least marks are in the
Math section. Statement II is false.
30. By observing the data given and the possibilities,
possibility 1 is true, and hence Lalita got the least score.
31. From the above possibilities we can only conclude that
Bhavesh's roll number is 1115.
Bulls Eye
www.hitbullseye.com

Analytical & Logical Reasoning (Previous SNAP Questions) EXPLANATIONS


1. Let us check option 2.As per G and B, people are rewarded and people change their lifestyle.
According to statement F, that means that the temperature level doesnt rise. Therefore, According to statement D, water level
doesnt rise.
This is consistent with statement H. Hence, Answer is option 2.
2. Let us check option 3. Going through each statement one by one we get,
If Vina dances, Kumar sings, Statement G says that Vina dances. Therefore Kumar sings. According to B, if Kumar sings,
audience dances.
According to C, If audience dances, concert will be successful.
This is consistent with statement H, that says that the concert will be successful.
3. A = 26, E = 22, I = 18,O = 12, U = 6, Ans = 84.
4. 5-12-18-23 = void
5. Series of squares = 1, 4, 16, 25, and as B =25, Hence answer is B
6. Since three surfaces have on colour painted on each of them. Therefore the no. of cubes with no sides painted = 6 6 6 =
216.
7. Each of the painted face with have 6 6 = 36 Cubes with exactly one face painted. The total al no of such cubes = 3 6 6 =
108
8. Cubes with at the most two faces painted
= Total Cubes Cubes with exactly 3 face painted = 343 1 = 342
9.-10.

Looking at the diagram


10. Looking at the diagram
11. Based on the inputs, the following tables gives the input output
X Y
Initial 3 2
Step I 6 3
Step II 18 4
Step III 72 5
Step IV 360 6

Stop (x = 360) (N = 100). Obviously four Steps are reqd.


12. The final value of x = 360
13. The final value of y = 6
14.
X Y
Initial 3 2
Step I 6 3
Step II 18 4
Step III 72 5
Step IV 360 6
Step V 2160 7
The final value of X = 2160
15.
X Y
Initial 2 3
Step I 6 4
Step II 24 5
Step III 120 6
Bulls Eye
www.hitbullseye.com

Step IV 720 7
Min Value of N = x = 720
16. Based on the inputs the following is the marking
Rank 1st 2nd 3rd 4th 5th 6th 7th
Name Raman Anil Sunil Tony
Obviously Deepak is ranked Third or Fourth Can Be True.
17. The following is the marking
Rank 1st 2nd 3rd 4th 5th 6th 7th
Name Raman Tony
Also Rank wise: Vicky > Priya > Ankit
Implies Vicky Should not be ranked lower than fourth.
18. Based on the inputs : Ankit is ranked first violates the initial condition. Therefore none of these is the only option.
19. If Sunil is ranked 2nd it implies
Raman is ranked Ist and Tony 7th
Also Rankwise : Vicky > Priya > Ankit
Therefore Deepak gets more than Vicky can be true.
20. If Vicky is ranked 5th
Priya is ranked 6th & Ankit 7th
Sunil scores the highest.
21. If 35 % people favour party Z vs 40% to Y, It is unlikely that party Z will win the elections.
22.
23. The relation between the given pair -Birth: Dirge(which means a mournful song sung at time of death),
So Marriage: Alimony( give after divorce)
24.
25.
26. Nilu must be a great pilot has been concluded on the basis that Nilu never received a violation.So there is an assumption
that anybody who does not receive violation from FAA must be a great pilot.
27. The last line is the conclusionwhich says that the existence of the death penalty does not. This conclusion has
been drawn from a particular example of TOM. Hence option C is the correct answer.
28. If we divide the sum of numbers in each row by 3 we get middle number of that row. On this pattern the answer should be 6.
29. The supposition in c involves reducing by one the number of restrictions on the advertising of legal services. Any such
reduction will, if the stated correlation exists, be accompanied by an increase in the number of lawyers advertising their
services, as C predicts. Therefore, C is the best answer. A does not follow from the stated information since it is still possible
that no lawyers would raise their fees. B does not follow from the stated information since it is still
possible that there would be no increase in the number of consumers using legal services. D does not follow the stated
information since it is still possible that none of the lawyers who do not advertise would decide to lower their prices.(
Question Taken from Official Guide of GMAT)
30. If D is true, the lawyers who begin advertising when the restriction is removed might all be among those who do not lower
their fees on beginning to advertise, in which case no decrease in consumer legal costs will occur. Therefore, D weakens the
argument and is the best answer. ( Question taken from Official guide of GMAT)
31.
A B C D
110 10 10 10
-10 +10 +20 +30
-20 +5 +5 -5
-30 +10
-10
-5
(35) (35) (35) (35)
32. 102 + 9 + 90 = 201
Hence 203 + 7 +70 = 280
33. In first alphabet, there is increase of 2 letters, So next number will start with Z. In the last Alphabet there is increment of 5
alphabets, So after N next letter will be S. So answer is c option.
34. Correct sequence is Brain, Ricky, Sachin. According to this pattern, One statement of everyone is right and another is wrong.
So answer is a option.
35. T/C = 6/4, T/T+C = 6/2. N(AUB)= 6x + 4x - 2x = 8x. Number of people who drink both = those drink neither = 2x. 10x =
60,x = 6. So answer is 12, (b) option.
36. For 2 intervals, there are 6seconds, So 8 intervals = 6/28= 24. So answer is (a) option.
37.
Bulls Eye
www.hitbullseye.com

Starting time Completing time


1 E-3 E-2
2 E-1 E-3

3 E-2 E-1
Refer to the above table, Answer is c option.
38. Refer to the above table, Answer is (a) option.
39. Sequence in increasing order is A,R,Q,C,B,P.
P is brightest among all. Hence (b) option.
40. R is dullest amongst the three students from Elite School. Hence, C option.
41. The number of students cannot be decided. So answer is (d) option.
42. D
C G

B F

E
A
H
Arjit is sitting to the immediate right of Himadri. Hence (a) option.
43. Refer the above table, Frederick is sitting to the opposite of Biplap. So answer is c option.
44. Refer the above table, Biplap is sitting to the immediate right of Chintan, So b option.
45.
Rocks

Balloon Tables
s

46. A C - E, C is sitting in the middle of the bench. So (b) option.


47. A and E are sitting at the extreme ends of the bench. So answer is (a) option.
48. 29800 24600
Required percentage = 21.14%
24600
49. 28000 100
Required percentage = 19.05%
(29800 28000 29100 30060 30000)
50. The common difference between the numbers given in first row is 2while the difference between the number given in the
second row forms an A.P with a common difference of 8.Difference between the consecutive numbers in the second row are
12,20,28 and 36
51. By options .Only d option satisfies all the conditions
52.
Idealists

German
Philosophers Marx

According to the given statements only d option is satisfied.


53. Let us assume the length of both candles to be 12 cm. So in 1 hour, the thick candle will be 2 cm shorter while thin candle
will be 3 cm shorter. Now go by option. For option (b), after 3hrs. length of thick candle remaining will be 12cm - 23 =
6cm, and length of thin candle remaining = 12cm - 3cm 3 = 3cm. Thus, it follows the given condition. Hence the answer.
54. 2x
According to given condition, if is the original fraction than
3x
2x 6 2 2x 2x 6 4
18x-54=12x x = 9 Numerator to original fraction = 2x=18
3x 3 3x 3x 9
55. By Checking options only (b) Satisfies the given conditions.
56. Third piece will make a pair of some colour.
Bulls Eye
www.hitbullseye.com

57. Let x be the number of people who know both Java and dot net
Java Dot net

450-x x 750-x

So 450 - x + x +750 - x = 1000 x = 200


58. Choices (b) is directly supported by the fact given in the second statement of the question which says, Athletes who do not
have well balance diets are bad athletes.
59. Option (b) Satisfies the Symmetry.
60. 17 x 1 1 1 1
17 = , 17 (1 - x) = 17x, 17 - 17x = 17x. So 34x = 17. Hence x = . So Value of 2x x = (2 ) =
1 x 2 2 2 2
61. If D moves in anti-clockwise A also moves in the same B also moves in the same
If D has to move in the given direction C has to move in clockwise E anti-clockwise F clockwise G anti-
clockwise B clockwise. The above two conditions B is moving in anti-clockwise as well as clockwise
simultaneously which is not possible.
62. Sun shine brightly is written as ba lo sul
Houses are brightly lit is written as kado udo ari ba
Light come from sun is written as dapi kup lo nro
Therefore, brightly is written as ba and sun is written as lo
63. As when hot waves cool it moves towards ground to become lighter. So 4th option.

64. No. of males = 5, No. of females = 4. Now if any males wants to check his number of brothers which are at least
4 and sisters are at least 3.
If any females wants to check No. of sisters which are at least 3 and brother are at least 4.
65.

Doctor
Fool
Joshi

Hence 2nd option.


66. Drawing a diagram .

W E

S
The direction is north west
67. By visualizing the cuboids we see that there are 2 cubes on the top and bottom of the cubes having one colour on
them. Therefore, 2 + 2 = 4. Similarly the side faces also have 2 single colour cubes on them. Hence 2 + 2 = 4.
The front and the back face have one single coloured cube therefore, 1 + 1 = 2. Hence total single colour cubes 4
+ 4 + 2 = 10.
68. By visualizing
69. As the cubes having two colours will be 6 on top and bottom face, and 4 on the sides faces(these include the
cubes having common faces with the cubes having 2 colours on the top, bottom, side faces) . Therefore,
Bulls Eye
www.hitbullseye.com

totl=6+6+4=16
70. A B C
1 6 3 8
2 4 7 1
3 2 9 5
Possible combination satisfies the second option.
71. Observing the pattern 4 8 = 32, 32 9 = 288, 288 10 = 2880
72. Let us take 60 lemons of first type and 60 lemons of 2nd type.
By selling first type at the rate of 2 for 0.50 Rs.
By selling second type at the rate of 3 for 0.50 Rs.
We will get 15 Rs. From 1st and 10 Rs. From 2nd.
By Mixing them and selling at the Rate of 5 for 1 Rs. We will get overall 24 Rs. So 1 Rs. Difference will create
a loss of Rs. 3. So for 3.5 Rs. Difference loss will be of 10.50 Rs. which is the first option.
73. Never change
74. By Reading the given assumption we can implicit I only.
75. The words are IMMODEST and INDECENT, thus the remaining letters are MS and DN respectively. Thus 4th
option.
76.
77.
Common Common
Abdul 1y
Mala 1y 1y 1y
Chetan 1y 1y 2y
So, total yellow will be equal to 1y + 1y + 3y = 5y
So, 2nd option.
78. As given in question O, N are males and M is female, so only statement 4 is correct.
79. A B

D
C
So final position will be as B and C are diagonally opposite which is 4th option.
80. Condition 1 W > P > N, Condition 2 W = P if H S condition
Going by options
Option 1. S W H P N, Condition 2 false
Option 2. W S P H N Condition 2 false
Option 4. P N H W S Condition 1 false
81.
16

20 10

So, answer will be 2nd option as 20.


Bulls Eye
www.hitbullseye.com

82. Going by the pattern (5 + 4) 8 = 72.


83. By inserting first row in ground and top in ground we will get a movement of half circle equal to 1800. So 3rd
option.
84. According to given conditions A, D are unmarried females and B, E are males. Hence C is female and is wife of
E. Again B does not play Chess or tennis, so he plays badminton. So E will play chess (Condition IV in ques.).
Again by condition III C will play tennis.
85. Same as above explanation
86. Same as above explanation
Indias Fastest Growing Test Prep Portal

Vous aimerez peut-être aussi